The Road To Pass Prometric

You might also like

Download as pdf or txt
Download as pdf or txt
You are on page 1of 460

‫ا‬ ‫ا ا‬

‫ا ر‬ ‫و‬ ‫ز‬ ‫و ا ا‬ ‫ها‬


‫ا آ وو ا‬ ‫د‬ ‫!‬ ‫"‬ ‫‪$‬ي‬
‫‪ 01‬ا ‪ ./‬ا )‬ ‫'& ن‬ ‫‪ ,‬و ‪()* +‬‬ ‫ا‪-‬‬
‫‪ 01 2‬ا ‪./‬‬ ‫) ‪3‬ا‬ ‫اءة ا آ‬ ‫‪ 0‬م‬
‫وا روا ‪ 9‬ت ‪.7‬‬ ‫‪ /‬اءة ا‪&:9$‬‬ ‫‪;)0‬‬

‫"> ا =&‬ ‫‪,‬‬ ‫و‬ ‫ا ‪"1‬‬ ‫ا ‪ '1‬ء‬ ‫و‬


‫?‬

‫وا ‪-‬حو‬ ‫و ‪B‬ا و ‪B‬اآ( ا آ‪A 3‬‬


‫ذن ا‬ ‫ا‪D‬‬

‫زي‬ ‫دأ‬
1-Musle spindle

Golgi tendon organ

2-Cross section of the spinal cord


3- Hallux valgus deformity

4-dermatomes
5-quads muscle L4 , L4 and L5 - knee jerk)
6-ASTNR

7-BAbinski reflex ------- stimulate the sole of the foot


from lateral to medial
12 – > ‫إ‬ F0 (E= ‫آ ن ا‬
8- This 9 – rigid plaster 10 – this is 11 – pt has
soma of sensory neuron
angle is cast to : lumber
a) S.A.CH
a) k angle b) Below pain
a) maintain
knee radiated to
c) C shape of stump
angle cast dorsum of
d) Q c) c) Above foot
angle A.F.O knee
rthosis prothesi a) L1 ,
s with L2
pelvic b) L4.L5
band c) S1.S2
‫آن‬
(E= ‫ا‬
F0
٣ >‫إ‬
,IJ‫أ‬
‫م‬1 ‫ا‬
>*9 ‫ا‬
13- picture number 13

14- picture number 14


15-picture number 15

16- picture number 16


17-muscle in picture :

a- Deltoid supplied by axillary nerve


b- Brachialis supplied by musclocutaneous nerve √
c- Supplied by suprascupular nerve
d- Rotator cuff

18- weakness and pain in insertion of this muscle

a- Supraspinatus tendinitis √
b- Rotator cuff tear
c- infraspinatus tendinitis

19- streching for which muscle

a)lateral rotator √
b)adductor
c)hamstring
d)calf muscle

Strength, Power, and Endurance Training


Muscle performance is defined as “the capacity of a muscle or group of
muscles to generate forces.Muscle strength is the “muscle force exerted
by a muscle or a group of muscles to overcome a resistance under a
specific set of circumstances.Muscle power is “the work produced per
unit of time or the product of strength and speed. Muscle endurance is
“the ability to sustain forces repeatedly or to generate forces over a period
of time.

Muscle Strengthening
Strength training produces a number of neuromuscular changes. There is
an increase in the production of maximal force due to changes in neural
drive (increased motor unit recruitment, increased rate, and
synchronization of firing rate) and changes in muscle (hypertrophy of
muscle fibers, improved metabolic/enzymatic adaptations, increased size
and number of myofibrils, muscle fiber type adaptation with conversion
of type IIB to type IIA). Connective tissue tensile strength and bone
mineral density are increased. Body composition is improved in terms of
body mass ratio
II. PNF Techniques

Reversal of Antagonists:
A group of techniques that allow for agonist contraction followed by
antagonist contraction without pause or relaxation.

• Dynamic Reversals (Slow Reversals):


Utilizes isotonic contractions of first agonists, then antagonists
performed against resistance. Contraction of stronger pattern is selected
first with progression to the weaker pattern. The limb is moved through
full ROM.
Indications: Impaired strength and coordination between agonist and
antagonist, limitations in ROM, fatigue

• Stabilizing Reversals:
Utilizes alternating isotonic contractions of first agonists, then antagonists
against resistance, allowing only very limited ROM.
Indications: Impaired strength, stability and balance, coordination

• Rhythmic Stabilization (RS):


Utilizes alternating isometric contractions of first agonists, then
antagonists against resistance; no motion is allowed.
Indications: Impaired strength and coordination, limitations in ROM;
impaired stabilization control and balance

Repeated Contractions, RC (Repeated Stretch):


Repeated isotonic contractions from the lengthened range, induced by
quick stretches and enhanced by resistance; performed through the range
or part of range at a point of weakness. Technique is repeated (i.e., three
or four stretches) during one pattern or until contraction weakens.
Indications: Impaired strength, initiation of movement, fatigue, and
limitations in active ROM

Combination of Isotonics (Agonist Reversals, AR):


Resisted concentric, contraction of agonist muscles moving through the
range is followed by a stabilizing contraction (holding in the position)
and then eccentric, lengthening contraction, moving slowing back to the
start position; there is no relaxation between the types of contractions.
Typically used in antigravity activities/assumption of postures (i.e.,
bridging, sit-tostand transitions).
Indications: Weak postural muscles, inability to eccentrically control
body weight during movement transitions, poor dynamic postural control
Rhythmic Initiation (RI):
Voluntary relaxation followed by passive movements progressing to
active-assisted and active-resisted movements to finally active
movements. Verbal commands are used to set the speed and rhythm of
the movements. Light tracking resistance is used during the resistive
phase to facilitate movement.
Indications: Inability to relax, hypertonicity (spasticity, rigidity);
difficulty initiating movement; motor planning deficits (apraxia or
dyspraxia); motor learning deficits; communication deficits (aphasia)

Contract-Relax (CR):
A relaxation technique usually performed at a point of limited ROM in
the agonist pattern. Strong, small range isotonic contraction of the
restricting muscles (antagonists) with emphasis on the rotators is
followed by an isometric hold. The contraction is held for 5 to 8 seconds
and is then followed by voluntary relaxation and movement into the new
range of the agonist pattern. Movement can be passive but active
contraction is preferred.

• Contract-relax-active-contraction (CRAC):
Active contraction into the newly gained range serves to maintain the
inhibitory effects through reciprocal inhibition.
Indications: Limitations in ROM

Hold-Relax (HR):
A relaxation technique usually performed in a position of comfort and
below a level that causes pain. Strong isometric contraction of the
restricting muscles (antagonists) is resisted, followed by voluntary
relaxation, and passive movement into the newly gained range of the
agonist pattern.

• Hold-Relax-Active Contraction (HRAC):


Similar to HR except movement into the newly gained range of the
agonist pattern is active, not passive. Active contraction serves to
maintain the inhibitory effects through reciprocal inhibition.
Indications: Limitations in PROM with pain
Stroke
Stroke or brain attack is the sudden loss of neurological function
caused by an interruption of the blood flow to the brain. Ischemic stroke
is the most common type, affecting about 80 percent of individuals with
stroke, and results when a clot blocks or impairs blood flow, depriving
the brain of essential oxygen and nutrients. Hemorrhagic stroke occurs
when blood vessels rupture, causing leakage of blood in or around the
brain.
Galliazi fracture : is fracture of radius with sublaxation of ulna
Montagia fracture : is fracture of ulna with sublaxation of
radius
March fracture : is fracture of metatarsal bones due to long
standing ( stress Fracture )
pulmonary ventilation : exchange between atmospheric air and
lung air
inspiratory capacity : the volume of gas that can be taken into
the lungs in a full
inhalation
total lung capacity : the amount of gas contained in the lung at
the end of a maximal
inhalation
vital capacity : the maximal volume of gas that can be exhaled
from full inhalation
tidal volume : the volume of gas inhaled and exhaled during
one respiratory cycle
residual volume : the amount of gas remaining in the lung at
the end of a maximal
exhalation
stroke volume : the volume of blood ejected from a ventricle at
each beat of the heart,
equal to the difference between the end-diastolic volume and the
end-systolic volume.
end-diastolic volume(EDV) : the volume of blood in each
ventricle at the end of
diastole, usually about 120-130 mL but sometimes reaching
200-250 mL in the normal
heart
end-systolic volume(ESV) : the volume of blood remaining in
each ventricle at the end
of systole, usually about 50-60 mL but sometimes as little as10-
30 mL in the normal
heart.
Smith amputation level : above ankle joint
idiopathic sclerosis : lateral curvature of thoracic spine
extrafusal ms fiber innervated by : Alfa motor neuron
intrafusal ms fiber innervated by : Gamma motor neuron
Milwaukee brace : used in scoliosis of thoracic spine
Boston brace : used in scoliosis of lumbar spine
‫ت‬ PNF
1- D1 flexion diagonal for prepare pt for gait
2- D2 flexion for upper limb in parkinsonism
3- D1 extension for upper limb in parkinsonism
4- D2 extension and D1 flexion to facilitate rolling in spinal cord
injury .
3- D2 flexion  rotator cuff, impingement, supraspinatous tendinitis
4- D2extention  pitcher ! ‫ا‬
5- Rhythmic initiation  parkinsonism
6- Rhythmic stablization  increase stability ( Attaxia )
Heart layers

The epicardium : is a thin layer of connective tissue and fat, and serves
as an additional layer of protection for the heart, under the pericardium.

The myocardium : is the muscle tissue of the heart, composed of


cardiac muscle cells called cardiomyocytes, which contract like other
muscle cells, but also conduct electricity to
coordinate contraction.

The endocardium : is composed of endothelial cells which provide a


smooth, non-adherent surface for blood collection and pumping and
may help regulate contractility.
Types of erb's palsy

Erb's child : hasn't MORO Reflex

Erb's palsy or Erb–Duchenne palsy  is a paralysis of the arm caused


by injury to theupper group of the arm's main nerves, specifically the
severing of the upper trunk C5–C6 nerves

Klumpke’s Palsy : involves C7 and T1. There is weakness of the wrist


and finger flexors and of the small muscles of the hand. Unfortunately,
there is no specific treatment for this type of Erb’s palsy

Voluntary muscular contractions

Concentric contraction : the force generated is sufficient to overcome


the resistance, and the muscle shortens as it contracts. This is what
most people think of as a muscle contraction

Eccentric contraction : the force generated is insufficient to overcome


the external loadon the muscle and the muscle fibers lengthen as they
contract. An eccentric contraction is usedas a means of decelerating a
body part or object, or lowering a load gently rather than letting it
drop

Isometric contraction :the muscle remains the same length. An


example would beholding an object up without moving it; the muscular
force precisely matches the load, and no movement results.

Isotonic contraction : the tension in the muscle remains constant


despite a change in muscle length. This can occur only when a muscle's
maximal force of contraction exceeds the
total load on the muscle

Isokinetic contraction : the muscle contraction velocity remains


constant, while force is allowed to vary. True isovelocity contractions
are rare in the body, and are primarily an analysis method used in
experiments on isolated muscles that have been dissected out of the
organism

‫ت‬ us
** Pulsed us has no thermal effect
** continous has thermal effect (heat)

** frequency has reverse relation with distance so 1MHz is


deeper than 3 MHz so to HEAT a DEEP muscle use continous 1
MHz us used for treating adhesion and muscle strain specially
in fatty patient because us not high absorbed by fat but high
absorbed by muscle , nerve cell and protien. Contious for
stiffness or adhesion or muscle spasm But non pulsed for
strain or strain or tendinitis to promote healing no need for
heating effect . Frequency 1 and 3 mhz.

Short wave not used in fatty patient because highly absorbed


by fat and cause burn .

Short wave : frequency 27.12 mhz and wave length 11.06 m.

Microwave : highly absorbed by water content .


Infrared : 1- near infrared ( luminous )

2- far infrared ( non luminous )

Ultraviolet : 3 types

Uva : from 315nm to 390 nm ( long )

Uvb: from 280 to 315 nm ( mid )

Uvc: less than 280 nm ( short )

Erythema : due to histamine

Pigmenta9on : due to melanin a:er 2 days of exposture to


ultraviolet ray
Erythema of ultraviolet :
UE Muscles
Origins, Insertions, Actions and Innervations of UE Muscles

Muscle Origin Insertion Innervation Action

external occipital
protuberance medial scapular
lateral 1/3 of
1/3 of superior nuchal spinal accessory elevation upward
Upper clavicle and
line ligamentum CXI and ventral rotation of
Trapezius acromion
nuchae spinous rami of C3,4. scapula scapular
process
process of 7th cervical retraction
vertebrae

superior
spinal accessory scapular
Middle spinous process of border of
CXI and ventral retraction and
Trapezius thoracic vertebrae 1-5 spine of
rami, C3,4 elevation
scapula

scapular
spinous process of medial 1/3 of spinal accessory
Lower depression,
thoracic vertebrae 6- spine of CXI and ventral
Trapezius retraction and
12 scapula rami, C3, 4
upward rotation

vertebral
border of scapular
scapula, dorsal scapular C5, elevation,
Levator transverse process of
between and ventral rami downward
Scapulae cervical 1-4
superior angle of C3,4 rotation, and
and scapular retraction
spine

scapular
medial border
retraction,
Rhomboid Spinous process of of scapula
dorsal scapular, C5 elevation,
Major thoracis 2-5 from spine to
downward
inferior angle
rotation

medial border scapular


lower part of
of capula at retraction,
Rhomboid ligamentum nuchae,
the root of the dorsal scapular, C5 elevation and
Minor spinous process of C7
spine of the downward
and T1
scapula rotation
A: abduction,
horizontal flexion
A: lateral 1/3 of
and med.
clavicle, M: acromion deltoid
Deltoid Axillary, C5, 6 rotation; M:
process, P: inferior tuberosity
abduction, P:
spine of scapula
abduction, horiz
ext., lat rotation.

costal surface
scapular
superior lateral of vertebral
Serratus Long thoracic, protraction,
surfaces of upper 8 or border along
Anterior C5,6,7. upward rotation,
9 ribs inferior angle
stabilization
of scapula

groove on inf. depresses clavicle


subclavian branch
1st rib at the jct of clavicle (btwn and pulls
Subclavius of brachial plexus,
costal cartilage costoclavicular inferiorly and
C5,6
and conoid lig) anteriorly

protraction with
fixed ribs,
elevates ribs with
medial pectoral
coracoid coracoid fixed
Pectoralis anterior surface of nerve from
process of (scapular
Minor 3rd, 4th, 5th ribs brachial plexus,
scapula depression,
C6, 7, 8
downward
rotation,
stabilization)

lateral lip of
the
C: ant surface of med. flexion,
intertubercula
1/2 clavicle; S: C:lateral pectoral adduction, horiz.
r sulcus of the
Pectoralis sternum to 7th rib, C5, 6 S: lateral and flexion, medial
humerus
Major cartilages of true ribs medial pectoral, rotation. S also
(bicipital
and aponeurosis of C7,8, T1. extends flexed
groove); crest
ext. obliques humerus
of greater
tubercle

Broad aponeurosis flat tendon


extends, retracts
that originates on the that twists
Thoracodorsal and medially
upon itself to
Latissimus Dorsi spinour processes of from brachial rotates the
T6-12 and L's, post. insert into the
plexus, C6,7,8 humerus at the
crest of ilium, post intertubercula
shoulder
surface of sacrum, r sulcus
lower 3-4 ribs and
attachment to inf.
angle of scapula

dorsal surface of adducts and


inferior angle of the intertubercula medially rotates
lower subscapular,
Teres Major scapula on lower 1/3 r groove humerus at the
C5-7
of scapular axillary (medial lip) shoulder, extends
border shoulder

lateral rotation of
post. aspect of
suprascapular, C5- humerus,
Infraspinatus infraspinatus fossa greater
6 stabilization of
tubercle
joint.

superior abduction of
medial 2/3 of surface of suprascapular, C5- humerus,
supraspinatus
supraspinatus fossa greater 6 stabilization of
tubercle joint.

lateral rotation of
superior 2/3 of dorsal inf. aspect of
humerus;
teres minor surface of axillary greater Axillary, C5-6
stabilization of
border of scapula tubercle
head of humerus.

medial rotation
entire anterior upper and lower
lesser tubercle of the humerus,
subscapularis surface of the subscapular,
of humerus stabilization of
subscapular fossa C5,6,7
joint

roughening in flexes arm at


Coracobrachiali musculocutaneous
Coracoid Process the medial glenohumeral
s , C5-7
shaft joint

supraglenoid tubercle
on scapula(long) and radial musculocutaneous forearm flexion
Biceps Brachii
coracoid process tuberosity , C5-6 and supination
(short)

anterior aspect of the


musculocutaneous
humerus and tuberosity of
Brachialis , C5-6 (and some forearm flexion
adjacent the ulna
C7 from Radial N.)
intermuscular septae

Triceps Brachii infraglenoid olecranon radial, C6-8 extension of


tubercle(long), process of forearm (long
superior 1/2 of post. ulna also adducts
humerus(lat), inferior shoulder)
2/3 of post. humerus
(medial)

lateral
olecranon; extends elbow
Posterior lateral
Anconeus upper 1/4 of radial, C7-8 (and stabilize
epicondyle
post surface of during pronation)
ulna

medial
epicondyle(humeral) middle 1/3 pronation and
Pronator Teres median, C6-7
and medial coronoid lateral radius flexion
process (ulnar)

anterior
surface of
Flexor Carpi flexion and radial
medial epicondyle base of 2nd median, C6-7
Radialis deviation
and 3rd
metacarpal

Palmaris palmar
medial epicondyle median, C7-8 flexion at wrist
Longus aponeurosis

Flexor Carpi posterior wrist flexion and


medial epicondyle ulnar, C7-8 and T1
Ulnaris border of ulna ulnar deviation

medial epicondyle
Flexor and coronoid 4 tendons to wrist flexion,
Digitorum process(humeroulnar) base of middle median, C8 and T1 MCP jt and IP jt
Superficialis , oblique line of radius phalanges flexion
(radial)

Anteromedial surface
Flexor of ulna and palmar median (lat. half)
flexes DIP, MP
Digitorum anteromedial surface surface of and ulnar (medial
and wrist
Profundus of interosseus distal phalynx half), C8 and T1
membrane

anterior surface of base of distal


Flexor Pollicus median(ant. flexes IP of
radius and radial 1/2 phalynx of
Longus interosseus), C7-8 thumb/mp
of interosseus thumb

linear ridge on distal distal ant. median nerve pronation


Pronator
anterior surface of surface of (ant. interosseus),
Quadratus ulna radius C7-8

flex neutral
proximal
lateral aspect forearm
Brachioradialis supracondylar ridge radial, C5-6
of distal radius (pronate/supinat
of humerus
e to neutral)

posterior
Extensor Carpi extension and
lateral epicondyle surface of 3rd deep radial, C7-8
Radialis Brevis radial deviation
metacarpal

4 tendons,
Extensor split 1 to post. interosseus
lateral epicondyle extends fingers
Digitorum dorsal MP and C7-8
2 to DP

Extensor Digiti PP of little post. interosseus, extends little


lateral epicondyle
Minimi finger C7-8 finger

post. surface extends and


Extensor Carpi lateral epicondyle and post. interosseus,
of 5th ulnarly deviates
Ulnaris post. border of ulna C7-8
metacarpal wrist

lateral epicondyle,
radial collateral lig., lateral surface
Supinator Deep Radial, C6-7 supination
annular lig, supinator of radius
crest of the ulna

prox. post ulna, lateral base of


Abductor post. interosseus, abducts MCP jt at
radius, interosseus 1st
Pollucis Longus C7-8 thumb
membrane metacarpal

post. radius and post.


Extensor proximal post. interosseus, extends proximal
interosseus
Pollucis Brevis phalynx base C7-8 phalynx
membrane

mid 1/3 of ulna and


Extensor post. interosseus,
interosseus DIP Extends DIP
Pollucis Longus C7-8
membrane

Posterior ulna and ext. hood of Extends


post. interosseus,
Extensor Indicis interosseus index (ex. digit phalanges of
C7-8
membrane tendon) index

Extensor Carpi Distal lateral dorsal surface Radial, C6-7 wrist extension
Radialis Longus supracondylar ridge of base of 2nd and radial
metacarpal deviation with
coulpled with
flexor carpi
radialis)

Hip muscles/actions
Hip muscles and actions

Question Answer

Name the muscle for this action: Combination of


Tensor fascia latae
flexion & abduction

Name the nerve & spinal root for: Tensor fascia Superior gluteal nerve and spinal root: L4
latae L5

Name the muscle for this action: Combination of


Sartorius
flexion abduction and lateral rotation

Name the nerve & spinal root for: Sartorius


Femoral nerve and spinal root: L2 L3
Iliopsoas (psoas part) & Pectineus

Name the muscles for this action: Flexion Rectus femoris iliopsoas pectineus

Name the nerve & spinal root for: Rectus


Femoral nerve and spinal root: L2 L3 L4
femoris

Name the nerve & spinal root for: Iliopsoas


Anterior rami nerve and spinal root: L2 L3
(iliacus part)

Gluteus maximus semitendinosus


Name the muscles for this action: Extension semimembranosus biceps femoris (long
head)

Name the nerve & spinal root for: Gluteus Inferior gluteal nerve and spinal root: L5 S1
maximus S2

Name the nerve & spinal root for:


semitendinosus semimembranosus biceps SciaFc nerve and spinal root: L5 S1 S2
femoris (long head)

Name the muscle for this action:


Gluteus maximus
Hyperextension
Name the muscles for this action: Abduction Gluteus medius & gluteus minimus

Name the nerve & spinal root for: Gluteus Superior gluteal nerve and spinal root: L5
medius & gluteus minimus S1 S2

Pectineus adductor longus adductor brevis


Name the muscles for this action: Adduction
adductor magnus & gracilis

Name the nerve & spinal root for: Adductor


Obturator nerve and spinal root: L2 L3 L4
longus adductor brevis

Name the nerve & spinal root for: Adductor


Obturator nerve and spinal root: L3 L4
magnus

Name the nerve & spinal root for: Gracilis Obturator nerve and spinal root: L2 L3

Name the muscle for this action: Medial


Gluteus minimus
Rotation

Name the muscle for this action: Lateral


Gluteus maximus & deep rotators
Rotation

Name the nerves & spinal roots for: Deep Deep rotator muscles for the Obturator
rotators - Obturator externus Obturatoe Anterior rami (2)obturatoe internus &
internus Gemellus superios Quadratus femoris (2)quadratus femoris nerves and spinal
Gemellus inferior & Piriformis root: L3 L4 L5 S1 S2

Name the proximal/distal attachments & the Proximal attachment: Rami of pubis &
nerve for Deep Rotator Muscle: Obturator ischium Distal attachment: Trochanteric
externus fossa Nerve: Obturator

Name the proximal/distal attachments & the Proximal attachment: Rami of pubis &
nerve for Deep Rotator Muscle: Obturator ischium Distal attachment: Greater
internus trochanter Nerve: to obturator internus

Name the proximal/distal attachments & the Proximal attachment: Ischial tuberosity
nerve for Deep Rotator Muscle: Quadratus Distal attachment: Intertrochanteric crest
femoris Nerve: to quadratus femoris

Proximal attachment: Sacrum Distal


Name the proximal/distal attachments & the
aGachment: Greater trochanter Nerve: S1
nerve for Deep Rotator Muscle: Piriformis
S2 segments

Name the proximal/distal attachments & the Proximal attachment: Ischium Distal
nerve for Deep Rotator Muscle: Gemellus attachment: Greater trochanter Nerve: to
superior obturator internus
Name the proximal/distal attachments & the Proximal attachment: Ischial tuberosity
nerve for Deep Rotator Muscle: Gemellus Distal attachment: Greater trochanter
inferior Nerve: to quadratus femoris

Name the action for: Tensor fascia latae Combination of flexion & abduction

Name the muscle for the Superior gluteal nerve


Tensor fascia latae
and spinal root: L4 L5

Combination of flexion abduction and


Name the action for: Sartorius
lateral rotation

Name the muscles for the Femoral nerve and


Sartorius Iliopsoas (psoas part) & Pectineus
spinal root: L2 L3

Name the action for: Rectus femoris iliopsoas


Flexion
pectineus

Name the muscle for the Femoral nerve and


Rectus femoris
spinal root: L2 L3 L4

Name the muscle for the Anterior rami nerve


Iliopsoas (iliacus part)
and spinal root: L2 L3

Name the action for: Gluteus maximus


semitendinosus semimembranosus biceps Extension
femoris (long head)

Name the muscle for the Inferior gluteal nerve


Gluteus maximus
and spinal root: L5 S1 S2

Name the muscle for the Sciatic nerve and spinal semitendinosus semimembranosus biceps
root: L5 S1 S2 femoris (long head)

Name the action for: Gluteus maximus Hyperextension

Name the action for: Gluteus medius & gluteus


Abduction
minimus

Name the muscles for the Superior gluteal nerve


Gluteus medius & gluteus minimus
and spinal root: L5 S1 S2

Name the action for: Pectineus adductor longus


Adduction
adductor brevis adductor magnus & gracilis

Name the muscles for the Obturator nerve and Adductor longus adductor brevis
spinal root: L2 L3 L4

Name the muscle for the Obturator nerve and


Adductor magnus
spinal root: L3 L4

Name the muscle for the Obturator nerve and


Gracilis
spinal root: L2 L3

Name the action for: Gluteus minimus Medial Rotation

Name the action for: Gluteus maximus & deep


Lateral Rotation
rotators

Name the deep rotator muscles for the Deep rotators - Obturator externus
Obturator Anterior rami (2)obturatoe internus & Obturator internus Gemellus superios
(2)quadratus femoris nerves and spinal root: L3 Quadratus femoris Gemellus inferior &
L4 L5 S1 S2 Piriformis

which of the quadriceps muscles crosses


rectus femoris
two joints?

which movement is created by all muscles of the


extension of the knee
quadriceps group?

what is the common attachment site for all of


tibial tuberosity
the quadriceps?

what attachment site do all of the hamstrings


ischial tuberosity
have in common?

semimembranosus is deep to which muscle? semitendinosus

which cable-like band of connective tissue can


be isolated just anterior to the distal biceps iliotibial tract
femoris tendon?

which is the origin of the rectus femoris? anterior inferior iliac spine (AIIS)

which is the insertion of the quadriceps femoris


tibial tuberosity
group?

gluteal tuberosity lateral lip of linea aspera

which of the following is an action of the vastus


extend the knee
lateralis?
which is the origin of the vastus medialis? medial lip of linea aspera

which of the following is an acton of the vastus


extend the knee
medialis?

which is the origin of the vastus intermedius? anterior and lateral shaft of the femur

which of the following is an action of the vastus


extend the knee
intermedius?

which is the origin of the short head of the


lateral lip of linea aspera
biceps femoris?

which is the insertion of the biceps femoris? head of the fibula

which of the following is an action of the biceps


laterally rotate the hip
femoris?

which is the origin of the semitendinosus? ischial tuberosity

which of the following is an action of the


flex the knee
semitendinosus?

which is the origin of the semimembranosus? ischial tuberosity

which is the insertion of the semimembranosus? posterior aspect of medial condyle of tibia

which gluteal muscle medially rotates and flexes


gluteus minimus
the hip?

how many muscles are there in the adductor


5
group?

gracilis is the only adductor that crosses which


tibiofemoral
joint?

which action is common to all muscles in the


medial rotation of the hip
adductor group?

the origin of which muscle can be located


between the ischial tuberosity and the adductor adductor magnus
tubercle?

which cable like band of connective tissue can


be isolated just anterior to the distal biceps iliotibial tract
femoris tendon?
which of the following is a part of the origin of
edge of sacrum
the gluteus maximus?

which of the following is a part of the insertion


gluteal tuberosity
of the gluteus maximus?

which of the following is an action of the gluteus


adduct the hip
maximus' lower fibers?

which of the following is a part of the origin of


posterior and anterior gluteal lines
the gluteus medius?

which is the insertion of the gluteus medius? greater trochanter

which of the following is an action of the gluteus


flex the hip
medius?

gluteal surface of the ilium between


which is the origin of the gluteus minimus?
anterior and inferior gluteal lines

which is the insertion of the gluteus minimus? anterior border of greater trochanter

which of the following is an action of the gluteus


abduct the hip
minimus?

which is a part of the origin of the adductor


superior ramus of the pubis
magnus?

medial lip of linea aspera and adductor


which is the insertion of the adductor magnus?
tubercle

which of the following is an action of the


extend the hip
posterior fibers of the adductor magnus?

which is the origin of the adductor longus? pubic tubercle

which is the insertion of the adductor longus? medial lip of linea aspera

which of the following is an action of the


assist to flex the hip
adductor longus?

which is the origin of the adductor brevis? inferior ramus of pubis

which is the insertion of the adductor brevis? pectineal line and medial lip of linea aspera

which of the following is an action of the


medially rotate the hip
adductor brevis?
which is the origin of the pectineus? superior ramus of pubis

which is the insertion of the pectineus? pectineal line of femur

which of the following is an action of the


laterally rotate the hip
pectineus?

which is the origin of the gracilis? iliac crest

which of the following is an action of the


medially rotate the flexed knee
gracilis?

posterior to the ASIS iliac crest

which is the insertion of the tensor fasciae


iliotibial tract
latae?

which of the following is an action of the tensor


abduct the hip
fasciae latae?

which is the origin of the sartorius? anterior superior iliac spine (ASIS)

which of the following is an action of the


flex the hip
sartorius?

which is the origin of the piriformis? anterior surface of sacrum

which is the insertion of the piriformis? greater trochanter

which of the following is an action of the


abduct the hip when the hip is flexed
piriformis?

which is the origin of the quadratus femoris? lateral border of ischial tuberosity

between the greater and lesser trochanters? intertrochanteric crest

obturator membrane and inferior surface


which is the origin of the obturator internus?
of obturator foramen

which is the origin of the obturator externus? superior and inferior rami of pubis

which is the insertion of the obturator externus? trochanteric fossa of femur

which is the origin of th gemellus superior? ischial spine

which is the insertion of the gemellus superior? upper border of greater trochanter
which is the origin of the gemellus inferior? ischial tuberosity

which is the insertion of the gemellus inferior? upper border of greater trochanter

bodies and transverse processes of lumbar


which is the origin of the psaoa major?
vertebrae

which is the insertion of the psoas major? lesser trochanter

which of the following is an action of the psoas


medially rotate the hip
major?

which is the origin of the iliacus? iliac fossa

which is the insertion of the iliacus? lesser trochanter

SUGGESTED ORDER OF
MUSCLE TESTS
1. Supine
Toe extensors
Toe flexors
Tibialis anterior
Tibialis posterior
Peroneals
Tensor fasciae latae
Sartorius
Iliopsoas
Abdominals
Neck flexors
Finger flexors
Finger extensors
Thumb muscles
Wrist extensors
Wrist flexors
Supinators
Pronators
Biceps
Brachioradialis
Triceps (supine test)
Pectoralis major, upper part
Pectoralis major, lower part
Pectoralis minor
Medial rotators of shoulder (supine test)
Teres minor and infraspinatus
Lateral rotators of shoulder (supine test)
Serratus anterior
Anterior deltoid (supine test)
2. Side-Lying
Gluteus medius
Gluteus minimus
Hip adductors
Lateral abdominals
3. Prone
Gastrocnemius and plantaris
Soleus
Hamstrings, medial and lateral
Gluteus maximus
Neck extensors
Back extensors
Quadratus lumborum
Latissimus dorsi
Lower trapezius
Middle trapezius
Rhomboids
Posterior deltoid (prone test)
Triceps (prone test)
Teres major
Medial rotators of shoulder (prone test)
Lateral rotators of shoulder (prone test)
4. Sitting
Quadriceps
Medial rotators of hip
Lateral rotators of hip
Hip flexors (group test)
Deltoid, anterior, middle, and posterior
Coracobrachialis
Upper trapezius
Serratus anterior (preferred test)
5. Standing
Serratus anterior
Ankle plantar flexors

Grades of muscle test

Gone—no contraction felt.


Trace—muscle can be felt to tighten but cannot
produce movement.
Poor—produces movement with gravity eliminated
but cannot function against gravity.
Fair—can raise the part against gravity.
Good—can raise the part against outside resistance
as well as against gravity.
Normal—can overcome a greater amount of resistance
than a good muscle.
\
Special tests
cervical
Foraminal Compression (Spurling) Test
Pt bends or side flexes head to one side. Examiner carefully presses
straight down on the head.

-- Test is positive if pain radiates into the arm toward which the
head is flexed during compression.
** This indicates pressure on the nerve root.
Distraction Test
Place one hand under the pt's chin and the other hand around the
occiput. The examiner slowly lifts the pt's head.
-- Test is positive if the pain is relieved or decreases when the head
is lifted or distracted.
** This indicates pressure on the nerve roots.
** Test may also be used to check the shoulder. Pt moves the arms
while traction is applied and the symptoms are often relieved or
lessened in the shoulder.
** This indicates pressure on the cervical nerve root.
Brachial Plexus Tension (Upper Limb Tension) Test
Patient lies supine and the examiner passively abducts the
patient's arm just behind the coronal plane to the point just short
of pain. The examiner then passively externally rotates the
glenohumeral joint to a position just short of pain while the elbow
is flexed. This shoulder position is held and the forearm supinated.
While these positions are held, the elbow is passively extended.

** Reproduction of symptoms implies problems of cervical origin


(C5 - C7 ), primarily C5 nerve root.
** Also, if the cervical spine is then flexed, symptoms that then
include aching in the cubital fossa extending to the forearm
(anterior and radial aspects) and onto the radial side of the hand
and tingling in the thumb and lateral three fingers will increase.
Side flexion of the head to the test side will decrease the symptoms
70% of the Fme, whereas side flexion to the opposite side will
increase the symptoms.
Shoulder Depression Test
Examiner side bends the pt's head while applying a downward
pressure on the opposite shoulder.
-- Test is positive, if pain increases.
** This indicates irritation or compression of the nerve roots,
foraminal encroachments such as osteophytes in the area, or
adhesions around the dural sleeves of the nerve and adjacent joint
capsule on the side being stretched.
Jackson's Compression Test
Pt rotates the head to one side. Examiner then carefully presses
straight down on the head. Repeat to other side.
-- Test is positive if on testing, pain radiates into the arm.
** This is indicative of nerve root pressure with pain indicating
nerve root affected
Valsalva Test
Examiner asks the pt to take a deep breath and hold it while
bearing down, as if moving the bowels.
-- Test is positive, if pain increases.
** This may be indicative of increased intrathecal pressure and is
usually due to a space-occupying lesion, such as a HNP, a tumor, or
osteophyte formation.
** Perform with care and caution. Pt may become dizzy and pass
out while performing the test as the procedure can block the blood
supply to the brain.
Vertebral Artery (Cervical Quadrant) Test
Pt supine, Examiner passively takes the pt's head and neck into
extension and side flexion. When this movement is achieved, the
examiner rotates the pt's head to the same side and holds it for
approximately 30 seconds.
-- Test is positive, if referring symptoms are noted to the side to
which the head is taken.
** This is indicative of nerve root compression in the lower cervical
spine. To test the upper cervical spine, the examiner pokes the pt's
chin and follow with extension, side flexion, and rotation. If
dizziness or nystagmus occurs, this indicates that the vertebral
arteries are being compressed.
** Indicates whether or not there is pathology of C7 (facial) nerve
Dizziness Test
Examiner actively rotates the pt's head as far as possible to the
right then to the left. Pt's shoulders are then actively rotated as far
as possible to the right, then to the left while keeping the eyes
looking straight ahead.
-- If pt experiences dizziness in both cases then the problem lies in
the vertebral arteries. If dizziness persists only when the head is
rotated.
** This test is indicative of a problem that lies in the semicircular
canals.
*2*

* Shoulder Tests *
TESTS FOR ANTERIOR SHOULDER INSTABILITY:

Anterior Drawer Test


Pt is lying supine and the examiner places the hand of the affected
shoulder in the examiner's axilla, holding the pt's hand with the arm
so that the pt remains relaxed. The shoulder being tested is
abducted between 80-120 degrees ; forward flexed 0-20 degrees;
and laterally rotated 0-30 degrees. The examiner then stabilizes the
pt 's scapula with the opposite hand pushing the spine of the
scapula forward with the index and middle finger. The examiner's
thumb exerts counter pressure on the pt's coracoid process. Using
the arm that holds the pt's hand, the examiner places the hand
around the pt's relaxed upper arm and draws the humerus forward.

-- Test is considered positive, if movement is accompanied by a click


and/or patient apprehension. The amount of movement available is
compared with that of the normal shoulder.
** This test is indicative of anterior shoulder instability
Protzman Test
Pt is siOng and the examiner abducts the pt's arm to 90 degrees
and supports the arm against the examiner's hip so that the pt's
shoulder muscles are relaxed. The examiner palpates the anterior
aspect of the head of the humerus with the fingers of one hand
deep in the pt's axilla while the fingers of the other hand are placed
over the posterior aspect of the humeral head. The examiner then
pushes the humeral head anteriorly and inferiorly.
-- This test is positive, if this movement causes pain and palpation
indicates abnormal anteroinferior movement.

** This test is indicative of anterior instability


Anterior Instability Test
The examiner stands behind the shoulder being examined while
the pt sits. The examiner places the examiner's "inside" hand over
the shoulder so that the index finger is over the head of the
humerus anteriorly and the middle finger over the coracoid
process. The thumb is placed over the posterior humeral head. The
examiner's other hand grasps the pt's wrist and carefully abducts
and laterally rotates the arm.
-- Test is positive if, on movement of the arm, the finger palpating
the anterior humeral head moves forward. When the arm is
returned to the starting position, the index finger will return to the
starting position as the humeral head glides backward.
** This test is indicative of anterior shoulder instability
Rockwood Test
Examiner stands behind the seated patient. With the arm by their
side, the examiner laterally rotates the shoulder. The arm is
abducted to 45 degrees, and passive lateral rotaFon is repeated.
The same procedure is repeated at 90 degrees and 120 degrees.

-- Test is positive, if the patient shows marked apprehension with


posterior pain when the arm is tested at 90 degrees. At 45 degrees
and 120 degrees, the paFent will show some uneasiness and some
pain. At 0 degrees, there is rarely apprehension.
** This test is indicative of anterior shoulder instability
Rowe Test
Patient lies supine and places the hand behind the head. The
examiner places one hand (clenched fist) against the posterior
humeral head and pushes up while extending the arm slightly.
-- Test is positive, if a look of pain or apprehension is noted.
** This test is indicative of anterior shoulder instability
Fulcrum Test
PaFent lies supine with the arm abducted to 90 degrees. Examiner
places one hand under the glenohumeral joint to act as a fulcrum.
The examiner then extends and laterally rotates the arm gently
over the fulcrum.
-- Test is positive if apprehension is noted.
** This test is indicative of anterior shoulder instability
Apprehension (Crank) Test For Anterior Dislocation
Examiner abducts and laterally rotates the patient's shoulder
slowly.
-- Test is positive, if a look or feeling of apprehension or alarm is
present on the patient's face and they resist further motion. The
patient may also state that the feeling experienced is what it felt
like when the shoulder dislocated previously.
** Imperative that this test is performed slowly or the humerus
may dislocate
** This test is indicative of anterior shoulder dislocation
TESTS FOR POSTERIOR SHOULDER INSTABILITY:

Posterior Drawer Test


Patient lies supine. Examiner stands at the level of the shoulder and
grasps the patient's proximal forearm with one hand, flexing the
paFent's elbow to 120 degrees and the shoulder to 80 to 120
degrees of abducFon and 20 to 30 degrees of forward flexion. With
the other hand, the examiner stabilizes the scapula by placing the
index and middle fingers on the spine of the scapula and the thumb
on the coracoid process. The examiner then rotates the forearm
medially and forward flexes the shoulder to 60 to 80 degrees while
at the same time taking the thumb of the other hand off the
coracoid process and pushing the head of the humerus posteriorly.
The head of the humerus can be felt by the index finger of the same
hand.
-- Test is positive, if patient exhibits apprehension. Test is usually
pain free.
** This test is indicative of posterior instability
Jerk Test
PaFent sits with the arm medially rotated and forward flexed to 90
degrees. The examiner grasps the patient's elbow and axially loads
the humerus in a proximal direction. While maintaining axial
loading, the arm is moved horizontally in adduction.
-- Test is positive, if a sudden jerk is produced as the humeral head
slides off the back of the glenoid. As the arm is returned to 90
degrees a second jerk may be felt as the humeral reduces.
** This test is indicative of recurrent posterior instability
Posterior Apprehension Test
Patient lies supine and the examiner forward flexes and medially
rotates the patient's shoulder. Examiner then applies posterior
pressure on the patient's elbow. Test should also be performed at
90o of abducFon, while the examiner palpates the humeral head
with one hand and the other pushes the head posteriorly.

-- Test is positive, if a look of apprehension or alarm is noted on the


patient's face and further motion is resisted and if the humeral
head moves more than 50% of its size. Clunk may be present.

** Test is indicative of posterior instability

TEST FOR INFERIOR AND MULTIDIRECTIONAL


INSTABILITY:
Inferior Shoulder Instability Test (Sulcus Sign)
Patient is sitting with the arm at the side relaxed. The examiner
grasps the patient's forearm below the elbow and pushes the arm
distally.
-- Test is positive, if the presence of a sulcus sign occurs.
** This test is indicative of inferior instability
Feagin Test
The paFent stands with his arm abducted to 90 degrees and the
elbow is extended resting on the top of the examiner's shoulder.
Examiner's hands are clasped together over the pt's humerus
between the upper and middle third. The examiner pushes the
humerus down and forward.
-- Test is positive, if a look of apprehension is seen on the pt's face.
** This test is indicative of anteroinferior instability
Rowe Test For Multidirectional Instability
The paFent stands forward flexed 45 degrees at the waist with the
arms relaxed, pointing to the floor. Examiner places one hand over
the pt's shoulder so the index and middle fingers sit over the
anterior aspect of the humeral head and the thumb sits over its
posterior aspect. The examiner then pulls the arm down slightly. To
test for anterior instability, the humeral head is pushed anteriorly
with the thumb while the arm is extended 20-30 degrees from the
vertical position. To test for posterior instability, the humeral head
is pushed posteriorly with the index and middle fingers while the
arm is flexed 20-30 degrees from the verFcal posiFon. To test for
inferior instability, more traction is applied to the arm and the
sulcus sign is evident.

TESTS FOR OTHER SHOULDER JOINTS:

Acromioclavicular Shear Test


The patient is in a sitting position and the examiner cups his hands
over the deltoid muscle with one hand on the clavicle and the other
hand on the spine of the scapula. The examiner then squeezes the
heels of the hands together.
-- Test is positive, if pain or abnormal motion at the
acromioclavicular joint is present.
** Test is indicative of acromioclavicular joint pathology

TEST FOR MUSCLE/TENDON PATHOLOGY:

Yergason's Test

With the pt's elbow flexed to 90 degrees and stabilized against the
thorax with the forearm pronated, the examiner resists supination
while the patient also laterally rotates the arm against resistance.

-- Positive result elicits tenderness in the bicipital groove or the


tendon may pop out of the groove.
** Indicative of bicipital tendinitis
Speed's Test (Biceps or Straight Arm Test)
Examiner resists shoulder forward flexion by the pt while the pt's
forearm is supinated and the elbow is completely extended.
-- Positive test elicits increased pain tenderness in the bicipital
groove.
** Indicative of bicipital tendinitis
Transverse Humeral Ligament Test
Examiner abducts and externally rotates the patient's shoulder as
the examiner fingers are placed in the bicipital groove while the
shoulder is externally rotated
-- Positive test, if the examiner feels the tendon snap in/out of the
groove as they externally rotate.
** Indicates tearing or rupture of the transverse ligament
TESTS FOR ROTATOR CUFF TEARS AND IMPINGEMENT:

Clunk Test
Patient lies supine. Examiner places one hand on the posterior
aspect of the shoulder over the humeral head. The examiner's
other hand holds the humerus at the elbow. Examiner fully
abducts the arm over the patient's head. The examiner then
pushes anteriorly with the hand over the humeral head while the
other hand rotates the humerus into lateral rotation.
-- Test is positive, if a "clunk" or grinding occurs.
** This test is indicative of a tear of the labrum or instability
-- If the above maneuver is followed with horizontal adduction
that will relocate the humerus, a clunk or click may also be heard
Drop Arm Test
Examiner abducts the patient’s shoulder to 90 degrees and then
asks them to slowly lower it to their side in the same arc of
movement.

-- Positive test is indicated if the patient is unable to return the


arm to the side slowly or has severe pain when attempting to do
so.
** Positive results are indicative of a tear in the rotator cuff
complex
Supraspinatus Test
PaFent's shoulder is abducted to 90 degrees in a neutral posiFon
and the examiner gives resistance to abduction. The shoulder is
then internally rotated and angled forward 30 degrees, so that
the thumb points to the floor. Resistance to abduction is given
again with the examiner looking for pain and weakness.
-- Positive sign is indicated, if pain/weakness wit resisted
abduction with internal rotation.
** Indicates a tear of the supraspinatus tendon/ muscle
Impingement Sign/Test
Examiner forcibly abducts the patient's arm through forward
flexion causing a jamming of the greater tuberosity against the
anterior lateral acromial surface.
-- Positive test, if patient's face shows pain upon motion.
** Indicates an overuse injury to the supraspinatus and
sometimes the biceps tendon
Hawkins-Kennedy Impingement Test
Examiner forward flexes the arm to 90 degrees and then forcibly
internally rotates the shoulder. This movement pushes the
supraspinatus tendon against the anterior surface of the
coracoacromial ligament.
-- Positive, if pain is felt on forward flexion of the arm.
** Indicate a test for supraspinatus tendinitis
*3*

*Thoracic outlet Syndrome Tests*


Allen Test
PaFent siOng, examiner flexes the paFent's elbow to 90 degrees
and shoulder abducted 90 degrees with external rotaFon. PaFent
rotates his head away from test side while the examiner palpates
the radial pulse.
-- Positive, if pulse disappears.
** Indicates Thoracic Outlet Syndrome
Adson Maneuver - Thoracic Outlet Syndrome
Patient sitting, the examiner locates the radial pulse. Patient
rotates the head toward the test shoulder. Patient extends his
head, while the examiner extends and externally rotates the
shoulder.
Patient takes a deep breath and holds it.
-- Positive, if pulse disappears.
** Indicates Thoracic Outlet Syndrome
Halstead Maneuver
Examiner finds the radial pulse, and applies a downward traction
on the arm, while his neck is hyperextended and head rotated to
the opposite side.
-- Positive, if absence or disappearance of pulse.
** Indicates Thoracic Outlet Syndrome
*4*
*Elbow Tests *
Ligamentous instability - Collateral ligaments
Patient's arm is stabilized at the elbow and wrist with a varus then
valgus force at the elbow.
-- Positive, if signs of laxity or increased mobility or pain.
** Indicates injury to the collateral ligaments
Lateral Epicondylitis
The examiner stabilizes patient’s elbow and thumb rests on the lateral
epicondyle.
Patient makes a fist, pronates, radial deviates, and extends the wrist.
-- Positive, if sudden severe pain is found at the lateral epicondyle.
** Indicates tennis elbow
Lateral Epicondylitis - Method Two
Patient sitting, examiner palpates the lateral epicondyle while the
patient pronates the forearm (by the examiner), full wrist flexion, and
extends the elbow.
-- Positive, if pain over the lateral epicondyle.
** Indicates tennis elbow
Medial Epicondylitis - Golfers Elbow
Examiner palpates the patient's medial epicondyle. Patient's forearm
supinated with elbow and wrist extended.
-- Positive, if pain over medial epicondyle.
** Indicates injury to the medial epicondyle
Tinel's Sign - Ulnar nerve injury
Examiner stabilizes hand/wrist and elbow, while palpating ulnar nerve
at medial epicondyle and olecranon.
-- Positive, if tingling sensation in ulnar distribution occurs.
** Indicates injury to the ulnar nerve
*5*
* Hand Tests *
Finkelstein Test - Tenosynovitis of the abductor and extensor pollicis
longus and brevis

Patient makes a fist with the thumb inside the finger. Examiner
stabilizes the forearm and ulnar deviates.
-- Positive, if pain occurs.
** Used to determine DeQuervain's disease

Tinel Sign - Carpal Tunnel Syndrome

Examiner taps over the carpal tunnel at the anterior wrist (Over
retinaculum).
-- PosiFve, if paresthesia, Fngling into the thumb, 1st, 2nd, middle,
and lateral 1/2 of ring finger occurs.
** Indicates carpal tunnel syndrome

Phalens - Carpal Tunnel Syndrome (Median Nerve)

Patient places the posterior aspects of the hands together.


Examiner flexes the wrist maximally and holds 1 min.
-- Positive, if tingling occurs as above.
** Indicates carpal tunnel syndrome
*6*
* Hip Tests *
Trendelenburg Test
PaFent stands on 1 foot and tries to maintain his position.
-- The unsupported pelvis should rise for negative.
-- If pelvis stays level or descends for positive.
** Indicates gluteus medius weakness
Leg Length - True
Measure the distance from the ASIS to the medial malleolus of the
ankle. To determine if the tibia or the femur is longer, have the
patient lie supine and bend their knees to 90 degrees.
-- If knee appears higher - tibia is longer
-- If knee projects further anteriorly - femur is longer
*** Perform true first
Leg Length - Apparent
Patient lies supine and measure from umbilicus to the medial
malleolus of the ankle.
Ober - ITB contraction
Patient sidelying, lift their leg into abduction with the knee flexed
to 90 degrees.
-- Release the leg and if normal the leg will drop - negative
-- If leg stays abducted - positive
** Indicates ITB tightness
Thomas Test - Hip flexor tightness
Hip flexor I - Patient lies supine and pulls their knee to their chest.
-- Positive, if thigh flexes.
** Indicates tightness of the iliopsoas
Hip flexor II
- Patient lies supine with their hips at the edge of the table. Patient
pulls 1 knee to their shoulder.
-- Positive, if straight knee stays extended.
** Indicates tightness of the rectus femoris and iliopsoas
*7*
* Knee Tests *
Valgus Stress - Medial collateral ligament
Patient is supine with examiner with one hand about the ankle
and the other hand over the head of the fibula. A medial stress is
applied at the knee.
Test is performed with:
** Knee in full extension / O degrees
** Knee flexed to 15 degrees
-- Positive, if laxity or pain is noted along medial joint line and MCL

** Indicates medial collateral ligament sprain or tear.

Varus Stress - Lateral collateral ligament


Patient is supine with examiner having one hand at the ankle
(laterally) and the other hand at the medial joint line of the knee.
A lateral stress is applied at the knee.
** Knee at full extension / 0 degrees
** Knee flexed 30 degrees
-- Positive, if laxity or pain is noted along lateral joint line and LCL.

** Indicates lateral collateral sprain or tear.

Anterior Drawer - Anterior cruciate ligament


PaFent is supine with their knee flexed to 90 degrees and foot
flat in a neutral position. Examiner sits on the table, on their foot
to stabilize, with their hands grasped about the knee wit fingers
over the hamstrings and thumbs on the medial and lateral joint
line The examiner pulls the tibia forward.
-- If movement occurs the test is considered positive
-- If the anterior draw is positive, then test should be repeated
with the tibia:
Internal rotation - 20 degrees
External rotation - 15 degrees
** Sliding forward of the tibia with external rotation indicates that
the post/med joint capsule, ACL, and possible MCL are torn with
anterior medial rotatory instability
** Sliding forward of the tibia with internal rotation indicates that
the post/lat capsule and ACL may be torn with anterior lateral
rotatory instability.
** Indicates sprain or tear of the anterior cruciate ligament.
Posterior Draw - Posterior cruciate ligament
Same position as that of the anterior draw test except with a
posterior force on the tibia.
-- Positive, if tibia displaces posterior on the femur.
** Sliding of the tibia backward with internal rotation indicates
that the LCL, lateral capsule, arcuate, and PCL with posterior
lateral rotatory instability
** Sliding backward of the tibia with external rotation indicates
that the MCL, medial capsule, and PCL are torn with posterior
medial instability.

** Indicates sprain or tear of the posterior cruciate ligament.


Lachman Drawer - Anterior cruciate ligament
Preferred because it does not force the acute knee into 90
degrees of flexion. Knee is posiFoned at 15 degrees of knee
flexion with the leg externally rotated. One of the examiners
hands grabs the distal end of the patient's thigh, while the other
hand grabs the proximal tibia with an anterior force applied.
-- Positive, if laxity or instability is noted, usually accompanied by
pain.

** Indicates sprain or tear of the anterior cruciate ligament.


Degrees
1st - 1.3 cm (1/2”)
2nd - 1.3 - 1.9 cm ( 1/2 - 3/4)
3rd - 1.9 cm or more ( 3/4 or more)

Pivot Shift - Anterolateral rotatory instability


Patient is supine with one hand head of fibula and the other hand
at the patient's ankle. The lower leg is internally rotated and knee
extended. The thigh is then flexed 30 degrees at the hip and the
knee flexed with a valgus force applied to the knee.
-- If positive for ACL, a palpable shift is felt or a pop in the early
stages of knee flexion.

** Indicates sprain or tear of the anterior cruciate ligament.

McMurray - Meniscal tear


Patient is supine with the knee fully flexed, the examiner places
one hand on the foot and the other hand on top of the knee with
fingers touching the medial and the thumb lateral joint line. The
ankle hand scribes small circles with the leg pulled into extension.
-- If positive, the hand may feel a clicking response in the joint line
and/or accompanied by pain.
** External rotation - Medial meniscus
** Internal rotation - Lateral meniscus

** Indicates a tear of the medial or lateral meniscus

Apley's Compression - Meniscal tear


PaFent lies prone with their knee flexed to 90 degrees. While
stabilizing the thigh, a hard downward pressure is applied to the
leg. The leg is rotated in and out at the tibia.
-- If pain results, positive for meniscal tear
** External rotation - medial meniscus
** Internal rotation - lateral meniscus

** Indicates a tear of the medial or lateral meniscus.

Apley's Distraction - Collateral ligament tear


Patient lies prone with the knee flexed to 90 degrees. The examiner
places their knee across the patient's thigh to stabilize and applies a
traction force, while moving it in and out.
-- Positive, if pain occurs for capsular and ligamentous pathology.
-- If meniscus is torn, no pain will occur.
** Indicates sprain or tear of the collateral ligaments.

Godfrey - Posterior cruciate ligament


PaFent is supine with hips and knees flexed to 90 degrees. The foot
is supported at the lower calf or ankle.
-- If positive, a posterior sag of the tibia occurs and there is
posterior instability. If pressure is applied, displacement may
increase.

** Indicates a sprain or tear of the posterior cruciate ligament.


Bounce Home - Meniscal tear
Patient lies supine with the heel of the patient's foot cupped in the
examiners hand. Patient's knee is completely flexed and then
passively allowed to extend.
-- If extension is incomplete or with a rubbery end-feel, then
something is blocking full extension.

** Indicates a tear of the medial or lateral meniscus.


Clarke's Sign - Patellar chondromalacia
Patient is supine with the knee extended and the examiner has his
hand pressing down on the upward proximal pole or the base of the
patella. Patient is asked to do a quad set and hold.

-- If there is pain, and cannot hold a contraction then the test is


positive.
** Indicates chondromalacia or patellofemoral syndrome.

Patellar compression - Chondromalacia of the patella


Patient is supine with their knee slightly flexed. The patella is
pressed in the femoral groove and moved forward/backward.
-- If positive, pain or crepitus will be noted.

** Indicates patellar chondromalacia.

Apprehension - Patellar dislocation


Patient is supine with the knee/patella relaxed. The examiner
pushes the patella laterally.
-- Positive if, apprehension is noted at the point of dislocation.

** Indicates patellar dislocation.


*8*
* Ankle Tests *
Fractures - Tibial or fibular
Patient is positioned supine or prone with the heel struck with a
reflex hammer.

-- Positive, if pain is felt or radiates through the leg

Anterior Drawer - Anterior talofibular ligament


Patient lies supine with their foot relaxed and the examiner
stabilizes the Fbia/fibula with the foot in 20 degrees of
plantarflexion. The examiner grabs the ankle and draws the talus
forward in the mortise.
Inversion gives an anterolateral stress with increased stress on the
ATF.
Straight anterior translation will indicate both medial and lateral
deficits.
-- If positive, laxity and pain will be noted along the ATF.

** Indicates sprain or tear of the ATF.

Posterior Drawer & Posterior talofibular ligament


Patient lies supine with their foot relaxed and the examiner
stabilizes the Fbia/fibula with the foot in 20 degrees of
plantarflexion. The examiner grabs the ankle and draws the talus
backwards in the mortise.
-- Positive if, laxity and/or along the PTF
** Indicates sprain or tear of the PTF.

Talar Tilt - Calcaneofibular ligament


Patient is supine or sidelying with the foot relaxed and knee flexed
to 90o to relaxed the gastrocnemius. Examiner applies adducFon
stress to the talus.

-- Positive, if lateral gapping is present.

** Indicates sprain or tear of the CFL.


Thompson Test - Achilles tendon
Patient lies prone with foot over the edge of table. Examiner
squeezes the calf muscle.
-- Positive, if plantarflexion is absent.

** Indicates strain, tear, or rupture of the Achilles tendon.


Myotomes
C4 Shoulder shrug
C5 Shoulder abduction, external rotation
C6 Elbow flexion, wrist extension
C7 Elbow extension, wrist flexion
C8 Ulnar deviation, thumb abduction, finger
abduction
Tl Finger adduction
L2 Hip flexion
L3 Hip flexion, knee extension
L4 Knee extension, ankle dorsiflexion
L5 Great toe dorsiflexion (extension) eversion
S1/S2 Ankle plantarflexion, knee flexion
S3/S4 Rectal sphincter

dermatomes:
• groin - L1
• anterior aspect thigh - L2
• lower third anterior aspect thigh and knee - L3
• medial aspect leg to the big toe - L4
• lateral aspect leg to the middle three toes - L5
• little toe, lateral border foot, lateral side posterior
aspect whole leg - S1
• heel, medial side posterior aspect whole leg - S2

The Oxford classification


0 = no contraction at all
1 = flicker of contraction only, movement
of the joint does not occur
2 = movement is possible only with gravity counterbalanced
3 = movement against gravity is possible
4 = movement against resistance is possible
5 = normal functional movement is possible.

Flags
'Yellow flags' are psychosocial factors
including a previous history of anxiety
and depression, impending compensation,
absence from work, sickness benefit, invalidity
benefit, passivity and high levels of
dependency and poor coping skills. 'Red flags'
are clinical features that should alert the therapist
to the possibility of severe pathology. They
include bladder and bowel malfunction, saddle
anaesthesia, bilateral paraesthesia, neurological
signs, unexplained weight loss, a past history of
carcino ma, general debility and fever.

painful arc of movement is observed


in patients suffering from impingement
syndromes, whereby the superior
aspects of the rotator cuff, biceps tendon and
bursae are impinged by repetitive overarm
activities. Pain is experienced between 90 and
130 degrees of abduction.

Match these five scenarios to the likely pathology:


1. reduced range of movement particularly on active and
passive rotations and abduction
2. painful arc of movement between 90 and 120 degrees
3. inability to actively abduct the arm away from the body and
maintain the position when the arm is placed there passively
4. pain and weakness on resisted elbow and shoulder flexion
5. excessive movement on passive anterior, posterior and
sulcus draw tests of the shoulder.
Answers
(1) Frozen shoulder (adhesive capsulitis).
(2)Impingement of supraspinatus under the acromion.
(3) Rupture of the rotator cuff musculature.
(4)Ruptured biceps brachii muscle.
(5) Global instability.

Key point
From the movement of supine to sitting,
one leg may appear to be longer in
supine and shorter in sitting. This is
caused by anterior rotation of the innominate bone
on the affected side and is an SI joint dysfunction.

Test Yourself
Match these five scenarios to the likely pathology:
1. local tenderness at the ischial tuberosity and pain on resisted
knee flexion
2. pain in the groin on coughing, resisted adduction sit-ups and
weightbearing
3. a 3-year history of pain and stiffness particularly on medial
rotation
4. local tenderness and heat palpated in the area of the greater
trochanter having an insidious onset
5. increased or exaggerated lumbar lordosis and a positive
Thomas test.
Answers
(1) Hamstring strain.
(2) Sportsman's hernia.
(3)Osteoarthritic (OA) hip.
(4) Trochanteric bursitis.
(5)Tightness in the hip flexors (iliopsoas.)

The toes
Look for:
• clawing (hyperextension of the metatarsophalangeal
joints and flexion of the other phalanges)
• mallet toe (flexion of the distal interphalangeal
joints)
• hammer toe (hyperextension of the metatarsophalangeal
and flexion of the proximal interphalangeal
joints)
• hallux valgus (lateral deviation of the first interphalangeal
joint)
• hallux rigidus (stiffness of the first interphalangeal
joint).\

Test Yourself
Match these five scenarios to the likely pathology:
1. One ankle keeps giving way and feels unstable. There is poor
proprioception on one leg.
2. There is pain in the plantar aspect of the heel on weight-
bearing, or toe extension.
3. There is pain under the medial malleolus, increasing on
resisted inversion.
4. There is longstanding, insidious pain and stiffness in the
ankle, increasing on weight-bearing
5. The patient has a history of an inversion strain combined with
swelling and bruising under the lateral malleolus.
Answers
(1) Lengthened lateral ligaments
. (2) Plantar fasciitis.
(3) Tendonitis of tibialis posterior.
(4) Osteoarthritis of the ankle.
(5) Lateral ligament sprain.

Sudeck's atrophy/reflex sympathetic


dystrophy (RSD)/algodystrophy/causalgia
The term complex, regional pain syndrome is now being
used to describe these pathological states. This is a
complication where the patient complains of severe
pain on movement, or at rest, out of proportion to the
initial injury. The limb is swollen. The skin appears
shiny and discoloured and feels cold; in extreme cases
this may lead to the limb becoming exquisitely tender
and discoloured. Osteoporosis and permanent contractures
may follow.

Contract-Relax (CR): A relaxation technique usually performed


at a point of limited ROM in the agonist pattern.
Strong, small range isotonic contraction of the restricting
muscles (antagonists) with emphasis on the rotators
is followed by an isometric hold.
Hold-Relax (HR): A relaxation technique usually performed
in a position of comfort and below a level that
causes pain. Strong isometric contraction of the
restricting muscles (antagonists) is resisted, followed
by voluntary relaxation, and passive movement into
the newly gained range of the agonist pattern.
Rhythmic Initiation (RI): Voluntary relaxation followed
by passive movements progressing to active-assisted
and active-resisted movements to finally active
movements.
Rhythmic Stabilization (RS): Utilizes alternating
isometric contractions of first agonists, then antagonists
against resistance; no motion is allowed.
Stabilizing Reversals: Utilizes alternating isotonic
contractions of first agonists, then antagonists against
resistance, allowing only very limited ROM.
Tenderness at the knee (tibiofemoral
joint)

Identify the joint line clearly by flexing the knee and observing for
hollows at the sides of the patella ligament - these lie over the joint line.

1. Tenderness at the joint line is common in meniscal and fat pad injuries.
2. Tenderness along the line of the collateral ligaments of the knee joint is
common at the site of a lesion following a tear, particularly at the upper
and lower attachments and at the ligament's midpoint. Associated
bruising and oedema may also be a feature of acute injuries.

3. Tenderness at the tibial tubercle - in children and adolescents,


tenderness and hypertrophy of the tibial tubercle prominence - is
associated with Osgood Schlatter's disease. Tenderness is also found
following acute avulsion injuries of the patella ligament
and its tibial attachment.

4. Tenderness and swelling in the popliteal fossa may indicate the


presence of a Baker's cyst. This conditionis associated with degenerative
changes or rheumatoid arthritis involving the knee joint.

5. Tenderness at the adductor tubercle may indicate strain in the adductor


magnus muscle.

6. Femoral condyle tenderness may indicate the presence of


osteochondritis dessicans.

Patellofemoral joint assessment

A knee assessment should include assessment of both the tibiofemoral


and patellofemoral joint. Observe the position of the patella and compare
both sides.

• Determination of a high or small patella (patella alta) is made by


calculating the ratio of the length of the patellar tendon to the longest
diagonal length of the patella. The normal value for this ratio is 1.02 plus
or minus 20% (Simmons and Cameron 1992). Patella alta is a
predisposing factor in anterior knee pain and recurrent dislocation of the
patella.

• Observe any tilting, lateral glide and rotation of the patella during a
quadriceps contraction. Compare this with the other side.

• McConnell (1996) described a 'critical test' for the patellofemoral joint.


Resisted inner-range quadriceps contraction is performed with the patient
sitting at various degrees of knee flexion to determine whether this
reproduces the patient's symptoms. Compare both sides (Figure 2.51).
• The McConnell critical test may be repeated with the patella
taped in the corrected position. This will determine whether the
taping is effective and should be incorporated into the treatment
programme. Taping is believed to enhance activation and earlier
timing of vastus medialis in quadriceps contractions and thus
restore patellar tracking to normal.

• Observe any excessive pronation of the feet which may


increase the Q angle (Figure 2.49).

• Test for tightness in the following structures: lateral


retinaculum, iliotibial band, hamstrings and calves. Tightness of
the above structures will increase dorsiflexion, and therefore
pronation of the foot and ankle during the gait cycle. All of this
will increase the Q angle (Olerud and Berg 1984).

• Perform passive accessory movements to test the mobility and


pain response of the patella in all directions. Observe pain,
laxity, or muscle spasm.

• Perform Clarke's test. The patient is asked to contract the


quadriceps whilst the patella is pressed firmly down against the
femur. Pain is produced in conditions such as chondromalacia or
osteoarthritis affecting the patellofemoral joint.

Test Yourself
Match these five scenarios to the likely pathology:

1. The knee is stiff and painful for about half an hour in the
morning, aches at the end of the day, and has been like that for a
long time.

2. The knee locks and has to be jiggled around to unlock it.

3. Since a tackle last week the knee keeps giving way and
becomes very swollen.
4. The knee is very red and swollen. The person also feels
feverish and generally unwell.

5. When the person walks downstairs he feels pain behind his


kneecap.

6. There is pain on the inside of the knee and it hurts doing


sideways movements.

Answers
(1) Osteoarthritis. (2) Torn meniscus. (3) Ruptured
anterior cruciate. (4) Infective arthritis. (5)
Chondromalacia patella. (6) Torn medial collateral ligament.
Tenderness

• Tenderness localised over and just proximal to the malleoli often occurs
following a fracture.

• Tenderness and pain in the area distal and inferior to the lateral
ligaments is common following inversion sprains. The anterior talofibular
ligament (ATF) is the most commonly injured since the ligament is most
often torn in the combined position of inversion and plantarflexion. This
is the loose packed position and one in which the anterior band of the
lateral ligament is particularly placed on stretch.

• Tenderness along the line of the long flexor tendons and/or the peroneal
tendons may indicate the presence of tenosynovitis. This maybe
accompanied by local thickening.

• Tenderness at the articular surface of the talus is common in


osteoarthritic conditions.

• Tenderness at the heel is found in conditions such as calcaneal exostosis


(bony spurs), tendocalcaneal bursitis and plantar fasciitis.

• Diffuse tenderness under the metatarsal heads may be a sign of Morton's


neuroma. This is a condition characterised by inflammation and pain
around 3rd and 4th digital nerves. Pain is reproduced on squeezing the
medial and lateral sides of the forefoot together.
• Diffuse tenderness and swelling on both the plantar and dorsal surfaces
of the forefoot is a common finding in rheumatoid arthritis.

• Tenderness on the mid posterior aspect of the calcaneus may


be a sign of a calcaneus bursitis.

• Tenderness along the Achilles tendon may be a sign of a sprain


or tendonitis in the Achilles tendon.

Assessment of movement
Active movements
These are movements performed by the patient's voluntary
muscular effort.

Passive movements
These are movements performed by an external source, such as
the physiotherapist or a pulley system. There are two types of
passive movements.

• Physiological passive movements are movements that can be


performed actively by the patient (e.g. flexion or abduction of
the shoulder joint).

• Accessory movements cannot be performed actively by the


patient (e.g. they incorporate glide, roll or spin movements that
occur in combination as part of normal physiological
movements). An example of an accessory movement is an
anterior-posterior glide at the knee joint.

Resisted movements
These are performed against the resistance of the physiotherapist
or weights by the patient's own effort.

Differentiation tests

If a lesion is situated within a non-contractile structure such as


ligament, then both the active and passive movements will be
painful and/or restricted in the same direction. For example both
the active and passive movement of inversion will produce pain
in the case of a sprained lateral ankle ligament. However, if a
lesion is within a contractile tissue such as a muscle, then the
active and passive movements will be painful and/or restricted
in opposite directions (Cyriax 1982). For example a ruptured
quadriceps muscle will be painful on passive knee flexion
(stretch) and resisted knee extension (contraction).

End-feel
During passive movements, the end-feel is noted. Different
joints and different pathologies have different end-feels. The
quality of the resistance felt at the end of range has been
categorised by Cyriax (1982). For example:
• Bony block to movement or a hard feel is characteristic of
arthritic joints.
• An empty feel, or no resistance offered at the end of range,
may be due to severe pain associated with infection, active
inflammation or a tumour.
• A springy block is characterised by a rebound feel at the end of
range and is associated with a torn meniscus blocking knee
extension.
• Spasm is experienced as a sudden, relatively hard feel
associated with muscle guarding.
• A capsular feel shows a hardish arrest of movement.
The Oxford classification
0 = no contraction at all
1 = flicker of contraction only, movement
of the joint does not occur
2 = movement is possible only with gravity
counterbalanced
3 = movement against gravity is possible
4 = movement against resistance is possible
5 = normal functional movement is possible.
Differentiation tests of muscles and tendons
These are contractile structures and are therefore tested by
performing a contraction against resistance. A pain response
and/or apparent weakness may indicate a strain of the muscle at
any particular point of the range of movement. Full range should
be checked since the muscle may be weak only at a particular
point in the range. Muscle length may also be tested, particularly
those muscles that are prone to become tight and then lose their
extensibility. Muscles that pass over two joints and have
mobiliser characteristics are particularly prone to tightness.
Examples of these are the hamstrings, rectus femoris,
gastrocnemius and psoas major. The length of the muscle is
tested by passively moving the appropriate joints. The stretch is
compared to the other side to determine reproduction of pain
and/or restriction of movement.

Passive insufficiency of muscles


This occurs with muscles that act over two joints (Figure 2.4a).
The muscle cannot stretch maximally across both joints at the
same time. For example, the hamstrings, may limit the flexion
of the hip when the knee joint is in extension since they are
maximally stretched in this position. However, if the knee is
flexed passively, then the hip will be able to flex further since
the stretch on the hamstrings has been reduced.

Active insufficiency of muscles


This, too, occurs with muscles that act over two joints (Figure
2.4b). The muscle cannot contract maximally across both joints
at the same time. An example is the finger flexors. If you are to
make a strong fist, you may notice that the wrist is in a neutral
or an extended position when you do this action. Now, if you
attempt to actively flex your wrist joint whilst keeping your
fingers flexed, you will find that the strength of the grip is
greatly diminished. This is because the wrist and finger flexors
are unable to shorten any further and so the fingers begin to
extend or lose grip strength.
Differentiation tests of ligaments
Ligaments are non-contractile structures and are tested by
putting the structure on stretch. Examples are a valgus strain of
the knee, to stretch the medial collateral ligament of the knee; or
passive inversion of the subtalar joint, to stretch the lateral
ligament of the ankle. A positive test would be a pain response
or observation or feel of any excessive movement of the joint
whencompared to the other side.
Characteristics of degenerative joint disease
Signs and symptoms may include:

• pain that increases on weight-bearing activities (standing and


walking, walking downstairs particularly)

• insidious onset of symptoms followed by progressive periods


of relapses and remissions

• pain and stiffness in the morning

• stiffness following periods of inactivity

• pain and stiffness that arise after unaccustomed periods of


activity
• bony deformity (e.g. characteristic varus deformity may follow
from collapse of the medial compartmental joint space)

• reduction of the joint space observed on X-ray, with bony


outgrowths or osteophytes.
SHOULDER JOINT

Active movements

Full active movements of the shoulder girdle and joint are


performed, noting any restriction, asymmetry and pain response.
Note the capsular pattern for the glenohumeral joint is limitation
of lateral rotation, abduction and medial rotation (Cyriax 1982).

Key point

A painful arc of movement is observed in patients


suffering from impingement syndromes, whereby the
superior aspects of the rotator cuff, biceps tendon and
bursae are impinged by repetitive overarm activities. Pain
is experienced between 90 and 130 degrees of abduction.

Impingement may be caused by loss of scapular stability. Faulty


patterns of scapula motion include early rotation and elevation
of the scapula (reversed scapulohumeral rhythm). This may
implicate weakness in the stabilisers (e.g. lower fibres of
trapezius, rhomboids and serratus anterior), or shortness and
overactivity in the upper trapezius and levator scapulae.

Impingement may also be caused by weakness or inhibition of


the rotator cuff muscles that produces a superior translation of
the humeral head (i.e. subscapular is, teres minor and lower
infraspinatus). There may also be late timing of lateral rotation
during abduction which may cause impingement.

Tightness of pectoralis minor can cause increased protraction of


the scapula which decreases the subacromial space. Repetition
of the movement may induce an element of fatigue, and
abnormal movements may derive from that. A juddering
movement of the scapula on return from elevation implicates
poor eccentric control.
Differentiation. If abduction reproduces the person's pain, then
differentiation between the glenohumeral and subacromial
structures may be required.

1. If the movement is repeated and compression applied to the


glenohumeral joint causes an increase in symptoms, then the
glenohumeral joint is implicated.

2. If the movement is repeated and a longitudinal force in a


cephalad direction is applied (increasing compression on the
subacromial structures), with an increase in pain, the
subacromial structures are implicated.

Failure to initiate or maintain abduction when placed


passively into abduction is a sign of rotator cuff rupture, and
the patient should be referred to a consultant for a
repair/further investigations.
THE HIP JOINT

. increased lumbar lordosis: this may suggest a fixed flexion


deformity of the hip(s).

. bruising in the abdominal or groin area: suggestive of a


sportsman's hernia.

Leg length discrepancy

Apparent leg length discrepancy is measured from the xiphoid


of the sternum to the tip of the medial malleolus using a tape
measure (compare with the other leg).

True leg length discrepancy is measured, using a tape measure,


from the ASIS to the tip of the medial malleolus. A difference in
leg length of up to 1-2 cm is considered normal by some
clinicians. If there is a leg length difference, measure the length
of the individual bones - i.e. thigh and leg

Key point

From the movement of supine to sitting, one leg may


appear to be longer in supine and shorter in sitting. This is
caused by anterior rotation of the innominate bone on the
affected side and is an SI joint dysfunction.

Muscle length assessments


The Thomas test

This test determines the presence of a fixed flexion deformity at


the hip. With the patient supine, the hip is fully passively flexed,
and the lumbar lordosis is obliterated. If the contralateral
(opposite) hip rises off the bed, this indicates a fixed flexion
deformity of that hip. This may be due to tightness or restriction
in the capsule, iliopsoas or rectus femoris.
To differentiate between the iliopsoas and rectus femoris as the
source of restriction, the patient's knee is passively extended
(Figure 2.8). If this results in the patient's hip dropping down
into less flexion, then the restriction is in the rectus femoris
muscle because by extending the knee an element of stretch has
been removed. If the hip is unaffected and remains, in the same
degree of flexion, independently of the knee
COMPLICATIONS OF FRACTURES

Critical blood disorders


Pulmonary embolism and deep vein thrombosis are two possible
complications of a fracture. Shock may be caused by
hypovolaemia or loss of blood. Femoral shaft fractures may
bleed as much as 3 pints (1.7 L) and pelvic fractures may lose 6
pints (3.4 L). Clinical signs of this are tachycardia (rapid heart
rate), pallor from reduced peripheral perfusion, hypoxia
(decreased oxygen saturation), confusion, and a state of
semiconsciousness.

Infection and tetanus are threats, especially following open or


compound fractures. Most people are now immunised against
tetanus or given booster tetanus injections if they have a large
open wound. Osteomyelitis (bone infection) can be stubborn to
respond to treatment.

Fat embolism (ARDS - acute


respiratory distress syndrome)

If a person sustains multiple fractures of large bones, or


crushing injuries, or if large amounts of marrow become
exposed, there may be leakage of microscopic fat globules into
the circulatory system. These may become trapped in the lungs.
Symptoms include respiratory distress, shortness of breath,
drowsiness, decrease in saturation of oxygen levels, and
petechiae (tiny haemorrhages which appear on the chest). ARDS
is potentially fatal.

Compartment syndrome
If muscles become damaged or inflamed at the time of injury,
and intramuscular pressure builds up with no means of release,
death (necrosis) of the tissues from ischaemia (lack of blood
supply) may result. It is defined as the condition in which high
pressure within a closed fascial sheath reduces capillary blood
perfusion below the level necessary for tissue viability.
Compartment syndrome is seen most commonly in the anterior
tibial muscles or forearm muscles. Clinical signs of a limb with
compartment syndrome are the five Ps:
• Pale
• Painful
• Pulseless
• Paraesthesiae
• Paralysed.
Treatment revolves primarily around accurate diagnosis. Check
colour, sensation and movement after any injury or surgery,
elevate and cool the limb. Surgical decompression (fasciotomy)
may be necessary as an emergency procedure.
Sudeck's atrophy/reflex sympathetic
dystrophy (RSD)/algodystrophy/causalgia

The term complex, regional pain syndrome is now being used to


describe these pathological states. This is a complication where
the patient complains of severe pain on movement, or at rest, out
of proportion to the initial injury. The limb is swollen. The skin
appears shiny and discoloured and feels cold; in extreme cases
this may lead to the limb becoming exquisitely tender and
discoloured. Osteoporosis and permanent contractures may
follow. Management is difficult. Sympathetic nerve blocks and
active physiotherapy management programmes are often
employed with varying degrees of success (Viel et al. 1999).
Vasodilator drugs such as guanethedine are occasionally
successful. It may also respond to nerve blocks , local analgesia,
TENS, and other local therapies but recovery is slow and may
take several months. Fortunately this complication is
comparatively rare.
Burn
Aetiology of Burns

The most common causes of injury are by fire, by chemicals, by


scalding, by electricity and by inhalation.

• Fire burns. These occur when the patient is caught by fire.


Because the clothes ignite, the burns are often partial- or full-
thickness. Flash flames tend to cause partial-thickness burns.

• Chemical burns. Caustic substances (e.g. cement) can cause


deep burns. The depth of tissue involved can be limited by
prompt action.

• Scalding. Hot water is the most common cause of scalds. It


may be as a hot drink, or by boiling fluid from a pan or kettle.
Scalds are common in the elderly, often caused when climbing
into a hot bath and being unable to climb out again. These burns
will vary in depth relative to the exposure time.

• Electrical burns. Burns will appear on the skin where there has
been contact with a live wire. There will be a burn at the entry
and exit site of the electric current. Neither the depth nor the
size of the burn is predictable. There can be extensive damage to
deep structures with little external evidence. Often this type of
injury is complicated by cardiac and respiratory arrest.

• Inhalation burns. Direct thermal injury can be sustained by


inhalation of flames, hot gases or steam. This places a major
threat to the upper airway, causing oedema of the larynx,
pharynx and trachea. Early diagnosis of an inhalation injury is
essential. The patient must be intubated before the oedema
becomes extensive, as this will prevent the passing of the
endotrachael tube.
Total burn surface area (TBSA)
The greater the total burn surface area, the poorer the prognosis.
A formula for gauging outcome is: percentage chance of
survival = [100 - (age in years + percentage TBSA)]. For
example, a 60-year-old with 30%TBSA has a 10% chance of
survival [100 - (60 + 30)].

A 20-year-old with the same TBSA has a chance of survival of


[100 - (20 + 30)], or 50%. It is interesting to note that if both
legs are burnt, 36% TBSA is affected. Therefore, a 50-year-old
with such extensive burns has [100 - (50 + 36)] or 14% chance
of recovery. A method for gauging the total body surface area is
'the rule of nines'. This rule divides the body surface into 11
areas, each constituting 9% of the total (Figure 5.3). The
perineum is counted as 1%.
Superficial Partial-Thickness Burn
With a superficial partial-thickness burn (Fig. 27.3)
damage occur through the epidermis and into the
papillary layer of the dermis. The epidermal layer is
destroyed completely , but the dermal layer sustain only
mild to moderate damage. The most common sign of a
superficial partial-thickness burn is the presence of Skin
Involvement and Scar Formation in the intact blisters
over the area that has been injured.
Deep Partial-Thickness Burn
deep parltial- thickness burn involves destruction of the
epidermis with damage of the dermis down to the reticular
layer. Appear mixed red or waxy white colour .

full thickness Burn

full-thickness burn all epidermis and dermal layer are


destroyed completely and In addition subcutaneous fat layer
may be damaged to some extent .

Subdermal BUrn
An additional category of burn, involve complete
destruction of all tissue from the epidermis down to and
through subcutaneous tissue Skin Involvement
extending into Fascia, muscle, or Bone and Scar
Formation.
Pediatric
Risk factor for osteoporosis
Orthosis
Prothesis
Amputation
Common gait abnormalities

The following lists are extracted from Engstrom and

Van deVen (1999).

Transtibial

• Excessive knee flexion.

• Insufficient knee flexion.

• Delayed knee flexion during the swing phase.

• Early knee flexion ('drop off).

• Lateral shift of the trunk.

• Lateral shift of the prosthesis.

• Rotation of the foot.

Transfemoral

• An abducted pattern.

• Rotation of the foot.

• Circumduction.

• Uneven step length.

• Uneven timing.

• 'Drop off.

• 'Foot slap'.

• Uneven heel rise.

• Rising up on the toes of the opposite limb ('vaulting').


• Medial or lateral heel travel in swing phase ('medial

whip'/'lateral whip').

• Terminal swing impact.

• Uneven arm swing.

• Lateral side bend of the trunk.

• Forward trunk flexion.

• Lumbar lordosis.

Residual limb pain

Residual limb pain can be attributed to a variety of causes,


including the formation of a neuroma - a nodule formed at the
end of a cut peripheral nerve, which folds back on itself and
creates an enlargement. Pain caused by an ill-fitting prosthesis is
referred to as 'prosthetic pain' and has a number of causes. In
addition, postoperative pain is likely.

Phantom limb pain

Phantom limb pain can be described as distressing pain


sensation felt by patients in the limb that is no longer there. It is
well documented and is a feature that can impact significantly
on the life of a patient (Weiss and Lindell 1996; Williams and
Deaton 1997; Hill et al. 1995). It is a pain that seems to be an
increasing factor with increasing age (Houghton et al. 1994).
The psychological status of the patient also has an impact on
phantom limb pain. Phantom limb pain is described variously as
cramping, squeezing, burning, sharp and shooting. Table 22.4
lists typical descriptions patients use to describe it.
Deformity
Gait
Joint motion patterns commonly reported include:
ankle plantar-dorsiflexion, foot rotation, knee flexion/ extension,
knee valgus/varus, knee rotation, hip flexion/extension, hip
abduction/adduction, hip rotation, pelvic tilt, pelvic obliquity,
and pelvic rotation. Although all these movement patterns are of
interest, only major movement patterns of the lower limb are
covered here. These include:
• plantar and dorsiflexion of the ankle joint
• knee flexion/extension
• hipflexion/extension
• motion of the pelvis in the coronal plane
• motion of the pelvis in the transverse plane.

Motion of the ankle joint

The movement about the ankle joint is of great importance as it


allows shock absorption at heel strike, progression of the body
forwards during the stance phase, as well as being vital in the
'push off' or propulsive stage immediately before the toe leaves
the ground. During the swing phase the motion of the ankle joint
allows foot clearance, which can be lacking in some
pathological gait patterns and is generally known as 'drop foot'.
The range of motion, which occurs in walking, varies between
20 degrees and 40 degrees, with an aver-age value of 30
degrees. However, this does not tell us how the motion of the
ankle varies throughout gait. During gait the ankle has four
phases of motion

Phase 1
At initial contact, or heel strike, the ankle joint is in a neutral
position; it then plantarflexes to between 3 and 5 degrees until
foot flat has been achieved (Figure 6.6). This is sometimes
referred to as 'first rocker' or 'first segment', which refers to the
foot pivoting about the heel or calcaneus. During this period the
dorsiflexor muscles in the anterior compartment of the foot and
ankle are acting eccentrically, controlling the plantarflexion of
the foot. This gives the effect of a shock absorber and aids
smooth weight acceptance to the lower limb.

Phase 2
At the position of foot flat the ankle then begins to dorsiflex.
The foot becomes stationary and the tibia becomes the moving
segment, with dorsiflexion reaching a maximum of 10 degrees
as the tibia moves over the ankle joint (Figure 6.7). The time
from foot flat to heel lift is referred to as 'second rocker' or
'second segment', which refers to the pivot of the motion now
being at the ankle joint with the foot firmly planted on the
ground. During this time the plantarflexor muscles are acting
eccentrically to control the movement of the tibia forwards.

Phase 3
The heel then begins to lift at the beginning of double support,
causing a rapid ankle plantarflexion reaching an average value
of 20 degrees at the end of the stance phase at toe off (Figure
6.8). The ankle plantarflexes at a rate of 250 degrees/s. This
rapid movement is associated with power production. During
this propulsive phase of the gait cycle the plantarflexor muscles
in the posterior compartment of the foot and ankle
concentrically contract, pushing the foot into plantarflexion and
propelling the body forwards. This is referred to as 'third rocker'
or 'third segment' as the pivot point is now under the metatarsal
heads.
Chest
Cardiology
NEUROLOGY
Effect of ex on insulin?

EXERCISES : 1- increase glucose uptake by muscles 2- increase body response


to insulin not increase production of insulin

1- Coll`s fracture may possibly cause injury to which of the following?

a- flexor pollicis

b- abductor pollicis

c- adduuctor pollicis

d- extensor pollicis longus √

2- Maximum stability of hip joint depends on;

a- bone, ligament, muscles √

b- muscles, ligament, bone

c- bone only

3- Which is not correct about oxford scale?

a- 0 = no contraction

b-1 = flicker contraction

c- 4 = can make full ROM against gravity √

4- the name of muscle cell is

a- sarcomere √

b- actin

c- myosin

d- no name

5- Which don't present in dermis ?

a- tactile

b- blood vessels

c-osteoblast √
6- when you walk in reception the young boy´s color change and become
cyanotic and can not

breath what you do

a- abdominal thrust

b- clow on back

c- put in supine position

d- a + b √

7- Muscle strength depend on all except:

a- size of muscle

b- lever arm

c- number of muscle spindle

d- type of ex √

8- Poliovirus transfer through :

a- Respiratory tract

b- Skin

c- Hepatic faluier

d- GIT √

9- In thoracic outlet syndrome, affection is:

a- superior to operative thoracic √

b- posterior to operative thoracic

c- inferior to operative thoracic

d- anterior to operative thoracic

10-The spinal cord...

a- Occupies the lumbar cistern

b- Has twelve (12) cervical segments


c- Contains the cell bodies of postganglionic sympathetic efferent neurons

d- Ends at the conus medullaris √

11- The thoracic outlet syndrome, is due to :

a- Upper opening chest √

b- Lower opening chest

c- Lateral opening chest

d- Anterior opening chest

12- When o2 demand in heart decrease what happened ?

a- decrease HR multiplied by systolic pressure

b- decrease diastolic pressure √

c- increase coronary arteries

d- increase HR multiplied by systolic pressure

13- Above knee prothesis long cause limb to do during gait :

a- vaulting √

b- lat bending

14- Heat produced by electrical stimulation not depends on :

a- Size of ttt area

b- Method of application

c- Shape of wave √

15- when patient coming for you has limited ROM due to pain to decrease
edema in addition to leg elevation you can use which type of electrical
stimulation:

a- faradic √

b- tens

c- galvanic
d- dyadinamic

16- Use of electro and elevation to decrease edema

17- decrease edema in ankle joint by:

a- hvgs ( high voltage galvanic stim ) √

b- interfrencial

c- mono phasic rectangiular

d- russian current

17- in electrical stimulation intensity of muscle contraction(shortening) depend


on :

a- shape of current and electrical device used

b- duration of ttt

c- size of electrodes

d- intensity and frequency of current √

18- all of the following are physiological effect of electrical stimulation except

a- Vasoconstriction of blood vessels √

b- Increase blood flow

c- Increase metabolic rate

19- Electric stimulation which is not true

a- Vasodilatation

b- Decrease metabolic rate √

c- decrease ms spasm

d- Vasoconstriction

20- applied electrical stimulation to area for long lasting period cause?

a- skin irritation √

b- weakness of ms
c- edema occure

21- In electrical stimulation intensity of muscle contraction(shortening) depend


on :

a- shape of current and electrical device used

b- duration of ttt

c- size of electrodes

d- intensity and frequency of current √

22- One of these electrical stimulation methods does not stimulate denervated
muscle

a- high voltage interrupted long duration

b- TENS √

c- galvanic

d- faradic

23- For denervated ms we use at least ‫ ر‬# $‫آ‬

a- 10 m.sec

b- 50 m.sec

c- 100 m.sec √

d- 300 m.sec

24- Which current used for stimulated denervated muscle

a- TENS

b- faradic

c- diadynamic

d- direct galvanic √

25- Frequency that can make titanic muscle contraction: ‫ ر‬# $‫آ‬

a- 10
b- 30

c- 50 √

26- Electrical Nerve Stim frequency is

a- 23-120

b- 50-150 √

c- 10-70

27- patient with burn to the dorsum of the hand with chronaxie 227 msec so
you use

a- faradic current

b- continous direct current

c- interrupted direct current √

d- TENS

28- For strengthing muscle we do electrical stimilation for

a- 0 pulse

b- 16 pulse √

c- 5 pulse

d- 10 pulse

29- pigmentation with U.V is due to:

a- melanin √

b- histamine

30- Erythema with U.V is due to:

a- melanin

b- histamine √

31- baby flacid klumps palsy need stimilation by

a- galvanic √
b- didynamic

c- reciprocal

32- Acceleration injury (whiplish injury) may cause

a- pain in posterior aspect of neck √

b- pain at post aspect of back

c- dislocation of vertebral colum

d- tearing of post para spinal ligament

33- Not in rotator cuff ms :

a- supra spinatus

b- infra spinatus

c- teres major √

d- sub scapularis

34- March Fracture occurs as a result of:

a- Direct trauma

b- Bone pathology

c- Recurrent stress √

35- FRACTURE GALLIAZI which is not correct

a- fracture of raduis and sublaxtion of ulna

b- fracture of ulna and sublaxtion of radius √

c- its occur by falling

36- which of the following is a possible cause for anterior pelvic tilting

a- weak lateral trunk

b- tightness tensor fascia lata

c- tight hamstring
d-sever weakness of abdominal muscles √

37- abnormal pattern of gait that not occur in Parkinson is

a- festinating gait

b- Propulsive gait

c- retropulsive gait

d- high steppage gait √

38- the most common site of fracture in old patient due to failing is ;

a- head of femur

b- trochanteric

c- shift of femur

d- neck of femur √

39- All of the following is clinical features in neck of the femur except

a- pain in the hip region

b- abduction of leg √

c- shortness of the limb

d- lateral rot. of the leg

40- which statement is incorrect about golgi tendon organ:

a- it monitor muscle length √

b- its affected by muscle contraction

c- its more scattered in tendon

d- its in the same series of muscle fibers

41- Which is correct about Golgi tendon

a- Detect limb inspace

b- Detect muscle length


c- Detect ms tension √

In addition to the Golgi tendon organs. Also nerve receptors, Golgi-Mazzoni


corpuscles are located in the fingertips and serve to enclose nerve endings in
this highly sensitive area of the body. They are found beneath the skin and
respond to heat, cold, pressure and other sensory stimuli.

42- the term apraxia refers to?

a- inability to perform purposfull movements √

b- swallowig

c- slow motion

d- involuntary movement
ALSO CALLED "clumsy syndrome"

43- The term dysapraxia refers to?

a- inability to perform purposfull movements √

b- swallowig

c- slow motion

d- involuntary movement

* Dyspraxia : is the partial loss of the ability to co-ordinate and perform skilled,
purposeful movements and gestures with normal accuracy

* Apraxia : is the term that is used to describe the complete loss of this ability

44- myositis ossificans most affected

a- wrist Joint

b- Elbow Joint √

c- Shoulder Joint

d- Knee Joint
45- winging of scapula caused by weakness in

a- serratus anterior √

b- teres minor

c- latissmus dorsi

d- rhomboidus

46- Effect of pool therapy :

a- increase tidal volume

b- increase vital capacity

c- decrease vital capacity √

Because hydro static pressure on chest wall and on abdomen wall which acts
on diaphragm and there is indirect effect on the shift of blood to the thorax
:resulting in decrease vital capacity

47-Pt has tear with rotation movement . Which test not need :

a- drawar

b- lackmen

c- trendburg √

48- Which of the following is not an acceptable long-term goal for a patient
with a complete C7

spinal cord injury?

a- Independence with dressing

b- Driving an automobile.

c- Balance a wheelchair for 30 seconds using a “wheelie”. √

d- Independence with performing a manual cough.


BCOZ: Nerves control elbow extension and some finger extension. Most can
straighten their arm and have normal movement of their shoulders. Can do
most activities of daily living by themselves, but may need assistance with more
difficult tasks May also be able to drive an adapted vehicle Little or no
voluntary control of bowel or bladder, but may be able to manage on their own
with special equipment

49- What is the best way to first exercise the postural (or extensor) musculature
when it is

extremely weak to facilitate muscle control?

a- Isometrically. √

b- Concentrically.

c- Eccentrically.

d- Iso kinetically.

50- An 87-year-old woman presents to an outpatient physical therapy clinic


complaining of pain in the left sacroiliac joint. The examination reveals higher
left anterior superior iliac spine (ASIS) than the right ASIS, shorter leg length
on the left side (measured in supine position), and left side posterior superior
iliac spine (PSIS) lower than the right PSIS. In what position should you place
the patient to perform the correct sacroiliac mobilization of the left innominate?

a- Right sidelying

b- Supine

c- Prone

d- None of the above √ ( BCOZ: in 87 years old the sacroiliac joint


disappeared )
51- A therapist is mobilizing a patient’s right shoulder. The movement taking
place at the joint capsule is not completely to end range. It is a large amplitude
movement from near the beginning of available range to near the end of
available range. What grade of mobilization according to

Maitland, is being performed?

a- Grade I

b- Grade II √

c- Grade III

d- Grade IV

Maitland Joint Mobilization Grading Scale

Grade I- Small amplitude rhythmic oscillating mobilization in early range of


movement

Grade II- Large amplitude rhythmic oscillating mobilization in midrange of


movement

Grade III- Large amplitude rhythmic oscillating mobilization to point of limitation in


range of

movement

Grade IV- Small amplitude rhythmic oscillating mobilization at end range of


movement

Grade V- (Thrust Manipulation) - Small amplitude, quick thrust at end range of


movement

52- If the line of gravity is posterior to the hip joint in standing, on what does
the body first rely to

keep the trunk from moving into excessive lumbar extension?

a- Iliopsoas muscle activity

b- Abdominal muscle activity


c- Anterior pelvic ligaments and the hip joint capsule. √

d- Posterior pelvic ligaments and the hip joint capsule.

BCOZ: In static standing the line of gravity is posterior to the hip joint. The
body relies on the anterior pelvic ligaments and the hip joint capsule. The
iliopsoas may be recruited at times, but anterior ligaments are used first to keep
the trunk from extending in static stance.

53- The therapist is treating a track athlete who specializes in sprinting and
wants to increase his or her speed on the track. To accomplish this goal, the
plan of care should include activities to develop fast-twitch muscle fibers.
Characteristics of this type of fiber include:

a- Fatigues slowly, fiber colors appear red and used more in aerobic

b- Fatigues quickly, fiber colors appear white and used in anaerobic √

c- Fatigues quickly, fiber colors appear white and used more in aerobic

d- Fatigues slowly, fiber colors appear white, and used more in anaerobic

Fast twitch muscles are good for rapid movements like jumping to catch a ball
or sprinting for the bus. They contract quickly, but get tired fast, as they
consume lots of energy.

54- A patient recently diagnosed with multiple sclerosis presents to a physical


therapy clinic. The patient asks the therapist what she needs to avoid with this
condition. Which of the following

should the patient avoid:

a- Hot tubs. √

b- Slightly increased intake of fluids.

c- Application of ice packs.

d- Strength training.
BCOZ: any hot application is prohibited in multiple sclerosis case

55- Which of the following is used to treat a patient referred to physical therapy
with a diagnosis

of Dupuytren’s contracture?

a- Knee continuous passive motion (CPM)

b- Work simulator set for squatting activities.

c- Hand splint. √

d- A two-pound dumbbell

BCOZ: Dupuytren’s contracture is a progressive thickening of the palmar


aponeurosis of the hand. The progression is gradual, and the interphalangeal
joints are pulled into flexion.

56- The therapist in an outpatient physical therapy clinic receives an order to


obtain a shoe orthotic for a patient. After evaluating the patient, the therapist
finds a stage I pressure ulcer on the first metatarsal head. Weight-bearing
surfaces need to be transferred posteriorly. Which orthotic is the most
appropriate for this patient:

a- Scaphoid pad.

b- Thomas heel.

c- Metatarsal pad √

d- Cushion heel.
BCOZ: Metatarsal pads successfully transfer weight onto the metatarsal shafts
of this patient.

A Thomas heel and a scaphoid pad are for patients with excessive pronation. A
cushion heel absorbs shock at contact.

57- At what point in the gait cycle is the center of gravity the lowest?

a- Double support. √

b- Terminal swing.

c- Deceleration.

d- Midstance

BCOZ: Movement of the lowest displacement occurs at heel strike and


double support.

58- history of foot ball players has injured by twisted knee and take out of
game and after little time it swelling and warm . after few days it locks during
climbing stairs and painful and cannot put full weight in walking :

a- patella fracture

b- tibia fracture

c- MCL rupture √

d- Rupture of something of fibers

59- 67-year-old woman presents to an outpatient facility with a diagnosis of


right adhesive capsulitis. The therapist plans to focus mostly on gaining
abduction range of motion. In which direction should the therapist mobilize the
shoulder to gain abduction range of motion:

a- Posteriorly

b- Anteriorly

c- Inferiorly √
60- Capsular tightness has limited your patients ability to fully extend her knee.
Treatment to restore joint motion

a- anterior glide , external rotation of tibia √

b- anterior glide , internal rotation of tibia

c- posterior glide, external rotation of tibia

d- posterior glide, internal rotation of tibia

Ant.glide increase extension

61- Common peroineal nerve supply

a- dorsiflexor

b- dorsiflexor with evertors √

c- planterflexor

BCOZ: common peroineal supply peroneus brevis musle which do dorsi flexion
and eversion

62- Boy have 16 years old have acute knee sprain after chronic we advice

a- knee protection

b- knee immoblization

c- range of motion √

d- hot application

63- A therapist receives an order to evaluate and treat a 76-year-old woman


who was involved in a motor vehicle accident 2 days ago. The patient’s vehicle
was struck in the rear by another vehicle. The patient has normal sensation and
strength in bilateral lower extremities but paralysis and loss of sensation in
bilateral upper extremities. Bowel and bladder function are normal. The patient
most likely has what type of spinal cord injury?

a- Anterior cord syndrome.

b- Brown-Sequard syndrome.

c- Central cord syndrome. √

d- There is no evidence of an incomplete spinal cord lesion.

BCOZ: This scenario describes a central cord lesion. It is common in the


geriatric population after cervical extension injuries (such as whiplash).

64- A 42-year-old receptionist presents to an outpatient physical therapy clinic


complaining of low back pain. The therapist decides that postural modification
needs to be part of the treatment plan. What is the best position for the lower
extremities while the patient is sitting:

a- 90o of hip flexion, 90o of knee flexion, and 10o of dorsiflexion

b- 60o of hip flexion, 90o of knee flexion, and 0o of dorsiflexion

c- 110o of hip flexion, 80o of knee flexion, and 10o of dorsiflexion

d- 90o of hip flexion, 90o of knee flexion, and 0o of dorsiflexion √

65- therapist performed trendlinburg test for pt with (Rt) weakness gluteus
medius , when test is (+ve) when patient perform single limb support on RT
lower limb what is correct of the following about pelvis drop

a- pelvis doesn't drop as the patient lean by trunk to RT

b- pelvis will drop at left side √

c- pelvis doesn't drop as the patient lean by trunk to LT

d- pelvis will drop at Right side


BCOZ: the +ve sign as the patient stands on affected leg and raising sound leg
the pelvis telt on the side of sound leg

66- which of the following is not benefit in physiotherapy :

a- increase muscle fiber size

b- increase heart rate

c- increase ligaments and tendons √

d- increase the power structure of bones

67- anterior horn cell in gray matter not include

a- ventral root

b- alpha motor neuron

c- gamma motor neuron

d- sensory neuron √
VENTRAL ROOT = MOTOR PATHWAY

68- what of these statement NOT prameters of functional skeletal muscle


function

a- balance

b- produce force

c- joint mobility √

d- endurance

BCOZ: There are four major functions of a skeletal muscle : carry out movements
of the body; support the body; provide for heat regulation of the body; and
maintain posture and muscle tone of the body.
69- 60 YEARS FEMALE has acute severe RH.ARTHRITIS PT in future do
strengthining ex

which one is better

a- isometric ex √

b- isotonic ex

c- iso kinetic ex

d- immobilization

70- the ability of the posture to modify external environment and preserve
aligment of one body

part to another?

a- Postural control √

b- Strength

c- Balance

71- patient with emphysema and has barrel chest which statement document it

a- depressed ribs angel

b- increase transverse diameter

c- increase ant.-post diameter of chest √

d- decrease intercostal space

72- patient with emphysema which not suspected to found :

a- depressed ribs angel

b- increase transverse diameter

c- decrease ant.-post diameter of chest √

d- decrease intercostal space


Barrel chest also refers to an increase in the anterior posterior diameter of the
chest wall resembling the shape of a barrel, most often associated with
emphysema. There are two main causes of the barrel chest phenomena in
emphysema:

Increased compliance of the lungs leads to the accumulation of air pockets inside
the thoracic cavity.

Increased compliance of the lungs increases the intrathoracic pressure. This


increase in pressure allows the chest wall to naturally expand outward.

73- patient with emphysema and has brallel chest which statement document it

a- depressed ribs ange

b- increase transverse diameter

c- decrease ant post diameter

d- increase intercostal space √


Also increase ant- post diameter

74- A patient with adhesive capsulitis of the glenohumeral joint should


demonstrate the greatest limitation of motion when performing shoulder

a- flexion.

b- abduction.

c- medial rotation.

d- lateral rotation. √

75- coronary artery supply heart during

a- systole and diastole

b- in between

c- diastole √
76- coronary arteries suplying heart when?

a- diastole √

b- systole

c- in between

d- not related to systole or diastole supplying heart every time

77- coronary arteries arise from aorta at

a- left atrim

b- base of aorta at level of aortic valve √

c- at a point near to the base of aorta

d- arise at point of left ventricle

coronary artery arises from the aorta above the left cusp of the aortic valve

78- which of the following is not needed in trapeius muscle test:

a- head flexion √

b- head extention

c- head rotation

d- head lateral flexion

79- apt with CVA referral for increase stability you could use

a- hold relax technique

b- rhythmic stabilization √

c- rhythmic initiation

d- repeated contractions
Rhythmic Stabilization: and Alternating Isometrics are very similar in that
they both encourage stability of the trunk, hip, and shoulder girdle

80-internal rot and external rot on which plane can performed

a- Longitudinal plane around horizontal axis

b- Axial plane around horizontal axis

c-transeverse plane around longitudinal axis √

81- Abduction to Adduction in which plan occurs

a- From sagital plan to coronal plan √

b- From coronal plan to sagital plan

c- From longitudinal to transverse

Flexion, extension and hyperextension occur primarily in the sagittal plane-


frontal axis of

the body (i.e. neck, shoulder, spine, hip, knee and ankles).

Lateral flexion and lateral extension occur primarily in the frontal plane-sagittal
axis of the

body (i.e. neck and spine).

Adduction and abduction also occur primarily in the frontal plane-sagittal axis of
the body (i.e. shoulder and hip).

Internal and external rotation, horizontal flexion and extension, supination and
pronation

all occur primarily in the transverse plane-vertical axis

Frontal Plane = coronal plane

Sagittal Plane = vertical plane

Transverse Plane = horizontal plan.


82- the type of exercise where distal part is fixed and proximal part moving
over distal part

a- open packed position

b- closed packed position √

c- curcit ex

d- isometric ex

83- patient reffered to u post lamectoy operation for physio therapy with
examination found heave

adhesions over operative region best modalty to soften this scar is ‫ ر‬# $‫آ‬

a- us √

b- sw

c- hot backs

d- IR

84- pt is refferd to physio dept with diagnosis of flexion deformity of Rt knee


with examination we will found ‫ ر‬# $‫آ‬

a- flexion knee + planter flexion + shortening of qudricepes

b- flexion knee + dorsi flexion + shortening of hamstring √

c- flexion knee + no ankle changes + shortening of quadriceps

d- flexion knee + planter flexion + lenghing of soles

43- patient during walking raise his hip to clear toes from ground due to ‫ ر‬#
$‫آ‬

a- paralysis of dorsiflexors √

b- paralysis of planter flexors

c- paralysis of quadriceps
85- pt with lordosis from standing and +ve Thomas test due to

a- strong lumbar ext

b- fixed flex deformity √

c- hamstring strain

86- Which isn't correct about hip during gait

a- max hip flex occure in terminal swing

b- slight hip ext before inicial contact

c- hip ext occure during double limb support

d- max hip ex occure in terminal swing √

87- protraction and retraction of shoulder occurs at

a- glenohumeral joint

b- scapulothoracic interface √

c- acromioclavicular joint

88- ulnar nerve late palsy due to fracture of

a- olocranion fossa

b- medial epicondyle √

c- lateral epicondyle

89- when u pushing automobile with shoulder flexion ,elbow extended the
main muscle of action is

a- serratus anterior √

b- biceps brachial

c- pectorals major

d- triceps brachials
The most frequent site of compression is the proximal forearm in the area of
the supinator muscle and involving

90- child came to u with erbs palsy c5,c6

diagnosed by erb engram where is the affection

a- weakness of external rotators

b- spasticity of external rotators

c- Up normal pattern of movement + weakness of ant deltoid + biceps spasm √

d- weakness elbow extensors

91- Radial nerve injury due to :

a- proximal part of humerus

b- shaft of humerus √

c- surgical neck of humerus

92- Common site of redial nerve lesion:

a- wrist

b- elbow

c- shoulder

d- spiral grove of humerus √

93- Radial nerve most probable compressed region ‫ ر‬# $‫آ‬

a- elbow √

b- finger

c- shoulder

d- wrist
94- supracondyler ridge of humerus injury to

a- Radial nerve

b- Brachial artery √

c- Ulnar

d- Median

95- Volkman's ischemic contracture occur due to :

a- fracture of medial condyle of humerus

b- fracture of lateral condyle of humerus

c- fracture lower third of humerus √

d- surgical neck fracture

96- VOLKMANS ISCHEMIC CONTRACTURE occur due to ‫ ر‬# $‫آ‬

a- fracture of medial epcondyle of humerus

b- fracture lower third of humerus √

c- fracture of lat epicondyle of humerus

d- surgical neck

97- Generally the ROM depends on : ‫ ر‬# $‫ا آ‬

a- elasticity of muscle , ligament and tendon

b- bony structure √

c- tightness of muscle, ligament and tendon

d- bulk of the soft tissue

98- sudeck's atrophy which is not true :

a- sever burning pain

b- sympathetic hyperactivity √

c- trophic changes occur


d- more perspiration

99- which is not true about sudeck's atrophy

a- osteoporosis of bones of hand

b- fully curable by taking sympathetic nerve blocker √

BCOZ :The symptoms are variable and will present themselves differently from
patient to patient. The main symptoms begin with a generalised burning pain;
this is usually followed by changes in the condition of the skin, which may
become shiny. In severe cases, the affected body part may swell and, due to
sympathetic nervous system dysfunction, the body part may perspire more than
usual. Because of the pain the patient may not want to move the injured part.
This leads to muscle wastage and a viscious cycle where stiffness and pain
become worse. If the condition persists there may be adverse changes to the
condition of the underlying bone.

100- to treat patient with deep heat, which is not form of deep heat ‫ ر‬# $‫آ‬

a- US

b- IR √

c- MWD

d- SWD

101- young patient with post fractue elbow deformity , referred for pt , what's
contraindicated to avoid myositis ossificans: vigorous passive stretch

102- an athlete with stable fracture neck of humerus , refereed to you to


improve mobility. Which form of exercise will you chose

a- isometric

b- resistive exercise in all directions √

103- an athlete with unstable fracture neck of humerus , refereed to you to


improve mobility. which form of exercise will you chose

a- isometric √

b- resistive exercise in all directions


104- when make mobilization for stiffness shoulder go to start

a- anterior

b- posterior

c- lateral

d- inferior √

105- Severe kyphoscoliosis will lead to

a- Left rather than right ventricular failure

b- Right rather than left ventricular failure

c- Frequent respiratory infections √

d- No cardiac abnormalities

106- if the proximal bone of joint fixed the distal part of joint move in
sequence so this is

a- ACL Training

b- closed chain ex

c- open chain ex √

107- Patient with shoulder pain during evaluation there is no passive or active
ROM CAUSED BY

a- Frozen shoulder ( adhesive capsulitis) √

b- Supra spinatus tendenitis

c- Rotator cuff tear

d- Acromioclavicular dysfunction

108- Knee joint ROM:

a- flexion 110 extension 0

b- flexion 130 extension 0

c- flexion 140 extension 0 √


109- Normal ROM of hip joint :

a- extension 35 flexion 125

b- extension 10 flexion 125 √

c- extension 0 flexion 125

110- Patient with spinal cord injury (T6) level you can expect all of this from
him except

a- Independent transfer by manual wheel chair

b- Independent transfer by sliding board

c- Independent bowel and bladder control √

111- anterior taleofibular ligament to be assessed resist

a- planter flex + ev

b- dorsi + inv

c- dorsi + eversion

d- plant + inv √

BCOZ: injury mechanism is planterflex+inversion, so to assessed the ligament


with resisted movement in the same direction for ligament and tendon but for
muscle resisted movement in opposite direction for muscle (dorsiflex.+eversion)

112- during ex theres subtalar pronation that’s mean ‫ ر‬# $‫آ‬

a- femur,tibia,pelvic internal rotation

b- femur,tibia,pelvic ext rot

c- femur ,tibia int rot with pelvic ext rot √

d- femur ,tibia internal rot with pelvic int rot

* primary Raynaud's syndrome is Mild bilateral symmetric attacks affecting


both hands Not treated by ICE
113- Raynaud’s disease ‫ ر‬# $‫آ‬

a- Bilateral symmetrical √

b- Unilateral symmetrical c- Cyanotic unsymmetrical


Secondary raynaud's syndrome is unilateral assymetrical , cyanosis

114- In Raynaud's diseaes it is?

a- vascular insufficiency disease √

b- intermittent claudication

c- thrombosis in lower limb

d- varicose veins

115- In Raynaud's which skin color

a- Bilateral asymmetrical

b- Unilateral asymmetrical

c- Lateral asymmetrical

d- Asymmetrical & cyanosis √

‫ر‬ ‫ض‬ ‫ا‬ ‫ال‬ ‫ا‬ bilateral symeterical

‫ا ا‬ ‫ر ا‬ ‫ا ن‬ ‫ال‬ ‫ا‬ ‫و‬

116- Ice massaging is contraindicated with:

a- Inflammation

b- Raynaud’s disease √

c- Muscle spasm

d- Acute burn

117- Burger’s disease associated with

a- smoking √

b- athlete performing max effort


118- Burger's disease pt complain of pain after walking in calf m.s in ttt your
goal is to

a- Decrease ms spasm

b- Strength hamstring

c- Increase venous return √

119- myositis ossificans most affected

a- wrist J

b- Elbow J √

c- Shoulder J

d- Knee J

120- when you sit up patient from long sitting position after long period of of
bed rest following major back surgery the patient experience an orthostatic
hypotension and suddenly will fallwhat is your response

a- immediately return patient to original long sitting √

b- call physician

c- write down this in patient review

d- assure him it s ok don't worry it is normal decrease blood pressure due to


long rest period

121- patient with winged scapula and excessive internal rotation needs to

a- stretch middle and lower trapezius

b- Stretch pectoralis major and strength middle trapezius √

c- strength pectoralis major and Stretch upper trapezius

122- from supine when there is shortening of this muscle the arm raised above
table and not

dowen the muscle is

a- biceps brachii
b- brachioradialis

c- brachialis

d- pectoralis major √

123- from supine when there is shortening of this muscle the forearm raised
above table and not down the muscle is

a- biceps brachii √

b- brachioradialis

c- brachialis

d- pectoralis major

124- The angle between Neck of femur and Shaft of femur :

a- 90 Degree

b- 120 Degree √

c- 150 Degree

d- 170 Degree

125- pt has median Nerve compression assesst him by

a- Weak of medial 2 fingers & decrease grasping objects

b- Numbness of 3 ½ fingers & weak opponents ms √

126- spastic diplegia means

a- primarily affecting the legs but arms are less involved and less severe √

b- only legs are involved

c- only arms are involved

d- one limb is involved

127- child can kick the ball in

a- 12 month

b- 24 month
c- 30 month √

128- what is the best electrical modalities chosen to treat infective wound

a- low level LASER

b- UV √

c- Galvanic Stimulation

d- S W

129- all the following is true about spascity except :

a- usually occur cerebral palsy infant

b- occur in SCI

c- associated with lower motor neuron lesion √

d- can cause hip dislocation

130- cryotherapy contraindication

a- peripheral artery disease √

b- acute injury

c- acute inflammation

131- What is not true about spine bifida :

a- Inadequate of folic acid intake

b- Diagnostic of fatel by ultrasound

c- The injury related of degree of affect of spine

d- Major affected of thoracic region √

132- prosthesis means

a- field which fabricate devices to replace missing limb √

b- persons who fabricate prosthesis

c- extenl device support existing limb


d- field which fabricate any type of supporting devices

133- prothetist ‫ ر‬# $‫آ‬

a- device that replace the amputated limb of the body

b- material from which the device are fabricated

c- person who fabricate and design the prosthesis √

d- company which fabricates the device

134- orthosis

a- device that replace the absent part of the body

b- an external device used in rehabilitation of patients with neuromuscular and


musculoskeletal disorders. √

c- person who fabricates orthosis

d- company which fabricates the orthosis

135- 32 years old foot player come to the physiotherapist with pain at the left
knee joint while the physiotherapist taking history patient said that the injury
occurred when he rotates to the right on weight bearing to the left knee . it was
flexed 60 degree. that means that the injury is at

a- medial collateral ligament

b- semilunar cartilage at the knee √

c- femoral condyles

d- upper shaft of the tibia

semilunar cartilage = meniscus

136- all is true about med. meniscus except

a- More injured than lat.men

b- Absorb of about 90% of load on knee joint

c- Injury from hyper ext injury √


137- all of the following muscles affected in Tennis elbow except

a- Extensor carpi radialis brevis

b- Extensor carpi ulnaris

c- Extensor digitorum

d- flexor carpi ulnaris √

138- About Tennis elbow all the following is true except

a- also called lateral epichondylitis

b- precipated by ulnar neuritis √

c- usually occur in dominant side

139- pattelar tendon loading orthosis (take idea about it ) from choice

a- reduce stress on hip by 50 %

b- can uses with assistive devices of both hands

c- carry overall leg st √

140- if there is weakness of right gluteus medius dropping of pelvis occurs at:

a- right pelvic drop at stance phase

b- left pelvic drop at stance phase

c- right pelvic drop at swing phase

d- left pelvic drop at swing phase √

The Trendelenburg test is used to assess hip stability. +ve sign=droping


in the unsupported sound(swing phase of gait)

141- 32 years old - pt with sever spinal deformity has difficulty in


respiration(dysapnea ) is due to :

a- ms weakness

b- usage of spinal orthotic

c- decrease lung capacity √


142- pt complain of groin pain with limited abduction with external rotation ;
you expect the

diagosis is ‫) ر‬0 D‫ آ‬:

a- sportsman hernia

b- adductor strain √

c- abductor sprain

d- external obliques weakness

143- which modality has greater effect in decrease pain in knee joint in fatty
ptn:

a- ultra sonic √

b- infrared

c- hotpack

d- parafin wax
144- U.S can be mostly absorbed by skeletal muscle as it mostly contain

a- adipose tissue

b- connective tissue

c- nerve endings

d- protein √

145- in normal person which can prevent lung collapse

a- forced expiratory volume

b- vital capacity

c- forced inspiratory volume

d- dead space +lung compliance +surfactant √

146- voltage of heart

a- -90 √

b- +90

c- +60

d- -30

147- A patient is referred to physical therapy with a ruptured Achilles tendon.


The patient reports that he was moving the lawn and was going down a steep
incline when he felt a sharp, sudden pain in the left heel region. Which of the
following would be proper location for the Achilles tendon insertion:

a- Talus

b- Calcaneus √

c- Cuboid

d- Navicular

148- Electrically insulating material in a nerve fibre is

a- Nodes of Ranvier

b- Axon
c- Dendrites

d- Myelin sheath √

149- patient with sever back pain, after examination you find disc herniation;
which one of the following will be the least to prove the condition

a- MRI

b- CT

c- plain X-ray √

d- mylogram

150- Microwaves are strongly absorbed by

a- Water √

b- Fat

c- Protein

d- Muscle tissue

151- patient has weak muscle in hip abduction the physiotherapist give him
poor grade that mean patient unable :

a- Complete full range of motion in supine position

b- incomplete full range of motion in supine position

c- complete full range of motion in side lying √

d- incomplete full range of motion in side lying

BCOZ: POOR MAY COMPLETE OR INCOMPLETE ROM WITH GRAVITY


ELIMINATED AND SUPINE POSTION IS GRAVITY ELIMINATED FOR
ABDUCTION,AS LONG AS THE EXAMINER GAVE A CHOICE INCOPLETE
ROM
152- percusiion is applied from

a- 1 to 3 min

b- 2 to 4 min

c- 2 to 5 min √

d- 2 to 6 min

153- patient coming to you suffering from pain at calves due to walk in 200
feets this patient has?

a- calf strain

b- arterial insufficiency √

c- thrombosis

d- varicos vein

154- patient complain from leg pain after walking 200 feet, pain is removed by
rest, diagnosis is:

a- peripheral vascular disease √

b- nerve lesion

c- calf muscle spasm

BCOZ: I THINK arterial insufficiency>>FATIGUE>>>PAIN AND IN


OTHER CHOICES

THE PATIENT CAN’T WALK ALL THIS LONG DISTANCE

155- in SCI (spinal cord injury) which of the following in not of


complications?

a- hyperreflexia

b- systemic hypertention √

c- pressur sorses

d- orthostatic hypotention
156- An athlete with stable fracture of neck of humerus, referred to you to
improve mobility,

which form of exercise will you choose

a- Isometric

b- Resistive exercise in all directions √

I chose resistive exercise because the fracture is stable and the aim of exercise is
mobility

157- while you examine a patient you find decreased tendon jerk at Achilles
tendon, what is the level of lesion ‫ ر‬# $‫آ‬

a- l4 – l5

b- l5 – s1

c- s1 – s2 √

158- Reflex of quadriceps ms

a- L4 √

b- L5

c- S1

d- S2

Achilles tendon S1-S2 & hamstring tendon reflex L5-S1 & Quadriceps ms
reflex L4-5

159- in which location the median nerve freqently or mostly entrapped?

a- forearm

b- arm

c- elbow

d- wrist √
160- the arterioles characterized with?

a- large thick walls supplying organs

b- a networks of small vessels have thin walls

c- a networks of small vessels have thick walls √

161- arterioles are

a- have muscular walls and move blood from heart to body tissues √

b- move blood from tissues to heart

162- in normal milestone baby can jump by both legs in swing at ?

a- 12 monthes

b- 24 monthes

c- 18 monthes

d- 30 monthes √

163- Baby can kick the ball at

a- 24 months

b- 30 months √

164- in normal milestone baby can jump by both legs in swing at ?

a- 12months

b- 24 months

c- 18 months

d- 30 months √

165- infant can make rolling at age of

a- 3 months

b- 4 months

c- 6 months √
d- 12 months

At 6 months the baby complete rolling but initiation of rolling starts from 4
monthes

166- when measuring ROM of the wrist ; put the axis in :

a- trapezoid

b- trapezium

c- lunte

d- capitate √ for radial and ulnar deviation

Wrist ext and flex : ulnar styloid process

167- Rickets is due to deficiency of vitamin?

a- a

b- b

c- k

d- d √

168- Patient with open wond in lumber area infected by fungus and bacteria
what is the best

modalities used

a- LASER

b- HVGS

c- UV √

169- patient with controlled diabetus mellitus adviced with exercise because

a- increase blood glucose uptake by ms √

b- no effect
c- increase insulin metabolism

170- Burns caused due to constant direct currents are called

a- Galvanic burns

b- Chemical burns

c- Electrolytic burns

d- All of the above √

171- valgus position of foot

a- planterflexion

b- dorsiflexion

c- eversion √

d- inversion

Varus is adduction of metatarsal joints and valgus is abduction of metatarsal joints

172- PATIENT move crutches first then move legs to them

a- swing through

b- swing to √

c- 2 point gait

173- doctors see on patients door word(terminal illness)what does this mean

a- disease with eldery people

b- with adult people

c- Irrevsable course √

d- neurologic origin
174- all of the following are objective measurments used in evaluation except:

a- time used for performance activity

b- visual analogue scale

c- goniometric ROM

d- disabitity decies √

175- patient with deep muscle strain with pain treatment by

a- pulsed 1 MHZ √

b- pulsed 3 MHZ

c- Non pulsed 1MHZ

d- non pulsed 5

176- patient with disc herniation and decrease back motion P.T assist

a- sternocostal angle

b- lumbosacral angle √

c- cobbs angle

d- cubitis angle

Cobbs angle determined in scoliosis cases only

177- prevention of transmition pain beyond the dorsal horn

a- periphral N injury

b- gate controle theory √

c- spinal cord injury

d- opiat theory
178- which statement incorrect about passive movement

a- passive movement segment must hold gently

b- proximal part must supported

c- passive movement from proximal to distal to increase lymph and venous


return √

179- closed pack position of knee is

a- ext knee

b- flex knee

c- ext knee with lat tibial rot √

180- in phase 1 cardiac rehabilition aim;

a- prevent bed rest complication begin with passive movement to active


movement √

b- increase ex endurance capcity

c- assist in progress functional ADL

181- which nerve which give back of the arm and forearm

a- radial √

b- ulner

c- median

d- musculocutaneous

182- muscle spindle is stimulated by

a- Passive stretch √

b- Passive movement

c- Active exercises
183- patient come with sever elbow pain we can use

a- Heat application

b- Cold application √

c- Faradic stimulation

If acute trauma use cold but if chronic use heat but if acute
inflammation so use cold

184- convulsant stage in polio takes from

a- 2 weeks to 2 years √

b- 2weeks to 4 years

c- 2weeks to end of life

185- quadriplegic patient have cerevical 6 fracture what is the last thing he can
be independent

a- feeding √

b- urinary & bowel action

c- use wheel chair

d- transfer

186- the number of joint and bones in the foot

a- 23 joint and 25 bones

b- 33 joint and 26 bones √

c- 23 joints and 17 bones

187- coax vara present as abnormality of

a- proximal femur √

b- distal femur

c- proximal tibia

d- distal tibia
188- In taping an athlete’s ankle prophylactically before a football game, in
what position should the ankle be slightly positioned before taping to provide
the most protection against an ankle sprain:

a- Inversion, dorsiflexion, abduction

b- Eversion, plantarflexion, adduction

c- Eversion, dorsiflexion, abduction √

d- Inversion, plantarflexion, adduction

189- fracture which occurred due to repeated minor trauma is

a- stress fracture √

b- fatigue fracture

c- compressed fracture

190- when stimulated fascial muscles which muscle not contract?

a- frontals

b- orbiculars oris

c- masster √

d- levator lbii

SUPPLIED BY THE MOTOR PART OF TRIGEMINAL NERVE


191- therapist performed trendlinburg test for pateint with (Rt) weakness
gluteus medius , when test is (+ ve) when patient perform single limb support
on RT lower limb , what is correct of the following about pelvis drop:

a- pelvis dosent drop as the patient lean by trunk to RT

b- pelvis wiil drop at left side √

c- pelvis will drop at Right side

d- pelvis doesn't drop as the patient lean by trunk to LT

‫ا; ل‬ 9 ‫ط‬0 8 453 ‫ض وه‬03 ‫ازن ا‬01 ‫ ر‬$‫ ه‬. , )- *+ ‫'ن ا )(! ا‬

192- if there is weakness of right gluteus medius dropping of pelvis occurs at:

a- right pelvic drop at stance phase

b- left pelvic drop at stance phase

c- right pelvic drop at swing phase

d- left pelvic drop at swing phase √

193- in thrombosis we do :

a- deep breathing exercises to enhance blood to heart.

b- deep breathing exercises to to improve lung capacity

c- no deep breathing at all √

194- elderly person 65 year of age during his gait:

a- swing phase more than 40%

b- swing phase less than 40% √

c- swing phase 40%


195- in wax therapy it`s temperature should be kept at?

a- 30-35

b- 40-44 √

c- 50-55

196- the main muscle resposiple for climbing stairs and coming from reclined
position bringing

knee to chest ?

a- rectus femoris

b- illiopsoas √

c- quadratus lumborum

d- pectinieus

197- MAXIMUM knee flexion in gait at ?

a- initial swing

b- mid swing √

c- initial contact

d- foot flat

198- in ttt of anterior neck burn 3rd degree which is contra indicated?

a- stretchig scar tissue so put head in extention

b- ultra sonic to decrease adhesions

c- put the head in flexion to avoid deformity √

199- anterior pelvic tilting is causing exaggerated lumbar lordosis due to?

a- adductor tightness

b- tightness of tensor facialata

c- bilateral fixed hip flexor contracture √

d- tightness in hamstring ms
200- If the proximal bone of joint fixed the distal part of joint move in
sequence so this is a-ACL Training

b- closed chain ex

c- open chain ex √

d- non of the above

201- in chondromalacia consider in the ttt:

a- patellar tapping and strenthening of the quadriceps √

b- strenthening vastus medialis only

c- strenthening vastus lateralis only

d- strenthening of hamstring

202- which of following not risk factor for osteoporosis?

a- enclosed family history

b- fracture in gym exercises √

c- low calcium intake

d- corticosteroid medications

203- patient wearing a below knee prosthesis complain of pain during walking
response of therapist will be:

a- keep wearing

b- refer to physician

c- refer to orthotics or prosthesis specialist √

d- say him its normal initially

204- A teenager comes to an outpatient facility with complaints of pain at the


tibial tubercle when playing basketball. The therapist notices that the tubercles
are abnormally pronounced on bilateral knees. What condition does the patient
most likely have:

a- Jumper’s knees.

b- Anterior cruciate ligament sprain.


c- Osgood-Schlatter disease. √

d- A & C

205- A therapist receives an order to evaluate a 72-year-old woman who has


suffered a recent stroke. The therapist needs to focus on pregait activities.
Which of the proprioceptive neuromuscular facilitation (PNF) diagonals best
encourages normal gait :

a- D1 √

b- D2

c- PNF is contraindicated

d- Pelvic PNF patterns only.

206- To treat effectively most patients with Parkinson’s disease, the therapist
should emphasize which proprioceptive neuromuscular facilitation (PNF)
pattern for the upper extremities :

a- D2 extension

b- D2 flexion √

c- D1 extension

d- D1 flexion

BCOZ: D2 flexion patterns support upper trunk extension, which is important


for patients with Parkinson’s disease who tend to develop excessive kyphosis.

And D1 extension for lower extremity in parkinsonism .


207- A pitcher is exercising in a clinic with a sports cord mounted behind and
above his head. The pitcher simulates pitching motion using the sports cord as
resistance. Which proprioceptive neuromuscular facilitation (PNF) diagonal is
the pitcher using to strengthen the muscles involved in pitching a baseball :

a- D1 extension

b- D1 flexion

c- D2 extension √

d- D2 flexion.

208- During evaluation of a patient, the therapist observes significant posterior


trunk lean at initial contact (heel strike). Which of the following is the most
likely muscle that the therapist needs to focus on during the exercise session in
order to minimize this gait deviation:

a- Gluteus medius

b- Gluteus maximus √

c- Quadriceps

d- Hamstrings

BCOZ: This gait deviation is caused by the patient leaning back to decrease the
flexion moment created at the hip at initial contact. The gluteus maximus is most
responsible for counteracting this flexion moment

209- what is not correct about bone

a- contain 90% of inorganic tissues √

b- compose of 2 types sponge ,compact

c- bone is hard

d- has blood supply nerve endings


210- A posterior lateral herniation of the lumbar disc between vertebrae L4 and
L5 most likely

results in damage to which nerve root?

a- L4 √

b- L5

c- L4 and L5

d- L5 and S1

From this image we can find that:

Nerve Root arise from the lower disc ex.:

Nerve Root 5 arise from behind L5-S1

211- What portion of the adult knee meniscus is vascularized:

a- Outer edges √

b- Inner edges

c- The entire meniscus is vascular

d- The entire meniscus is avascular


212- A 17-year-old football player is referred to the outpatient physical therapy
clinic with a diagnosis of a recent third-degree medial collateral ligament sprain
of the knee. The patient wishes to return to playing football as soon as possible.
Which of the below is the best protocol:

a- Fit the patient with a brace that prevents him from actively moving the knee
into the last available 20o of extension. Prescribe general lower extremity
strengthening with the exception of sidelying hip adduction. √

b- Do not fit the patient with a brace. All lower extremity strengthening
exercises are indicated.

c- Fit the patient with a brace that prevents him from actively moving the knee
into the last available 20o ¬of extension. Avoid all open-chain strengthening
for the lower extremity.

d- Do not fit the patient with a brace. Prescribe general lower extremity
strengthening with the exception of sidelying hip adduction

BCOZ :The screw home mechanism that is present in the last few degrees of
terminal knee extension stresses the MCL. Sidelying hip adduction also places the
MCL in position of stretch

213- What is the most likely cause of anterior pelvic tilt during initial contact
(heel strike)?

a- Weak abdominals √

b- Tight hamstrings

c- Weak abductors

d- Back pain
214- A 31-year-old man has loss of vision in one eye, staggering gait,
numbness in bilateral upper extremities, and decreased bowel and bladder
control. The episodes of the above symptoms have occurred every few weeks
for the past 6 months. Each episode has been slightly worse than the first. What
is the most likely condition :

a- Parkinson’s disease

b- Guillain Barre syndrome

c- Multiple sclerosis √

d- Amyotrophic lateral sclerosis

215- prevention of transmission of pain beyond the dorsal horn

a- peripheral n injury

b- gate control theory √

c- spinal cord injury

d- opiate theory

216- patient with L 3 myotome injury for assessment you resist

a- Hip extensors.

b- Knee flexors.

c- Knee extensors √

217- In what position should the therapist place the upper extremity to palpate
the supraspinatus tendon:

a- Full abduction, full flexion and full external rotation

b- Full abduction, full flexion and full internal rotation

c- Full adduction, full external rotation, and full extension

d- Full adduction, full internal rotation and full extension √


218- The therapist works in a cardiac rehabilitation setting. Which of the
following types of exercises are most likely to be harmful to a 64-year-old man
with a history of myocardial infarction:

a- Concentric

b- Eccentric

c- Aerobic

d- Isometric √

219- Which of the following is the most important to assess first during an
evaluation of a patient

with a recent stroke?

a- Sensory status

b- Motor control

c- Mental status. √

d- Ambulation potential

220- The therapist receives a referral to evaluate a patient with a boutonniere


deformity. With this injury, the involved finger usually presents in the position
of

a- Flexion of the proximal interphalangeal (PIP) joint and flexion of the distal
interphalangeal (DIP) joint.

b- Extension of the PIP joint and flexion of the DIP joint.

c- Flexion of the PIP joint and extension of the DIP joint. √ sure

d- Extension of the PIP joint and extension of the DIP


221- PNF initially voulantny relaxation then passive or active assisted
contration and lastly resistive mov it’s

a- Rhythmic intiation √

b- Rhythmic stabilization

c- Contract-relax

d- Hold-relax

222- Which of the following PNF techniques start with relaxation, followed by
active then active assisted and ends with mild resistance:

a- Rhythmic Initiation √

b- Rhythmic Stabilization

c- Hold Relax

d- Contract Relax

223- in wax therapy it`s temperature should be kept at

a- 30-35

b- 40-44 √

c- 50-53

224- bunion deformity is

a- hallucis varus

b- hallucis rigidus

c- hallucis valgus √

d- hammar toe
A hallux abducto valgus deformity, commonly called a bunion, is a deformity
characterized by lateral deviation of the great toe, often erroneously described as an
enlargement of bone or tissue around the joint at the head of the big toe

225- fixed stiffness of 1st IPJ joint of big toe called

a- hallucis varus

b- hallucis valgus

c- hammar toe

d- hallucis rigidus √

Dorsal bunions, a type of arthritis, can be one of the more common causes of
hallux limitus or hallux rigidus

226- U.S.FRQUENCIES ‫ ر‬# $‫آ‬

a- 1-3 MHZ √

b- 1-3 HZ

c- 1-3 KHZ

227- which is incorrect about burn in shoulder and pectoral region ‫ ر‬# $‫آ‬

a- put airplane splint

b- put in ext and abduction

c- put in flex add int rot √

228- pt has median Nerve compression assesst him by

a- Weak of medial 2 fingers & decrease grasping objects

b- Numpness of 3 ½ fingers & weak opponents ms √


229- accessory movement is not

a- necessary to complete of joints ROM

b- Passively occur

c- under voluntary control √

230- infrared can cause burn . what’s distance should be between patient and
the device

a- 75 cm √ ‫ ر‬# $‫آ‬

b- 30 cm

c- 10 cm

231- WHICH STATEMENT NOT CORRECT ABOUT TALIPES EQUINO


VARUS

a- club foot

b- maybe congenital

c- p,f and inversion

d- d.f and inversion √

232- in TALIPES EQUINO VARUS?

a- D.F eversion adducted

b- P.F inversion abducted

c- D.F eversion abducted

d- P.F INVERSION adducted √


233- valgus position of foot

a- planterflexion

b- dorsiflexion

c- eversion √

d- inversion

234- in Erbs palsy the most affected muscle

a- elbow flexors and supinators

b- shoulder flexor and abductors

c- medial rotators

d- a and b √

235- from supine when there is shortening of this ms the arm raised above table
and not down ,the

ms is?

a- biceps

b- brachioradialis

c- brachialis

d- pectoralis major √

236- in carpal tunnel syndrome the most affected ms is

a- flexor degotorum profundus √

b- flexor carpi ulnaris

c- adductor
237- which muscle doesn't affected in carpal

tunnel syndrome

a- flexor pollicis longus ◊ median

b- adductor pollicis √ ◊ ulnar

c- abductor pollicis previs ◊ median

238- Which of the following muscles is not supplied by the median nerve?

a- Flexor carpi radialis ◊ median

b- Flexor digitorum superficial is ◊ median

c- Flexor pollicus longus ◊ median

d- Abductor pollicus longus. √ ◊ Radial

e- Flexor pollicis brevis ◊ median nerve & The deep part by ulnar nerve

f- Abductor pollicis brevis ◊ median

239- 15 yr patient with carpel tunnel syndrome what's the expected symptoms:

1- weakness in the 2 middle fingers and loss grip.

2- loss of sensation in the first 3 lateral fingers and weakness of the flexor
policis √

240- to treat patient with deep heat, which is not form of deep heat

a- SWD

b- MWD
c- IR ( infra red) √

d- US ( ultra sound)

241- which of the following doesn’t use body oxygen

a- Aerobic system

b- Anaerobic system √

242- hip ROM

a- 10 degrees extension 125 flexion √

b- 35 extension 125 flexion

c- No extension 125 flexion

243- 25 years old with burn of dermis, epidermis, and subcutaneous tissues the
type of burn

a- medium thickness

b- partial thickness

c- full thickness √

d- superficial partial thickness

244- burn affecting epidermis and superficial part of dermis

a- superficial partial thickness √

b- deep partial thickness

c- full thickness
245- spastic diplegia means

a- primarily affecting the legs but arms are less involved and less severe √

b- only legs are involved

c- only arms are involved

d- one limb is involved

246- all the following is direct physiological =effect of hot back except

a- increase skin temperature

b- increase metabolic rate

c- decrease muscle spasm √ sure

d- local vasodilatation of capillaries


247- type of contraction that make shortening of muscle length

a- Eccentric

b- Isometric

c- Concentric √

248- patient with a contaminated surgical wound on lumbar region, what would
you use to clean

the wound

a- infrared

b- ultraviolet √

c- shortwave

d- low laser

249- what is best for scar tissue ‫ ر‬# $‫آ‬

a- ultra sound √

b- infrared

c- shortwave

250-what is the best electrical modalities chosen to treat infective wound

b- low level LASER

c- UV √

d- Galvanic Stimulation

251- Strain tendon use

a- non pulsed 3 mh

b- non pulsed 1mh

c- pulsed 3mh √ because tendon superficial

d- pulsed 1mh
252- using the following U.S parameters to heat a deep strain ?

a- pulsed 1 Mh

b- continuos 1 Mh √

c- pulsed 5 Mh

d- continuos 5 Mh

Heat so use continous and deep so use 1 mhz

253- all of the following is direct physiological effect of hot back except

a- increase tempreture

b- increase metabolic rate

c- decrease muscle spasm √ sure

d- local vasodilatation

254- Motor area 4

a- Control motor activity of opposite side of the body √

b- Control motor activity of same side of the body

c- Receive sensation

Called Broadmann's area or somatomotor area

255- patient with anterior neck pain which is contraindicated

a- put patient in neck flexion position √

b- use rigid neck orthoses

c- neck extension
256- What is the most likely cause of anterior pelvic tilt during initial contact
(heel strike):

a- Weak abdominals √

b- Tight hamstrings

c- Weak abductors

d- Back pain

257- child came to u with erbs palsy c5,c6 diagnosed by erb engram where is
the affection

a- weakness of external rotators

b- spasticity of external rotators

c- Up normal pattern of movement +weakness of ant deltoid +biceps spasm √

d- weakness elbow extensors

258- patient with elbow pain for 5 months and just stopped playing tennis,
which type of TENS

would you use:

a- Conventional TENS

b- Acupuncture like TENS c- Brief intense TENS

d- Burst TENS √

259- When treating chronic pain which type of TENS used :

a- Brief intense TENS b- Conventional TENS

c- Burst TENS √

d- Acupuncture like TENS

260- P.t has chronic brachialgia best tense use

a- convential

b- low free

c- burst √ sure burst tense for chronic pain


d- intense
261- During evaluation of a hemiplegic patient you found that there is balance
deficiency, the cause is

a- weakness of glutei

b- weakness of quadriceps

c- spasticity of planter flexors √

d- weakness of dorsi flexors

262- patient complaining from back pain, with examination you find decreased
sensation over big

toe what level of lesion would you expect

a- l 3 - l4

b- l4 – l5 √

c- l5 – S1

263- A therapist receives an order to evaluate and treat a 76-year-old woman


who was involved in a motor vehicle accident 2 days ago. The patient’s vehicle
was struck in the rear by another vehicle. The patient has normal sensation and
strength in bilateral lower extremities but paralysis and loss of sensation in
bilateral upper extremities. Bowel and bladder function are normal. The patient
most likely has what type of spinal cord injury?

a- Anterior cord syndrome.

b- Brown-Sequard syndrome.

c- Central cord syndrome. √

d- There is no evidence of an incomplete spinal cord lesion.

Central Cord Syndrome: is when the damage is in the centre of the spinal
cord. This typically results in the loss of function in the arms, but some leg
function may be preserved. There may also be some control over the bowel
and bladder. It is possible for some recovery from this type of injury, usually
in the legs, gradually progressing upwards

.
264- anterior taleofibular ligament to be assessed resist

a- planter flex+inv √

b- dorsi+inv

c- dorsi+eversion

d- plant+ev

265- during ex theres subtalar pronation that’s mean

a- femur,tibia,pelvic internal rotation

b- femur,tibia,pelvic ext rot

c- femur ,tibia int rot with pelvic ext rot √

d- femur ,tibia internal rot with pelvic int rot

subtalar pronation =eversion

266- Which of the following choices is TRUE about Mitral Valve:

a- It connect the left atrium with the left ventricle and it is a tricuspid valve

b- It connect the left atrium with the left ventricle and it is a bicuspid valve √

c- It connect the left atrium with the left ventricle and it is a semi lunar valve

d- It connect the right atrium with the right ventricle and it is a bicuspid valve

267- the heart valve responsible in preventing blood from return to right atrium
from right ventricle is :

a- mitral or bicuspid

b- tricuspid √

c- pulmonary
The heart consists of four chambers, two atria (upper chambers) and two ventricles
(lower chambers). There is a valve through which blood passes before leaving each
chamber of the heart. The valves prevent the backward flow of blood. These valves
are actual flaps that are located on each end of the two ventricles (lower chambers
of the heart). They act as one-way inlets of blood on one side of a ventricle and
one-way outlets of blood on the other side of a ventricle. Normal valves have three
flaps, except the mitral valve, which has two flaps. The four heart valves include
the following:

tricuspid valve: located between the right atrium and the right ventricle

pulmonary valve: located between the right ventricle and the pulmonary artery

mitral valve: located between the left atrium and the left ventricle

aortic valve: located between the left ventricle and the aorta

268- which statement not correct about fracture neck of femur ‫ ر‬# $‫آ‬

a- its fatal

b- need arthoplasty

c- lead to avascular necrosis

d- occur mostly in children √

Most occur in eldery

269- 25 years old patient with acute spinal cord injury admitted to the hospital
then referred to you while taking history there is DVT which of the following is
not allowed to be done

a- elevation of the head of the plinth for good drainage √ sure

b- passive movement exercise c- wear compressing socks

270- convalescent stage of polio :

a- 2weeks-2years √

b- 2weeks-4years

c- 0-2week
d- 2week to through life

271- normal action potential

a- 30 to – 60 mVolt

b- 60 to – 90 mVolt

c- 30 to – 90 √ sure

d- 20 to – 80

272- Below knee amputation what do YOU see?

a- there is knee flexion

b- there is no knee flexion

c- there is edematous stump √

d- there is deformed stump

273- patient suffering from morning stiffness this is a sign of :

a- swelling of the body tissues

b- prolonged stiffness with inflammation. √

c- muscle inflammation.

d- bonny projection at the joint


274- Patient is suffering from morning stiffness; the most likely developing
condition is due to:

1-Bones are growing beyond their margins √

2- Fasciitis (efers to an inflammation of the fascia )

3- Accumulation of lactic acid

4- Muscle Spasm

275- person fall on lat aspect of shoulder last thing occur is

a- dislocation sc joint √

b- dislocate acromio clavicular joint

c- fracture medial clavicle

d- fracture middle third of clavicle

276- sciatic can be injured

a- positerior hip dislocation √

b- anterior hip

c- dislocation

d- neck of femur-ACL tear


277- in early bronchitis there is:

a- dry cough √ Acute bronchitis

b- wheezing sound

c- peripheral cyanosis

d- cough with expectoration Chronic bronchitis

278- The bronchitis is:

a- dry cough

b- wheezing sound

c- peripheral cyanosis

d- cough with expectoration √

279- in chronic bronchitis

a- dry cough

b- productive cough √

c- frothy sputum

280- chronic bronchitis symptoms

a- nocturnaldysnea

b- frothy sputum

c- bronchospasm and productive cough √

281- infant with erbs palsy he can full recover with good biceps & deltoid at:

a- 3 months

b- 6 months

c- 9 months

d- 12 months √ not sure


282- for vertebrobasilar insufficiency all of the following is contraindicated
except

a- cervical manipulation

b- high grade mobilization

c- passive side bending √

d- rotation of the neck

283- burn in the body's response to thermal insult from external agent such as :

a- heat and cold

b- chemicals

c- electricity and radiation

d- all of the above √

284- middle cerebral artery which not affected

a- shoulder

b- upper limb √

c- lower limb

d- wrist

285- C.O.G during locomotion observation can change the gait due to

a- c.o.g go upward and oblique to stance phase

b- downward during double limb support

d- move forward and backward √


286- ex which occur in round and consist of continuous training

a- open kinematic chain

b- close kinematic chain

c- circuit training √

d- isometric

287- which is correct about volkmans ischemic contracture

a- affect flexor of forearm √

b- affect palmar fascia lead to ulnar neuritis

288- One of the possible complications following a fracture is Volkmann's


ischemic contracture. This condition:

a- Is caused by an intelference with the venous return

b- Is caused by an interference of the nerve supply

c- May occur if the fracture is sustained in the upper extremity

d- None of the above √

‫ن‬$ =< *9 ? ‫ ر ا‬$< ‫ > ا‬M"‫ > ' د‬2 ‫م و‬1 ‫ ن ا‬0 9 > M&)F LI= ‫ن ا‬$ =< ‫ ر ا ول‬$< ‫ا‬
=) ‫ م ا‬:D 7I" ! LI= ‫ن ا‬$ =< @ ? ‫ > ا‬L M/ ' M 2 LI= ‫ا‬

289- Burns caused due to constant direct currents are called

a- Galvanic burns

b- Chemical burns

c- Electrolytic burns

d- All of the above √

290- pt ask to go upstairs with crutch ‫ ر‬# $‫آ‬

a- Sound limb go up 1st √

b- Affected limb go up 1st


291- which statement incorrect during training with cane up and down stairs
$‫ ر آ‬#

a- unaffected leg put first during upstairs

b- unaffected leg put down during up stairs

c- affected leg put first in down stairs

d- affected and cane becomes after unaffected leg during downstairs √

292- As a result of a gait analysis, a therapist has determined that the patient
ambulates with excessive foot pronation. This deviation would not occur as a
result of:

a- compensated forefoot varus

b- internal tibial rotation

c- a weak tibialis posterior

d- excessive ankle dorsiflexion √


293- You wish to mobilize a patient’s shoulder using an inferior glide
technique. It would be best to use this technique be propositioning the patient’s
arm in:

a- 95 degrees of abduction with lateral rotation

b- 125 degrees of abduction and internal rotation √ sure sure sure

c- 55 degrees of abduction and neutral rotation d- 95 degrees of shoulder


flexion and neutral rotation

294- ms respnsible of climbing stairs and coming from sitting to standing

a- gltus max

b- illio psoas

c- Quadrecips √

d- hamstrings

295- best muscle to strengthen for ascending stair and initiating stand up from
sitting is

a- Gluteus maximums

b- Quadriceps √

c- Hamstring

d- Soleus

296-which movement not accompany with hyper extension of cervical region

a- poking chin

b- decrease flexion of lower cervical region √

c- Increase activity of SCM AND levator scapulae

297- Arterial blood pressure means ‫ ر‬# $‫آ‬

a- difference between systole and dystole

b- lat pressure exerted on arterial pressure √

c- max blood comes from ventricles at end of diastole


d- max blood comes from ventricles at begining of systole

298- Arterial Blood Pressure is defined as:

a- Is the pressure exerted by circulating blood upon the walls of arteries √

b- Is the pressure exerted by circulating blood upon the walls of veins

c- The high point in blood pressure within an artery

d- The pressure of the blood in the arteries during ventricular diastole

299- Absolute refractory period in cardiac ms

a- slowler than skeletal ms

b- contain systole and beginning of dystole √

c- contain diastol and beginning of systole

300- Cardiac output means ?

a- Blood pumped from ventricle / min √

b- Blood pumped from ventricle /beat

c- Blood still in ventricle during systole

d- Blood still in ventricle during diastole

301- What the true about apex of heart :

a- Is downwards of the left side √

b- Is downwards of the right side

c- 3rd degree of intercostals ribs in line of sternum

d- 5th degree of ribs

302- While evaluating the gait of a patient with left hemiplegia, you note toe
drag during mid swing on the left. The least likely cause of this deviation
would be:

a- inadequate concentric activity of the ankle dorsiflexors

b- excessive extensor synergy


c- knee and ankle joint pain √

d- decreased proprioception

303- A patient with adhesive capsulitis of the glenohumeral joint should


demonstrate the greatest limitation of motion when performing shoulder:

a- flexion

b- abduction

c- medial rotation

d- lateral rotation √

304- PT is assessing a patient’s ability to perform basic activities of daily


living. The assessment tool chose by PT measures bathing, toileting, dressing,
transfers, continence and feeding. The tool does not assess the patient’s ability
to maneuver in a wheel chair The therapist is using which of the following
tests?

a- Barthel Index

b- Katz Index of Activities of Daily Living √

c- Kenny Self-Care Evaluation

d- Functional Status Index

The Katz Index ranks adequacy of performance in the following six functions:
bathing, dressing, toileting, transferring, continence, and feeding

305- Which of the following increase lumber lordosis

a- shorting in iliopsos ms √

b- shorting in one iliotibial band

c- shorting in hamstring
306- with disk herniation with decreas back motion assisst :

a- sternocostal angle

b- lumboscaral angle √

c- cobbs angle

d- cubitis angle
307- Ratchety or tremors In parkinsonism pt :

a- cogwheel rigidty √

b- lead pipe rigidty

c- spastcity

308- when evaluating Parkinson’s pt, you observed uniform muscle resistance,
this means :

a- cogwheel rigidity

b- leadpipe rigiditiy √

c- spasticity
309- Acute Inflammatory Demyelinating Polyradiculoneuropathy is also
referred as

a- Guillain–Barré syndrome √

b- Compressive myelopathy

c- Friedreich's ataxia

d- None of the above

310-Immediately after switch

TENS there sever numbness and exaggeration of symptom your response

a- switch off the machine √

b- increase wave length

c- decrease frequency

d- decrease pulse time

311- Pt has anginal pain during walking on traid mail u ask for cardilogest why

a- the pain is sever strapping pain

b- the pain was in chest and radiated to left shoulder √

c- the pain radiated to rt arm

d- the pain radiated to back


312- As a result of a gait analysis, a therapist has determined that the patient
ambulates with excessive foot pronation. This deviation would not occur as a
result of:

a- compensated forefoot varus

b- internal tibial rotation

c- a weak tibialis posterior

d- excessive ankle dorsiflexion √

313- knee R O M

a- flex 130 ext 0

b- flex 140 ext 0 √

c- flex 110 ext 10

Hip ROM 10 extension 125 flexion

314- At any age the child can walk independably

a- 12 month √ as in book of tidy ( from 11 to 15 month )

b- 18 month

c- 24 month

d- 30 month

315- At any age the child can jumb in one leg holding on

a- 18 month

b- 24 month √ not sure

c- 30 month
316- You are evaluating a 48 year-old tennis player with a lower extremity
problem. You would use the Thompson test to assess for:

a- anterolateral rotational instability of the knee

b- iliopsoas tightness

c rectus femoris tightness

d- Achilles tendon rupture √

317- THOMPSON test for the ?

a- shortening of illiotipial band

b- integrity test of achillis tendon √

c- shortening of illiopsoas

d- integerity test of quadriceps tendon

318- A 13 year-old girl has a structure right thoracic idiopathic scoliosis. The
clinical features you

would expect to find include:

a- a high right shoulder,a prominent right scapula and a left hip that protrudes √

b- a high left shoulder, a prominent left scapula and a right hip that protrudes

c- a high right shoulder, a prominent left scapula and a right hip that protrudes

d- a high left shoulder, a prominent right scapula and a left hip that protrudes

319-patient has scoliosis c on RT thoracolumbar we found: 0!) * ‫ ول‬A ‫ا‬

a- lat flexion of thoracolumbar on Rt convex

b- lat flexion and Lt convex

c- lat flex and concave on Lt side √


320- Rt C Scoliosis we find?

a- Lateral trunk flexion & concave side to RT

b- Lateral trunk flexion &convex side to RT √

c- Lateral trunk flexion &convex side to LT

d- Lateral trunk flexion & shorten RT musculature

321- The best position for artificial hip anklosis

a- flex 20,abduction 10,extrnal R 5 √

b- no flex ,abduction 10,extrnal R 5

c- abduction 5,extranal R 10

Arthrodesis, also known as artificial ankylosis

Surgical Considerations:

- position of hip fusion:

- neutral abduction, external rotation of 0-30 deg &, 20-25 deg of flexion;

- avoid abduction and internal rotation;

- this position is design to minimize excessive lumbar spine motion

322- prevention of transmission of pain beyond the dorsal horn ‫) ر‬

a-peripheral n injury

b- gate control theory √

c- spinal cord injury

d- opiate theory
323- A 50-year-old man has a persistent cough, purulent sputum, abnormal
dilation of bronchi, more frequent involvement of the left lower lobe than the
right, hemoptysis, and reduced forced vital capacity. What is the most likely
pulmonary dysfunction:

a- Chronic bronchitis

b- Emphysema

c- Asthma

d- Bronchiectasis √

324- all of these are equilibrium-coordination tests except

a- finger to finger √

b- jumping

c- grasp

d- jogging

Non equilibrium – coordination tests as in book of physical rehabilitation

Finger to finger

Finger to nose

Heel to knee

325- physiotherapist asks pt to flex the wrist and abduct it ; which muscle acts
in this action ?

a- flexor carpi ulnaris

b- flexor carpi radialis √

c- flexor digitorum profundus

d- biceps brachii
326- Therapist performed trendlinburg test for pateint with (Rt) weakness
gluteus medius , when test is (+ ve) when patient perform single limb support
on RT lower limb , what is correct of the following about pelvis drop:

a- pelvis dosent drop as the patient lean by trunk to RT

b- pelvis will drop at left side √

c- pelvis will drop at Right side

d- pelvis dosent drop as the patient lean by trunk to LT

327- Parasympathetic action of heart

a- decrease heart rate from 120 ppm to 70 ppm √

b- increase in female more than male

c- decrease in adults more than child

d- decrease in athelets more than athelets

328- Aim of cardiac parasympathetic

a- Inhibiting all cardiac

b- decrease ventricle contractility √

c- stimulation all cardiac

Parasympathetic action of hear decreases (heart rate, force of contraction, and


blood pressure)
329- congenital dislocation occure in :

a- hip √

b- knee

c- shoulder

d- elbow

330- Abduction to Adduction in which plan occurs :

a- From segital plan to coronal plan √

b- From coronal plan to segital plan

c- From longitudinal to transverse


331- all the following is clinical feature of fracture of neck of femur except?

a- pain in hip region

b- abduction of hip √

c- shortness of limb

d- lateral rotation of limb

332- electrotherapy has physiological effect of heat that not dependant on


which of the following

a- type of machine

b- size of area

c- duration and Frequancy of application

d- way of application √

333- When evaluating inspiration capacity using spirometer

a- Maximum inspiration after normal expiration √

b- Maximum inspiration after maximum expiration

c- Normal inspiration after normal expiration

334- tibia # and fixed by plate and escrow .what modality is contra indicted

a- faradic

b- ultra Sonic

c- short wave √
d- ice application

335- Rheumatoid arthritis

a- cold limb

b- radial deviation

c- pain ,wet ,swelling √

336- The first thing affected in Rheumatoid arthritis :

a- capsule √

b- articular cartilage

337- Type of laser used in PT ttt

a- High

b- Soft √ not sure

c- Mid

338- posterior shear test for

a- sacroilliac joint √

b- hip joint

c- lumbosacral joint
339- Parkinson’s gait, which is not true

a- Propulsion gait

b- Festinating gait

c- Retropulsion gait

d- Wide base of support with arm swing √

340- parkinsonism patient has complain of falling and decreased balance what
will you do

a- lowering COG √

b- decrease BOS

c- traction

d- passive movement

341- Difficult speech is called

a- Dysarthria √

b- Dysphagia

342- best treatment of supraspinatus tendinitis is

a- infrared

b- shortwave

c- ultra sound with deep friction massage √

343- close pack position of knee is

a- flexion 20 degrees

b- extension

c- extension with lateral rotation √

344- which of the following is not included in shoulder complex

a- clavicle

b- scapula
c- humerus

d- sternum √

345- which of the following you cannot palpate while examining shoulder ‫ ر‬#
$‫آ‬

a- 2nd rib

b- Sternoclavicular joint

c- 1st rib √

346- during examination of shoulder, you found increased motion (laxity) in


anterior, posterior and inferior directions, the condition is

a- Global shoulder instability √

b- Rotator cuff tear

347- An athlete with stable fracture of neck of humerus, referred to you to


improve mobility,

which form of exercise will you choose

a- Isometric

b- Resistive exercise in all directions √

348- Motor neuron disease, which of the following is not true

a- It may affect anterior horn cells and cause lower motor neuron weakness

b- It may affect cranial nerve nuclei and cause upper motor neuron weakness √
349- spastic diplegia means

a- primarily affecting the legs but arms are less involved and less severe √

b- only legs are involved

c- only arms are involved

d- one limb is involved

350- During evaluation of a hemiplegic patient you found that there is balance
deficiency, the cause is

a- weakness of glutei

b- weakness of quadriceps

c- spasticity of planter flexors √

d- weakness of dorsi flexors

351- when examine patient for carpal tunnel syndrome, which nerve do you
examine

a- Radial n.

b- Ulnar n.

c- Median n. √

352- patient with angina pectoris, pain distribution

a- apex of heart

b- chest only

c- chest with left shoulder and arm √

353- old patient with productive cough, fever and pulmonary congestion, The
diagnosis is:

a- Heart failure

b- Pneumonia √

c- Pulmonary effusion

d- Cystic fibrosis
THERE IS FEVER, Cystic fibrosis most significant sign is Salty-tastingskin and it is
an inherited (genetic) disease. Pulmonary effusion pleuraleffusions are usually
caused by underlying medical conditions, symptoms of these conditions are also
often present such as Congestive heart failure. Pneumonia

354- A 65 year old patient comes with fever productive

cough and pulmonary congestion the diagnosis is

a- cystic fibrosis

b- pulmonary oedema

c- heart disease

d- bacterial and viral pneumonia √

355- Sudeck’s atrophy which is not true :

a- Sever burning pain

b- Sympathetic hyperactivity √

c- Trophic changes occur

d- More perspiration

356- SWD is contraindicated in

a- Skin infection

b- Peripheral vascular disease √

c- haematoma

357- Intra medullary nail contraindication use ◊ Swd

358- deformity associated with coll’s fracture ( fracture lower end of radius)

a- Dinner fork deformity √

b- Pes planus

c- Talipus equinovarus
359- Coll's fracture may cause late

a- flexor pollicis

b- abd pollicis

c- ext policis longus √

d- add policies

360- muscle that moves eye brows medial and inferior and make vertical
wrinkles?

a- Currigator √

b- Frontalis

361- which ms draw air brows together to downward and inward making
horizontal wrinking:

a- pectineus

b- corrugator √

c- orbicularis oris

d- lateral ptyroid

362- which muscle Raise the skin of the chin.:

a- buccinators

b- levator libii ◊ Elevates the upper lip

c- mentalis ◊ Elevates and wrinkles skin of chin, protrudes lower lip

d- platysma ◊ Draws the corners of the mouth inferiorly and widens it


363- which of the following Closes the jaw:

a- masseter √

b- levator lipii

c- mentalis

d- orbicularis oris

364- which of following muscle doesn't closes the mouth:

a- masseter

b- ptrygoidus medialis

c- temporalis

d- suprahoid ms √

365- which of the following Closes the mouth:

a- masseter √ not sure

b- levator lipii

c- mentalis

d- orbicularis oris √

366- which muscle Close and protrude the lips, as in whistling:

a- masseter

b- mentalis

c- orbicularis oris √

d- depressor libii

367- which of following ms compress cheecks ‫ ر‬# $‫ آ‬:

a- buccinators √

b- platysma

c- orbicularis occuli
d- nasalis alar portion

368- muscle that moves angle of mouth inferior and back

a- Platysma √

b- Mentalis

c- Masseter

369- muscle that close the jaw

a- Masseter √

b- Mentalis

c- Buccinators

370- when stimulate facial ms at bells palsy which ms not contracted

a- frontalis

b- masseter √

c- orbicularis oris

d- levator labii

371- Patient with facial palsy in LT side he cannot make horizontal Lt lateral
side by eye which ms affected

a- LT latralies ◊ Rectuce medialis pull eye toward nose - Oculomotor nerve

b- RT latralies ◊ Rectus lateralis pull eye away from mid line - Abducens nerve

c- Rt mediales ◊ Rectuce medialis pull eye toward nose - Oculomotor nerve

d- a and c both right answers √


372- one of the following is not an aerobic exercise:

a- weight lifting √

b- jogging

c- running a marathon

d- runnin1000 meters

373- one of the following dose not use phosphotognase of the body:

a- weight lifting √

b- jogging

c- running a marathon

d- diving

374- one of the following is not depends on oxygen of the body:

a- weight lifting √

b- jogging

c- running a marathon

d- diving
375- one of the following is not depends on oxygen of the body:

a- aerobic

b- anaerobic √

376- Which not depend on adinosin triphosphate

a- aerobic

b- anaerobic √

377- physiological effects of warming up ‫ ر‬# $‫آ‬

a- Decrease oxygen deficiency √ sure in book of physical rehabilitation

b- Increase myocardial demands

378- All the following is true about ligaments except : ‫ ر‬# $‫آ‬

a- high collagen content

b- laxity lead to hyper mobility

c- connect on bone to another

d- usually heals fast and need not support √

379- which muscle act in Medial rotation, adduction and extension of shoulder
joint:

a- pectoralis minor

b- serratus anterior

d- latissmus dorsi √

380- muscle of swimming

a- pectolalis major

b- serratus anterior

c- latissmus dorsi √
381- which is synovial joint

a- Temporomandibular joint √

b- Symphisis pubis

382- Which is true about SA node:

a- It’s type of myocytes with higher rhythm √ because higher frequency than av node

not sure it's my opinion

b- type of myocytes which generate electrical impulse and control heart rate

c- consist of nerve cell

d- in AV node

383- S.A.NODE

a- its part of myocarduim and consist of high rhythem √

b- its in A.V. node

c- consist of nerve cell

d- its part of myocardim contractility tissue

384- impulses reach atrium early to contract before ventricles through :

a- Purkinje fibers

b- AV node √

c- Bundle of His

d- Atrioventicular bundle

385- Location of SA node

a- top of Rt atrium

b- Top of Lt atrium

c- Inter ventricular septum


386- pacemaker is

a- locating in AV node

b- contain connective tissues

c- contain nerve tissues

d- in SA node √

387- When you examine shoulder joint by asking patient to abduct shoulder to
90 degrees then lower slowly, this is:

a- Codman’s Test √

b- Infraspinatus test

388- pt with sever spinal deformity 32 years old has difficulty in respiration is
due to :

a- ms weakness

b- usage of spinal orthosis

c- decrease lung capacity √

389- knee joint ROM:

a- flexion 110 – extension 0

b- flexion 130 – extension 0

c- flexion 140 – extension 0 √ sure


390- when use iontophoresis ; it is a type of :

a- low frequency TENS

b- HVGS

c- direct current √

d- Russian current

391- Iontophoresis uses which type of current?

a- dyadinamic

b- direct current √

c- ultrasonic

d- tens

392- In yellow and red flags which of the following not considered as red flag
in these situations :

a- history of carcinoma

b- psychosocial factor √

c- bowel problems

d- excess weight loss

Flags

'Yellow flags' are psychosocial factors including a previous history of anxiety and
depression, impending compensation, absence from work, sickness benefit,
invalidit benefit, passivity and high levels of dependency and poor coping skills.

'Red flags' are clinical features that should alert the therapist to the possibility of
severe pathology. The include bladder and bowel malfunction, saddle anaesthesia,
bilateral paraesthesia, neurological signs, unexplained weight loss, a past history
of carcino ma, general debility and fever.
393- Which of the followin fractures treated by skeletal traction?

a- knee

b- patellofemoral

c- shaft of femur √

d- tibia

394- coll`s fracture may possibly cause injury to which of the following?

a- flexor pollicis

b- abductor pollicis

c- adduuctor pollicis

d- extensor pollicis longus √

395- Splenius capitis response of :

a- neck flexion

b- neck extension √

c- neck rotation

d- neck side bend

396- which of following not needed in testing the splenius capitis ms?

a- head flexion √

b- head extension

c- head rotation

d- head lateral flexion

397- which of following not needed in testing the trapezius ms?

a- head flexion √

b- head extension

c- head rotation
d- head lateral flexion

398- which of following not needed in testing the scalene ms

a- head flexion

b- head extension √

c- head rotation

d- head lateral flexion

399- Which of the following is a possible cause for anterior pelvic tilting :

a- weak lateral trunk ms

b- tight tensor facia lata

c- tight hamstring

d- sever weakness of abdominal muscles √

400- In contracture of lower lumbar back ms it affect pelvis by : ‫ ر‬# $‫آ‬

a- anterior pelvic tilting with upper thorcic kyphosis

b- anterior pelvic tilting with lordosis √

c- posterio pelvic tiltig with lordosis

d- no effect on the pelvis


Pic explain causes of lordosis and so ant.pelvic tilt

401- Patient has contracture lower back that make

a- ant tilt of the pelvis + lordosis √

b- ant tilt of the pelvis + kyphosis

c- pos tilt of the pelvis + lordosis

d- no lumbar spin change

402- Anterior pelvic tilting is causing exaggerated lumbar lordosis due to :

a- adductor tightness

b- tightness of tensor facia lata

c- bilateral fixed hip flexor contracture √

d- tightness in hamstring ms

403- hyperextension of the hips, an anterior pelvic tilt and anterior


displacement of the pelvic

a- Flat back

b- Sway back √ sure as in tidy

c- lordosis

d- thoracic kyphosis

404- one joint hip flexor contracture result in

a- no effect on lumber

c- anterior pelvic tilting √

d- posterior pelvic tilting

405- hyperextension of the hips, an anterior pelvic tilt and anterior


displacement of the pelvic

a- Flat back with Post pelvic telt

b- Sway back √ Hyperlordosis


c- lordosis M& A‫ال ا‬P= ‫آ ش > ا‬ M $‫) ن ا‬D‫ ه‬. ‫آ‬

d- thoracic kyphosis

406- pt has post pelvic tiled , flat lower thoracic & increase upper back
kyphosis suffer of :

a- Flat back √

b- Sway back Saddle back

407- Bennett's fracture is ?

a- little finger

b- thumb √

c- middle finger

d- distal end of radius


408- Type of ex in which entire muscle is lengthening against external force

a- eccentric √

b- concentric

c- closed chain ex

d- Open chain ex

409- In degenerative joint disease which is not occur?

a- increased with weight bearing on the joint

b- gradual onset √

c- stiffness at morning

d- increaser after prolonged period of inactivity

Characteristics of degenerative joint disease

Signs and symptoms may include:

• pain that increases on weight-bearing activities (standing and walking, walking


downstairs

particularly)

• insidious onset of symptoms followed by progressive periods of relapses and


remissions

• pain and stiffness in the morning

• stiffness following periods of inactivity

• pain and stiffness that arise after unaccustomed periods of activity

• bony deformity (e.g. characteristic varus deformity may follow from collapse of
the medial compartmental joint space)

• reduction of the joint space observed on X-ray, with bony outgrowths or


osteophytes.
410- in ttt of parkinsonion’s pt :

a- strengthening of elongated muscles and stretching of rigid flexors √

b- strengthening of elongated muscles and stretching of rigid extensors

c- strengthening of flexors & extensors

d- none of the the above

Strengthening exercises should emphasise the extensor or antigravity muscles as


these become most weakened.

Flexibility exercises should focus on rotation (neck, trunk, hips and shoulders)

Stretching exercises the flexor muscles (e.g. the hip flexors - front of hip,
hamstrings - back of knee and calf muscles at the back of the leg) , as these tend to
become tight.

411- Which of the following is true about anterior cerebella artery:

a- Effect in lower extremity in same side

b- Effect in upper extremity in same side

c- Effect in lower extremity in opposite side √

d- Effect in upper extremity in opposite side

412- Patient with C.V.A damage LT ant.cerebral artery affect :

a- ipsilateral leg

b- contra lateral leg √

c- ipsilateral arm

413- patient has C V A and middle cerebral artery affected which part will
have the best complete recovery ‫ ر‬# $‫آ‬

a- shoulder

b- elbow

c- hand

d- hip √
414-patient with C.V.A damage LT ANTERIOR CEREBRAL ARTERY
AFFECT

a- contralateral leg √

b- ipsilateral leg

c- ipsilateral arm

Middle cerebral artery:

(supplies most of the convexity of the cerebral hemispheres)

Dense contralateral hemiplegia, Contralateral homonymous hemianopia, Cortical


type of sensory loss, Speech problems in left hemisphere lesions, with neglect of
contralateral side Lesions in the right hemisphere result in parietal damage,
visuospatial disturbances and left sided neglect

Posterior cerebral artery:

Visual disturbance, Contralateral homonymous field defect, Memory disturbance


and contralateral sensory loss

Anterior cerebral artery:

Contralateral monoplegia, Cortical sensory loss, Sometimes behavioural


abnormalities associated with frontal lobe damage

415- Which ms of body flexes hip and abducting it and (Laterally) rotates it?

a- sartorius √

b- tensor facia lata

c- illiopsoas

d- rectus femoris

416- Which ms of body flexes hip and abducting it and (Medially) rotates it?

a- sartorius

b- tensor facia lata √

c- illiopsoas
d- rectus femoris

417- In clinical examination you must do all of the following except:

a- observe the patient examine both affected unaffected

b- review patient sheet

c- focus only on areas of patient's complain √

d- respect the sesitizers of patients

418- Osteomyelities is :

a- infective inflammation of the bone due infective bacteria entered into bone √

b- decreased bone density and increased porosity

c- vit d defficiency

d- due to repetitive trauma

419- The arterioles characterized with?

a- large thick walls supplying organs

b- a networks of small vessels have thin walls

c- a networks of small vessels have thick walls √

420- The main muscle resposiple for climbing stairs and coming from reclined
position bringing knee to chest?

a- rectus femoris

b- illiopsoas √

c- quadratus lumborum

d- pectinieus
421- If the proximal bone of joint fixed the distal part of joint move in
sequence so this is ?

a- ACL Training

b- closed chain ex → ( disital bone of joint fixed the proximal part of joint
move )

c- open chain ex √

d- non of the above

422- modality can increase the temperature of fat?

a- ultraviolet

b- ultrasonic √

c- S.W.D

d- FES

423- Which of the following muscles is weak and the patient can't supinate
forearm to open the door using it

a- brachialis ◊ Flexion at elbow joint n. musculocutaneous nerve (C5, C6)

b- biceps brachii √ →Flexes elbow and supinates forearm n. musculocutaneous


nerve (C5, C6)

c- brachioradialis →Flexion of Elbow and pronates forearm n. radial nerve

d- coracobrachialis →Flex and medially rotate the arm n. Musculocutaneous


nerve (C5, C6,C7)

424- physical therapist is evaluating elbow flexion from neutral position ;


which muscle should be evaluated ?

a- biceps brachii →flex with supination

b- brachialis →flex with pronation or supination – main flexor elbow

c- brachioradialis →√ → flex from neutral

d- anconeous
424- PT evaluate patient who is unable to open the door using supination which
ms. should be expected for weakness :

a- Biceps brachii √ →Flexes elbow and supinates forearm n. musculocutaneous


nerve

b- Anconeus →Assist in ext. & stabilize the elbow during pronation and
supination

c- Brachioradialis →Flexion of Elbow and pronates forearm n. radial nerve

d- Flexor carpi ulnaris →Flexion and adduction of the wrist

425- PT evaluate patient who is unable to open the door using supination which
ms. should be excepted for weakness :

a- Biceps brachii →Flexes elbow and supinates forearm n. musculocutaneous


nerve

b- Anconeus →Assist in ext. & stabilize the elbow during pronation and
supination

c- Brachioradialis →Flexion of Elbow and pronates forearm n. radial nerve

d- Flexor carpi ulnaris √ →Flexion and adduction of the wrist

426- What muscle make lateral rotation of Humeral head :

a- Pectoral major →adducts and medially rotates the humerus

b- Tears minor √ →laterally rotates the arm, stabilizes humerus

c- Tears major m→edial rotation of the humerus

d- Rotator cuff

427- In which location the median nerve freqently or mostly entrapped?

a- forearm

b- arm

c- elbow

d- wrist √
428- A THERAPIST performin test for patient and test was (+ ve) the thigh of
the patient rased some inches above examination table what is the test name?
what is the shortened muscle?

a- ober test →for ilio tibial band

b- sraight leg raising test f→or hamstring

c- thomas √ →for illiopsoas

d- thomson f→or gastrocnemeius

429- baby supine lying abducting both shoulders with 90 elbow flexion lower
limbs extended and adducted at hips and extended at knees and planter flexed
ankles , what is this reflex?

a- Tonic labyrinthine reflex

b- Moro reflex

c- STNR √

d- ATNR

430- You have evaluated anine month old who cann't assume or maintain
quadriped position without assistance his parent innsest that child has already
begn to walk with assistive you susbect that what parents say is

a- protective ext dowenward

b- spontinous stepping √

c- positive supportin reaction

d- negative supporting reaction

431- What the most primitive reflex of human body.

a- Symmetrical tonic neck reflex

b- sucking reflex √

c- asymmetrical tonic reflex

d- tonic lybrinthine
432- In pulmonary edema there is?

a- lower limb ischemia

b- lower limb edema and frothy secretions √

c- thin clear sputum

433- which statement incorrect about brown sequard syndrome ‫ ر‬# $‫ آ‬:

a- ipsilaterl babinski sign

b- ipsilateral loss of pain and temp √

c- contralateral loss of pain and temp

Contralateral loss of pain and temperature sensation,, Ipsilateral spastic paralysis


below the level of the lesion, ipsilateral loss of Babinski sign

434- A patient presents with a hemisection of the spinal cord (Brown-Sequard)


at the T12 level. When examining the patient's right lower extremity, which of
the following would you likely find? ‫ ر‬# $‫آ‬

a- Muscle paralysis

b- Loss of position sense

c- Loss of pain sensation √

d- Loss of vibratory sense

435- the number of the joints & bones on foot

a- 23 joints & 25 bones

b- 33 joints & 26 bones √

c- 23 joints & 17 bones

436- Hyaline cartilage innervated by

a- blood vessels

b- synovial fluid √

c- epiphyseal growth
hyaline cartilage

A type of cartilage found on many joint surfaces, it contains no nerves or blood


vessels. Cartilage is not innervated and therefore relies on diffusion to obtain
nutrients. This causes it to heal very slowly. It has high elasticity and helps cushion
and protect bones.Hyaline cartilage is covered externally by a fibrous membrane,
called the perichondrium, except at the articular ends of bones and also where it is
found directly under the skin

437- When u pushing automobile with shoulder flexion ,elbow extended the
main muscle of action is

a- serratus anterior √

b- biceps brachial

c- pectorals major

d- triceps brachials

438- pt is refferd to physio dept wih diagnosis of flexion deformity of rt knee


with examination we will found ‫ ر‬# $‫آ‬

a- flexion knee +planter flexion +shortening of qudricepes

b- flexion knee +dorsi flexion +shortening of hamstring √

c- flexion knee +no ankle changes +shortening of quadriceps

d- flexion knee +planter flexion +lenghing of soles

‫ * ا )(! ا‬5D C B‫* ! آ‬+> ‫ن‬0# .),+ 9 =< : ‫ ر ا ول‬$< ‫ا‬

‫ ر ا ول‬$< ‫* ا‬ B ‫ ا‬E,+ =< : @ ? ‫ ر ا‬$< ‫ا‬

‫ * ا م‬5D GD0 ‫ ا‬E,9 * ‫ و‬4 # ‫* ا‬ ;# ! $9F ‫ن‬0# .),+ 9 =< : C ‫ ر ا ا‬$< ‫ا‬

439- Facial nerve nuclei innervations :

a- Bilateral pyramidal innervations √

b- Unilateral pyramidal innervations

c- Bilateral no pyramidal innervations


facial nuclei are divided into two halves upper half receive bilateral pyramidal tract
supply from both sides but lower half receive unilateral pyramidal tract supply from
the opposite side only

440- coupling media of ultrasonic which of the following ‫ ر‬# $‫ آ‬:

a- aquatic gel √

b- glycerol

c- liquid paraffin

d- crrogel

441- coupling media of ultrasonic which of the following of least effect: ‫ ر‬#
$‫آ‬

a- aquatic gel

b- glycerol

c- liquid paraffin √

d- crrogel

442- pt with neurological condition with evaluation found is unable to perform


movement after many repetition with adequate rest period comes normal this pt
is

a- mythenia graves √

b- myopathy

c- hyprertonia

443- unable to perform movement from first time without repetion

a- mythenia graves

b- myopathy √
444- The therapist treatment patient by phototherapy what this modality:

a- Low Laser therapy √

b- Fluid Therapy

c- Ultrasound

d- TENS

445- Pt has 32 years with spinal deformity & dyspnea due to :

a- Muscle weakness

b- Spinal orthosis

c- Decrease lung volume √

446- heart at rest supply ms?

a- 2 ml\min 100g ms

b- 3 ml\min 100g ms

c- 4 ml\min 100g ms √

447- which is CORRECT ABOUT patellar tendon bearing orthosis

a- increase loading of proximal leg √

b- suspect decrease 50% load

c- unload of hip joint

d- used with elbow assistive device

448-Patellar tendon loading-orthosis which isn’t correct?

a- Carry proximal leg load

b- Carry 50% of load

c- Used with assistive elbow cruches

d- Decrese load in hip joint √


449- Patellar tendon bearing POP cast is indicated in the following

FRACTURE :

a- Patella

b- Tibia √

c- Medial malleolus

d- Femur

450- pt with new SCI referred to u for prevention of bed rest complication
which not included in your program

a- passive movement

b- elastic bandaging for prevention of deep venous thrombosis √

c- alternative positioning to avoid bed sources

d- respiratory ex

451- Most compression site of superfacial peroneal nerve is :

a- Lateral condyle of femur

b- Lateral head of fibula √

452- Type of cont which entire MS acting over two joint:

a- active insuffiency √

b- passive insuffiency

c- isometric cont

453- Type of ex which ms cannot stretch over two joint muscle at same time

a- active insuffiency

b- passive insuffiency √ sure

c- isometric cont
454- When entier muscle acting over tow joint:

a- active insuffiency √ ( any ms action be ACTIVE and any stretch be PASSIVE


)

b- passive insuffiency

c- isometric cont

455- Fracture of thumb affected :

a- Abductor policis longus ( and opponones pollicus because inserted in


metacarpal bone not inserted in proximal, middle and distal phalanx )

b- Extensor policis longus √

456- Monteggia fracture means which of the following ‫ ر‬# $‫آ‬

a- fracture of the radius and sublaxation of the lower end of ulna → Galeazzi
Fracture

b- fracture of the ulna with radial head sublaxation √

c- green stalk fracture with minor angulation

d- fracture of the radioulnar joint

457- Patient stay at hospital treated by corticosteroid then go to physiotherapist,


this patient will

have problem it is :

a- myopathy

b- osteoprosis √

c- deprssion
d- deacreas apeptite

458- OA's in old age due to:

a- traumatic

b- tear and wear √

459- Myotome is :

a- muscle supplied by certain nerve √

b- skin supplied by certain nerve

460- Dermatome is :

a- muscle supplied by certain nerve

b- skin supplied by certain nerve √

461- What is the closed-packed position of the shoulder?

a- Internal rotation and abduction

b- External rotation and abduction √

c- Internal rotation and adduction

d- External rotation and adduction

462- patient has amputation and use artificial limb during walking he takes
abduction gait: this may due to

a- limb is high

b- stress on adductor longus muscle

c- medial aspect is short

d- tight of gluteus medius √

463- when detect target heart rate in unstable angina we do

a- valsalva mannouver

b- isometric ex

c- isotonic ex √ not sure


d- heavy arm ex

464- Therapist measure target heart rate to determine

a- frequency of exercise

b- intenisty of exercise √ sure

c- duration of exercise

d- type of exercise

465- after pt do ex at gym. to check the tolerance of patient...we take heart rate
from

a- femoral artery 10 sec *6

b- brachial artery 15 sec*4

c- radial artery 30 sec * 2 √ sure

d- carotid artery 60 sec

466- patient with vascular impairment he not complain from intermittent


claudication ,,,the bed ex are

a- long period up ex

b- long period down ex

c- short period up ex √ →to avoid intermittent claudication

d- short period down ex → to assist venous return by effect of gravity

467- When evaluation of hip abduction of LT hip, it was poor grade ; so :

a- can apply full ROM of abduction in side lying

b- can apply full ROM of abduction in supine √

c- can apply half ROM of abduction in supine

d- can apply full ROM of abduction with max resistance in supine


468- patient has lung cancer and make right lung pneumoectomy this leading to
except

a- decrease vital capacity

b- decrease residual volume

c- increase tidal volume √

d- Rt shift of trachea

469- Baseball player the most commn inflamatory site is

a- bicipital tendinitis

b- suprasupinatus and subacromion bursa

c- a + b

d- rotator cuff √

470- which is not related to the shoulder triangle

a- Clavicle

b- Sternum √

c- humerus

471- which part not included in posterior neck triangle

a- clavicle

b- trapezuis

c- scaleni

d- sternum √
472- patient first time prosthesis wear with sever pain at stump sit what will
you do

a- ask pt not to wear prosthesis till meet prosthesis √

b- assure the pt it normal and he will adapt it

c- ask the pt to meet his physician

d- ask pt to meet prosthesis's with continue wearing it

473- The level of superficial abdominal reflex is?

Abdominal - above umbilicus (T8, T9, T10)

and below umbilicus (T10, T11, T12)

474- Acute adhesive capsulitis:

a- acute inflammation capsule with fibrous

b- chronic inflammation capsule with fibrous

c- capsule inflammation √ →Acute stage

d- fibrous capsule → Advanced stage

475- A 77 year-old female has a compression fracture at T12 secondary to


severe osteoporosis. The patient is active and alert though in some pain. To
help prevent further risk of compression fractures, elements of your therapeutic
program could include recommending:

a- active flexion to strengthen abdominals and use of a semi-rigid lumbosacral


brace

b- active flexion to strengthen abdominals and use of a dorsal-lumbar corset

c- active strengthening of back extensors and use of a dorsal-lumbar corset √

d- active strengthening of back extensors and use of a lumbosacral corset

476- Doctors see on patients door word(terminal illness) what does this mean

a- disease with eldery people

b- with adult people


c- irrevsable course √

d- neurologic origin

477- P.T perform LACHMAN TEST TO ASSESS

a- A.C.L √

b- M.C.L

c- P.C.L

478- Patient uses crutches and move the crutches forward and move his body to
the crutches the gait is

a- Swing to √

b- Swing through

c- Two point gait

479- with controlled diabetus mellitus adviced with exercise patient because

a- increase blood glucose uptake by ms √

b- NO effect

c- increase insulin metabolism


480- Shortness of which muscle cause limitation in shoulder abduction and
lateral rotation

a- pectoralis major √

b- serratus ant

c- sub scapularis

d- teres major

481- Pt cannot raise hand in abduction or mentain abduction position due to:

a- Adhesive capsulitis

b- Supra spinatous tendenitis

c- Rotator cuff tear √

482- ms spindle stimulated by :

a- passive stretch √

b- passive movement

c- active ex

483- manipulation of is described in the mangement of :

a- osteoprothesis

b- spinal dysfunction √

c- osteomylitis

484- precaution of U.S:

a- intramedullary nail√

b- joint stiffness

c- bursitis

485- site of pain of planter faciatis

a- ant to calcaneus √

b- post to calcaneus
c- lat to calcaneus

d- med to calcaneus

486- The most appropriate position to strength sternocleidomastoid is

a- supine

b- sitting √

c- prone

d- standing

487- long measurement of lower limb from :

a- ASIS to medial malleulus √

b- ASIS to lateral malleulus

c- greater trochanter to lat malleulus

488- use of can in contralateral hand will :

a- not affected gait pattern

b- reduce energy expendure in normal person

c- decrease the floor reaction force on the opposite foot √

d- not affected floor reaction force on the opposite foot

489- weakness in adductor ms to test it patient

a- supine position √

b- bside lying

c- prone lying

With gravity eliminated


490- Fracture due to repeated minor injury:

a- pathological

b- fatigue

c- stress √

d- compression

491- pt with neck pain and he couldn't extend his elbow bcoz its painful,, this
pain reffered to :

a- c5

b- c6

c- c7 √

d- c8

492- myotome of this biceps muscle is

a- C 3-4

b- C 5-6 √

c- T 1-2
493- 7 years old young boy, had FRACTURE of lateral condyle of femur. He
developed malunion as the FRACTURE was not reduced anatomically.
Malunion will produce:

a- Genu valgum √

b- Genu varum

c- Genu recurvatum

d- Dislocation of knee

494- pyramidal tract make decessation at

a- medulla √

b- midbrain

c- pons

d- cerebral cortex

495- ......................is resistance to passive stretch of muscle

a- flexibility

b- ms tone √

c- elongation of ma

496- T.E.N.S frequency

a- 10-70 HZ

b- 12-20HZ

c- 5-50Hz

d- 1-250 Hz √

497- A patient sustained injury to the upper limb 3 years back. He now presents
with valgus deformity in the elbow and paresthesias over the medial border of
the hand. The injury is likely to have been:

a- Supracondylar FRACTURE humerus

b- Lateral condyle FRACTURE humerus √


c- Medial condyle FRACTURE humerus

d- Posterior dislocation of the humerus

498- In valgus deformity of elbow there is stretch on

a- median

b- radial

c- ulnar nerve √

499- Obesity is a risk factor for

a- Osteoporosis

b- Osteoarthritis √

c- Both

d- Neither
500- paralysis of all intrinsic muscles of hand except ab po br

a- median n

b- ulnar n √

c- radial

501- main character at cardiac patient :

a- bluish limbs

b- pain at the calf

c- edema of limbs √

d- abdominal breathing

502- in radiating energy when we use it 30 inch from the patient then change it
to 15 inch so there will be

a- radiating energy will become 4 times √

b- no changes in it

c- will be increased 2 times

503- motor unit contain

a- Motor neuron and their axons & all m.s supplied √

b- Motor neucli in A.H.C & axons supply m.s

c- Motors neucli in P.H.C &axons supply m.s

504- complication of fracture rib is

a- Surgical emphysema √

b- Bronchitis

c- Bronchiactasis

505- with age the gait distributed:

a- swing phase increase above 40%

b- swing phase increased above 60%


c- stance phase increased above 40% √

d- swing phase decreased to below 60%

506- elderly person 65 year of age during his gait:

a- swing phase more than 40%

b- swing phase less than 40% √

c- swing phase 40%

507- Erb´s palsy branch nerve affected is

a- C5,C6 √

b- C 5,C6,C7

c- C8-T1

d- C5-T1

508- patient with shoulder pain during evaluation there is no passive or active
ROM in abd. and

rot. that may caused by :

a- frozen shoulder √

b- supraspinatus tendonitis

c- rotator cuff tear

d- acromioclavicular dysfunction
509- Right Ventrical of the heart

a- have bulk of muscle

b- have low oxygen blood √

c- have greater oxygen blood

d- neither of all

510- diaplegia mean

a- one limb affected

b- two limb affected

c- three limb affected

d- four limb affected and lower limb more than upper limb √

511- sever old osteoarthritis patient. You recommended for him

a- hot backs + exercise + walking

b- walkers √

; ‫ ا‬9 9. ‫اآ‬0 ‫ ا‬K L B‫ود !* ا آ‬0! ‫ ا‬4! 9 +9‫ إ‬43 ‫ك وا‬0! ‫ن‬0 453 , 8 9'
5)

c- cold backs + exercise + walking

d- cane

512- which current used for wound healing

a- TENS

b- faradic

c- diadynamic

d- high voltage galvanic √ → Ionophoresis


513- at the beginning of the muscle activity which make increase of blood
supply to it

a- sympathetic adrenergic √

b- sympathetic cholerigric

c- parasympathetic

514- during under water exercises which will be difficult

a- moving the limb horozantly

b- moving the limb toword bottom √ ( against bouyncy force of water )

c- moving the limb to the surface.

516- ulnar nerve give supply to

a- whole hand

b- outer3 1/2 fingers

c- inner 1 1/2 fingers √

d- posterior aspect of the hand

517- patient with C5 quariplegia on tilting table raised 50 degree during


transfere we used

a- hydrolic lifiting

b- sliding the patient √

c- two persons carry & transfere

518- pt referred to u suffering from bradykinesia that’s means :

a- abnormal involuntary movement

b- stereotype movement

c- slowness of movement √

d- in complete ROM
519- The angle between Neck of femur and Shaft of femur :

a- 90 Degree

b- 120 Degree √

c- 150 Degree

d- 170 Degree

520- Most compression site of superficial peroneal nerve :

a- lat condyle of femur

b- lat head of fibula √

521- P.T detect alarm signs for cardiac 60-patient during training program
through

a- monitor H.R AND B.P √

b- BLOOD OXYGEN LEVEL

c- o2 level

522- pulmonary ventilation means ‫ ر‬# $‫آ‬

a- gas exchange between alveoli and veinous blood

b- exchange between atmospheric air and lung air √

c- utilizing o2 by cells which essential for all vital process

523- phantom pain is due to :

a- amputation √

b- meniscectomy

c- neurotemesis

d- axonotemesis
524- patient with left above knee amputation complains of phantom limb
pain. simply that means

a- Dull aching pain at the thigh of the diseased side

b- distressing pain sensation felt by patients in the limb that is no longer there √

c- pain is described variously as cramping, squeezing, burning, sharp and


shooting

525- when you sit-up a patient from long sitting after prolonged period of bed
rest following major back surgery, the patient experience an orthostatic
hypotention and suddenly will fallwhat is your response?

a- immediately return patient to original long sitting √ sure

b- call the physician

c- write down this in patient's review

d- assure him it`s OK dont worry it`s normal decrease in BP due to a long rest
period

526- pt close his eyes and move his shoulder and then ask him about degree
and postion of limb so we test

a- light touch

b- fine touch

c- propioception √

527- meaning of rigidity is : ‫ ر‬# $‫آ‬

a- decreased ROM

b- Muscle stiffness √

528- patient came to clinic with pain and stiffness in morning means

a- systemic degenerative disease √

b- ms spasm

c- joint infection
529- A 65 years old pt with well treated colle's fracture. few months later came
with sever pain in hand with coldness color changes . x ray show big callus
around radial head This may be due to :

a- osteoarthritis of wrist

b- prearticular ossification

c- suedek's atrophy √

530- which nerve supplying Deltoid ms :

a- axillary √

b- suprascapular

c- long thoracic

d- musculocutanieus

531- patient complains of cystic fibrosis in LT lingula

a- put patient on RT sidelying √

b- put patient on LT sidelying

c- half lying

d- prone lying

532- patient during use of treadmill complains of chest pain the physiotherapist
call for cardiologist because he suspect angina pectoris as the pain was :

a- sever strapping pain in chest

b- pain at chest radiating to left shoulder &arm √

c- pain at apex of heart

d- pain in chest radiatig to both medial &lateral pectoral region

533- the most important factor to ttt endurance ‫ ر‬# $‫ آ‬:

a- intensity →increase Strength and power

b- duration of ttt →increase Strength and power

c- repetitions per set √


534- cardiac and normal person can achieve gaol from training ex by

a- decrease o2 consumption at max ex level

b- decrease o2 deficiency √

c- increase endurance and capacity for ex training

535- before provide ex program must do WARM UP its benefit

a- pooling of venous blood

b- increase O2 demand of heart

c- decrease 02 deficiency √

536- which action not needed during test of longesimus capitus

a- extension

b- flexion √

c- side bending

d- rotation

Action of longesimus capitus → extention and sidepending

537- Syme's amputation also called

a- supramalleolar amputation √

b- below knee amputation

c- above knee amputation

538- hemiplegic patient can't raise toes from ground on affected limb during
gait

a- dorsiflexor paralysis √

b- planterflexor paralysis

c - hip flexors paralysis

d - hip extensors
539- Which three muscles attach to the first cuneiform bone?

a-The anterior tibialis, the posterior tibialis,and the fibularis (peroneus)longus √

b- The extensor digitorum,the flexor hallucis,and the fibularis(peroneus) longus

c- The anterior tibialis, the peroneus brevis, and the fibularis (peroneus) longus

d-The flexor carpi ulnaris,the flexor pollicis brevis,and the flexor pollicis
longus

540- the ability of the posture to modify external inviroment and pneserve
aligment of one body part to another?

a- Postural control √

b- Strength

c- Balance

541- all of the following in the shoulder complex except ? ‫ ر‬# $‫آ‬

a- Sterno-claviculer joint

b- Acromio-clavcular joint

c- Gleno-humeral joint

d- Costo-strrnal joint √

542- patient complains of cystic fibrosis in LT lingual ‫ ر‬# $‫آ‬

a- put patient on RT side lying √

b- put patient on LT side lying

c- half lying

d- prone lying
543- Spondylolisthesis is ‫ ر‬# $‫آ‬

a- the anterior displacement of upper vertebra on below √

b- the posterior displacement of upper vertebra on below

c- the anterior displacement of lower vertebra on upper

d- the posterior displacement of lower vertebraon upper

544- Fracture dislocation L4-5 will result in :

a- Quada Equina

b- Flaccid Paraplegia √

c- Spastic Quadriplegia

d- Spastic Paraplegia
545- Pt of bells palsy long time treated with (E.S) you

a- M.S weakness

b- no change

c- fibrosis in the affected side √

546- protraction and retraction of shoulder occurs at

a- glenohumeral joint

b- scapulothoracic interface √

c- acromioclavicular joint

547- Application of intermittent traction to cervical depend on:

a- wt of patient

b- localization of pain

c- proper diagnoses and knowing problem √

548- P.t with neck flex dysfunction proper position of traction is

a- axial flex

b- slight flex √ not sure

c- neutral

d- axial ext

549- all of the follwoing are synovial joints except:

a- Elbow joint

b- Knee joint

c- Hip joint
d- Symphysis pubis √

550- the normal angle between the femur and the neck of femur is 126. when
the angle increase the deformity is:

a- coxa valgo √

b- genu Valgus

551- 45 years male presented with an expansile lesion in the centre of

femoral metaphysis. The lesion shows endosteal scalloping & punctuate

calcifications. Most likely diagnosis is:

a- Osteosarcoma

b- Chondrosarcoma √

c- Simple bone cyst

d- Fibrous dysplasia

551- Which of the following muscles can perform shoulder extension, internal
rotation and adduction:

a- Teres Major

b- Serratus Anterior

c- Pectoralis Major

d- Latissimus Dorsi √ →ms for swimming

552- An infant who can recognize tone of voice, sit independently and pulls –
to- stand through kneeling is most likely to be:

a- 2-3 Months of age

b- 4-5 Months of age

c- 6-7 Months of age

d- 8-9 Months of age √


553-Arteriosclerosis is:

a- A condition in which an artery wall thickens

b- A condition in which a vein wall thickens

c- Hardening (and loss of elasticity) of medium or large arteries √

d- Hardening (and loss of elasticity) of medium or large veins

554- Chronic inflammation of inner most layer of artries

a- thrombosis

b- plaque

c- atherosclerosis √

d- neuritis

555-Which of the following modalities is recommended for patients with


chronic Tennis elbow:

a- Splinting

b- Friction Massage √

c- Icing

d- TENS

556- all of the following muscles affected in tennis elbow except

a- Ext carpi radialis brevis

b- Ext carpi ulnaris

c- Ext digitorum

d- flex carpi ulnaris √

557- patient with prosthesis has excessive flexion during gait due to

a- rigid S.A.C.H ( Solid ankle cushioned heel ) √

b- SOFT S.A.C.H

c- the socket too big


558- Which of the following movements cannot be measured from supine:

a- Hip abduction

b- Hip external rotation

c- Hip extension √

d- Hip flexion

559- in treatment chondromalacia which not needed?

a- S.W.D

b- Manipulation

c- Bandaging

d- Strength hamstring √ sure

560- Which of the following measures/procedures are not of clinical


importance for DVT patients:

a- Frequent turning

b- Raising head above the level of the body √ sure

c- Wearing compressive socks

d- Passive range of motion exercises

561- All of the following are associated with supracondylar FRACTURE of


humerus, except:

a- It is uncommon after 15 years of age

b- Extension type FRACTURE is more common than the flexion type

c- Cubitus varus deformity commonly results following malunion

d- Ulnar nerve is most commonly involved √

562- Ex’s do with optimal performance and max capacity with well being is ?

a- Fitness √

b- Power
c- Co-ordiration

563- which action not needed during test of longssimus capitus

a- extension

b- flexion √

c- sidebending

d- rotation

564- When the knee is at its maximal amount of flexion during the gait cycle,
which of the following muscles are active concentrically?

a- Hamstrings √

b- Gluteus maximus

C- Gastrocnemius

d- all of the above

565- Which phase is not found in gait cycle :

a- initial contact

b- heel flat √

c- midswing

d- terminal swing

566- What stage of gait at which quadriceps produce maximum momentum


(torque) on normal gait

a-Toe off

b- Heel strike√

c-Foot flat

567- Patient with DM, the exercise benefit :

a- Not affect

b- Increase uptake of glucose by ms √


568- Pt with backache with herniated disc the least methods to approved:

a- MRI

b- CT

c- mylography

d- plain xray √

569- Which is correct about supra abdominal reflex innervation

a- T7 - T9 √

b- T6 - T8

c- T11 - T12

570- There are post contusion of the patient thumb, the least movement allow

a- radial and ulner deviation

b- thumb adduction and flexion √

c- extension and flexion of the wrist

d- supination and pronation

571- patient with depression of metatarsal pad and claw toe the clinical picture

a- Hyper ext. of metatarsal bone and extension of I.P.JT

b- Flex. of metatarsal bone and extension of I.P JT

c- Abd. of metatarsal bone and flexion I.PJT

d- hyper ext. of metatarsal bone and flexion of I.P JT √

572- Inversion injury at the ankle can cause all of the following except:

a- FRACTURE tip of lateral melleolus

b- FRACTURE base of the 5th metatarsal

c- Sprain of extensor digitorum brevis √ not sure

d- FRACTURE of sustentaculam tali


573- The therapist is ambulating a patient with an above-knee amputation. The
new prosthesis causes the heel on the involved foot to move laterally at toe-off.
Which of the following is the most likely cause of this deviation?

a- Too much internal rotation of the prosthetic knee√

b- Too much external rotation of the prosthetic knee

c- Too much outset of prosthetic foot

d- None of the above would cause this deviation.

574- A therapist is testing key muscles on a patient who recently suffered a


spinal cord injury. The current test assesses the strength of the long toe
extensors. Which nerve segment primarily innervates this key muscle group?

a- L2

B- L3

C- L4

D- L5 √

575- A patient asks the therapist to explain the function of his medication
verapamil (a calcium antagonist). Which of the following points should be
conveyed in the therapist's explanation?

a- Verapamil causes decreased contractility of the heart and vasodilation of the


coronary arteries √

b- Verapamil causes decreased contractility of the heart and vasoconstriction of


the coronary arteries

c- Verapamil causes increased contractility of the heart and vasodilation of the


coronary arteries

d- Verapamil causes increased contractility of the heart and vasoconstriction of


the coronary arteries
576- While assessing the standing posture of a patient, the therapist notes that a
spinous process in the thoracic region is shifted laterally. The therapist
estimates that T2 is the involved vertebra because he or she notes that it is at
the approximate level of the:

a- Inferior angle of the scapula

b- Superior angle of the scapula √

c- Spine of the scapula

d- Xiphoid process of the sternum

577- A patient comes to the therapist because she has noted a pronounced tuft
of hair on the center of her spinal column in the lumbar area. The therapist
notes no loss in motor or sensory function. This patient most likely has what
form of spina bifida?

a- Meningocele

b- Meningomyelocele

c- Spina bifida occulta √

d- None of the above

578- Persuading a sedentary patient to become more active, the therapist


explains the benefits of exercise. Which of the following is an inappropriate
list of benefits?

a- Increased efficiency of the myocardium to obtain oxygen, decreased high-


density lipoprotein (HDL) cholesterol, and decreased cholesterol √

b- Decreased low-density lipoprotein (LDL) cholesterol, decreased


triglycerides, and decreased blood pressure

c- Increased efficiency of the myocardium to obtain oxygen, decreased


cholesterol, and decreased LDL

d- Both Band C are inappropriate lists


579- When comparing the gait cycle of young adults to the gait cycle of older
adults, what would a therapist expect to find?

a- The younger population has a shorter step length

b- The younger population has a shorter stride length

c- The younger population has a shorter period of double support √

d- The younger population has a decrease in speed of ambulation

580- A therapist is treating a patient with a venous insufficiency ulcer over the
medial malleolus. The wound is moist and not infected. The involved lower
extremity is swollen, and the patient reports no pain around the wound. The
physician has ordered wound care 3 times a week. Which of the following is
the best treatment?

a- Warm whirlpool

b- Unna boot dressing between therapy sessions

c- Intermittent compression pump

d- B and C √

581- A 68-year-old man is being treated by a physical therapist after a right


below-knee amputation. The patient is beginning ambulation with a preparatory
prosthesis. In the early stance phase of the involved lower extremity, the
therapist notes an increase in knee flexion. Which of the following are possible
causes of this gait deviation?

a- The heel is too stiff √

b- The foot is set too far anterior in relation to the knee

c- The foot is set in too much plantarflexion

d- All of the above

582-Ex’s benefits for healthy & cardiac pt through which of the following ?

a- Increase product of H.R multi plying systolic B.P

b- Decrease product of H.R multi plying systolic B.P √

c- Increase collateral arteries and decrease inside wall cholesterol


.= " 3= ‫ ا‬101+"
0‫ا‬ ‫اآ‬BD‫ ؟ ه‬mucsle test for funtion and pain‫ ب‬D‫آ‬

‫ب‬ '$‫ ا‬WX;+D ‫ و‬actions only and muscle name ‫ت‬ 9 ‫ ا‬١٦١ ‫ ا > ص‬١٥٦ ‫ ص‬facial
‫ز‬$ ‫ ش‬E& ‫و‬

EX;+" W ‫ و‬+D

test‫ ا] ء ال‬E 1\D=D & ‫ آ ت ا‬+ ‫ ه ا‬0‫ وا‬7 E& '‫( ا [& و‬9‫ ا‬١٨٨ ‫ ا > ص‬١٨٤ ‫ص‬ neck

upper limb

٣٧٦ ‫ ا > ص‬٣٠٧ ‫ص‬

2 ‫اآ‬BD‫ '[& ه‬3)

action

effect of contracture

effect of weakness

test of shortening

lower limb

) ‫ ا‬1 ‫ ر " آ‬IDA‫ و ا‬a [ ‫و" ] ا‬ ‫ و " ] ا‬F‫آ‬$‫ ا ) م ` ا‬2; ٤٦٧ ‫ ا > ص‬٤٣٦ ‫ص‬
> ‫` ا‬$

٢١٣ ‫ص‬

back

‫>؟؟‬ 0‫ ا‬g;+D‫ ه‬2 f A W -DI ‫اع ' ( و‬1J &D " ? ED;d ‫( '[ ت‬A‫د ار‬ ‫ا‬ cI 9

١٢٢ ‫ ص‬١٢٠ ‫ ص‬١١٩ ‫ ص‬١١٨ ‫اول ص‬1- ‫ا‬

contracted and weak‫ض وا [ ت‬ ‫( ا‬9‫ ا‬7 ‫ول‬1- ‫ا‬ X;+D‫ ه‬i d 3‫ آ‬W

anatomical position of joints‫و‬

‫ ت‬+;J ‫د‬ ‫ ت‬+D $‫ا‬ E ‫ت‬ & ‫و‬

٨٤ ‫ ا > ص‬٨١ ‫ص‬ planes of body ‫ا‬1 (E

' ) 1A (=- ‫ا‬ 3J ; ‫اع ا‬ ‫ول‬1 ٣٦ ‫ص‬


‫اع ا [ ت‬ 9‫ ر‬٣٧ ‫ص‬

active passive insufficincy ) 1A ٤٤ ‫ ص‬٣٩ ‫ ص‬٣٨ ‫ص‬

muscle test position of body durin test

oxford grading scale for ms test٢٢ ‫ص‬


‫ا‬ ‫ا‬ ‫دآ‬ ‫ا‬ ‫ا‬
٢٠١٤ - ٩ – ٢٥ ‫ن م‬ ‫ا‬ ‫ا‬
1- Acromegaly is due to malfunction of :
a- Pituitary gland √
b- Thyroid gland
c- thymus

2- Which is not correct about oxford scale?


a- 0 = no contraction
b-1 = flicker contraction
c- 4 = can make full ROM against gravity √

3- the name of muscle cell is


a- sarcolemma √
b- actin
c- myosin

4- When o2 demand in heart decrease what happened ?


a- decrease HR multiplied by systolic pressure
b- decrease diastolic pressure √ + ‫آ‬$ .
c- increase coronary arteries
d- increase HR multiplied by systolic pressure

5- Above knee prothesis is long cause limb to do during gait :


a- vaulting √
b- lat bending
c- medial and lateral sweep
d-lordosis

6- From supine we can measure RoM of following excpet


A. Shoulder flexion
B. Shoulder Extension
C. Hip Extension √
D.Hip flexion
7-the term aphasia refers to?
a- speech problem √
b- swallowig
c- slow motion
d- involuntary movement
8- the marked impairment of production and comprehension of
language :

a-fluent aphasia
b-nonfluent aphasia
c- global aphasia √

9- patient has hand injury without open wound and making tight band
only after few 3 week referral to you and have contracture of hand and
wrist causes of these are :

a- Dupuytren’s contracture √ E 1‫ آ‬D W


b-Volkaman contracture

10- patient has shoulder dislocation and reduction for this dilocation and
referral to you for early mobilization to prevent :

a- Stiffness √
b- Recurent dislocation
c- Osteoarthritis

11- all of the following stance phase except :

a-Heel strike
b-Mid stance
c-Heel off
d-Acceleration √

12-which of following not needed in testing Semispinalis capitis


muscle which mov not needed
a-head flexion √

b-head extension
c-head rotation
d-head lateral flexion

13- in SCI which of the following in not of complications


a-hyperreflexia
b-systemic hypertention √
c-pressure sores
d-orthostatic hypotention
14- which of following not risk factor for osteoporosis:
a-enclosed family history
b- #in gym exercises √
c-low calcium intake
d-corticosteroid medications

15- the heart valve responsible in preventing blood from return to


right atrium from right ventricle is :
a-mitral or bicuspid
b-tricuspid √
c-pulmonary

16- what is the best electrical modality shosen to treat infective


wound :
a-Low Level laser
b-ultraviolet radiation √
c-galvainc stimulation
d-S.W.D

17- what is the best electrical modality chosen to treat knee with
fatty patient :
a-Low Level laser
b-ultrasonic √
c-galvainc stimulation
d-S.W.D

18- in normal milestone baby can walk indepedently


at :
a-12monthes
b-24 monthes
c-18 monthes √
d-30 monthes

19- location of SA node


A- top of rt atrium √
B- Top of lt atrium
C- Inter ventricular septum

20- role of AV node in heart contraction :

a-impulses reach atrium early to contract before


ventricles through √
21- raynaud’s disease
a- Bilateral asymmetrical
b- Unilateral symmetrical
c- Cyanotic unsymmetrical √

22- patient with long period of corticosteroid which complication of


corticosteroid :
a-Osteoprosis √
b-osteomyelitis
c-depression

23- disease in children and early childhood due to malfunction of


calcium – phosphorus metabolism

a-rickets √
2-osteomyelitis

24--for relaxation of parkinson's patient and to facillitat ex


which incorrect :
A. Rhithmic initiation
B. Rhithmic stabilization √
C. Sit pt on rocking chair

25-posterior neck triangle which is incorrect


A. clavicle
B. Sternoclidomastoid
C.sternum √
D. Upper trapezius

26- groin pain during abd ,external rotation


A. Add sprain
B. Add strain √

c.groin hernia

27-pt with sci there is weekness in his ankle plantar flexor to test ankle
plantar reflex
A. s1-s2 √
B. l1-l2
C. s2-s3

28- delayed ulnar nerve due to fracture of


a- olocranion fossa
b- medial epicondyle √
c- lateral epicondyle

29- choronaxiae definition :

a) Minimum time to depolarize the nerve when current intensity is


twice the rheobase √

30- all the following is true about motor neuron disease except
A- Affect anterior horn cell in spinal cord and cause lower motor
neuron lesion
B- Affect corticospinal tract and cause upper motor neuron
Lesion
C- Affect cranial nerve and Most common facial nerve affected
with lower motor neuron lesion
d-Affect brain stem and cause upper motor neuron
Lesion and cause lower motor neuron lesion √ E 1‫ آ‬D W
31- ask about number 2 is group1 and 2 afferent axon

32- muscle in picture :


e- Deltoid supplied by axillary nerve
f- Brachialis supplied by musclocutaneous nerve √
g- Supplied by suprascupular nerve
h- Rotator cuff

33- weakness and pain in insertion of this muscle

d- Supraspinatus tendinitis √
e- Rotator cuff tear
f- infraspinatus tendinitis
34- which of the following doesn"t contribute to stabilizers of
a-sternoclavicular J

b-interarticular disc

c-ant. and pos. sc ligaments

c-acromiclavicular ligament √

d- interclavicular ligament

35- electrical action potential through normal membrane:

a)-60 to -90

b)-30 to -90 √ E ‫ و‬- . ‫ و ا آ‬1‫ آ‬D W a

c)-20 to-70

d) -20 to -60

36- in carpal tunnel syndrome there is compression on


A-ulnar nerve
B-radial nerve
C- median nerve √
D- musculocutaneous nerve

37- physiological effects of warming up


a-Decrease oxygen deficiency √
b-Increase myocardial demands
c-pooling of venous return

nb : warming up Decrease oxygen deficiency and prevent


accumalation of lactic acid while cooling down exercise prevent
pooling of venous return

38-years old with burn of dermis,epidermis,and subcutaneous


tissues the type of burn
a-full thickness
b-partial thickness
c-full thickness √
d-superficial partial thickness

39- Burger’s disease associated with


a-smoking √
c- athlete performing max effort
d- the amount of blood reaching
e- d-skeletal muscle reaches

40- main aim in phase 1 cardiac rehabilition aim;


a-prevent bed rest complication begin with passive movement to
active movement √ 1‫ آ‬D W
b-increase ex capcity
c-assist in progress functional ADL

41- patient complains of cystic fibrosis in LT lingula


a-put patient on RT sidelying √
b-put patient on LT sidelying
c- half lying
d-prone lying

42- physiotherapist asks pt to flex the wrist and abduct it ; which


muscle acts in this action
a-flexor carpi ulnaris
b- flexor carpi radialis √
c-flexor digitorum profundus
d- biceps brachii

43- patien during use of tradmail complains of chest pain the


physiotherapist call for cardiologist because he suspect angina
pectoris as the pain was :
A sever strapping pain in chest
B pain at chest radiating to left shoulder &arm √
C pain at apex of heart
D pain in chest radiatig to both medial &lateral pectoral region

44- chronic inflammation of inner most layer of arteries is called


A thrombosis
B plaque
C atherosclerosis √
D neuritis

45- the type of exercise when training occur in a sequence


a-open packed position
b-closed packed position
c-curcit ex √
d- isometric ex

46- patient during walking raise his hip to clear toes from ground
due to
a-paralysis of dorsiflexors √
b-paralysis of planter flexors
c-paralysis of quadriceps

47- streching for which muscle

a)lateral rotator √
b)adductor
c)hamstring
d)calf muscle

48-patient with burn to dorsum of hand with chronaxi of 227


sec so you can use
a-faradic current
b-contiuse direct current
c-interrupted direct current √
d- TENS

49- shortening of muscle during contraction and can add


resistance:
a-concentric √
b-eccentric
c-isomtric
50- radial nerve injury due to
a-proximal part of humers
b-spiral groove of humers √
c-surgical neck of humers

51- Right convex Scoliosis we find


A- Lateral trunk flexion & concave side to RT
B- Lateral trunk flexion &convex side to RT √
C- Lateral trunk flexion &convex side to LT
D- Lateral trunk flexion & shorten RT musculature

52- indirect effect of heating with (E.S)


A- Vasodilitation
B- increase metabolic rate
C- muscle relaxation √
D- stimulation of sensory nerve ending

53- Patient with DM, the exercise benefit :


a- Not affect
b- Increase uptake of glucose by ms √ not sure
c-increase insulin metabolism
d-decrease insulin secretion

54- patient suffer diplopia from means


a-hemianopia
b-anomia
c- double vision √
d-agnosia

55- us with highest coupling media

a- aquasonic gel √
b- liquid paraffin
c-distelled water
d- air

56- benefit of isotonic exercise except :


a- high mechanical enegy
b- less resistance to fatigue
c-high efficiency
d-not delayed muscle soreness √ not sure

57- tremor :

a- fine type of involuntry movement


b-involuntry movement with twisting √ not sure

58- clonus :
a) response occur when make streching √ not sure

59- avoid passive streching in fracture elbow to prevent

a) myositis ossifican √

60- Pt has 32 years with spinal deformity & dyspnea due to :


a) Muscle weakness
b) Spinal orthosis
c) Decrease lung volume √

61- Ice massaging is contraindicated with:


a- Inflammation
b- Raynaud’s disease √
c- Muscle spasm
d- Acute burn:

62- Type of laser used in PT ttt


a) High
b) Soft
c) Mid √
‫ ا‬, D ‫ و ا‬1‫ وب آ‬- ‫ زي ا‬D ‫و‬ ‫ا‬
Soft
k ‫ > ا‬I0 ‫ > د‬I0 E) ‫ ا‬7&! 1?‫ال وا‬P9 ‫ي‬1 ' ‫و ا‬
Soft

63- For denervated ms we use at least ‫ ر‬# $‫آ‬


a- 10 m.sec
b- 50 m.sec
c- 100 m.sec √
d- 300 m.sec

64-to treat patient with deep heat, which is not form of deep heat
a- US
b- IR √
c- MWD
d- SWD

65-parkinsonism patient has complain of falling and decreased balance


what will you do
a- lowering COG √
b- decrease BOS
c- traction
d-passive movement

66- pt has osteoprosis with no fracture complain of mid and low


backpain and difficult breathing with any activity your advice is
a-trunk ext.and abdominal stabilizing ex √ not sure
b- trunk rotation and abdominal stabilization ex
c-trunk rotation and extention
g- trunk flex and extention

67-32 years old foot player come to the physiotherapist with pain at the
left knee joint while the
physiotherapist taking history patient said that the injury occurred when
he rotates to the right on
weight bearing to the left knee . it was flexed 60 degree. that means that
the injury is at
a- medial collateral ligament
b- medial semilunar cartilage at the knee √
c- femoral condyles
d- upper shaft of the tibia

68-sym's amputation :
a) supra malleolar √
‫ ا‬+D ‫ا‬ " ‫ دآ‬A‫ ا‬M&:9‫ا‬
2- to prevent knee effusion in fractur case without any active mov
A. Faradic
B. Tens
C.direct galvanic
D. Direct Interrupted current for long period

' ‫ا‬P9volkman ischemic contract


Affect flexors of forarm

10- lt cva pt has homonymous hemianopia .what can we do to compensate this problem
A. Help him to sleep on right side
B. Teach him to turn toward right side
C. Set a person to remind him to turn his head to right
D. Put food table ,telephon, ...on right side

9- irrevirsable risk factor in coronary artery disease


A. Smoking
B. High bp
C. Lipoprotine a
D. History of occlusive peripheral artery

8-orthotist a0 "

14-which ms when bilateral weeknes the pt walk with crutches


A.tensor facia lata
B. Gluteus max
C. Gluteus med

11- atnr 9‫ر‬


12- strech hip lat rotator 9‫ر‬

5-for relaxation of parkinson's patient and to facillitat ex .which incorrect


A. Rhithmic initiation
B. Rhithmic stabilization
C. Sit pt on rocking chair

15-spina bifida ...incomplet closure of vertebral body


16- not occur in stance phase
A. Deceleration
B. Heel off
C. In.c
D. Terminal stance

7-To test Semispinalis capitis which mov not needed


A.flex
B. Ext
C. Lat flex

6- anterior neck triangle which is incorrect


A. Mandible
B. Sternoclidomastoid
C.sternum
D. Upper trapezius

5-for relaxation of parkinson's patient and to facillitat ex .which incorrect


A. Rhithmic initiation
B. Rhithmic stabilization
C. Sit pt on rocking chair

3-ms spindle .which is incorrect?


A. Has capsule for each spindle
B. Dedict any tention in ms
C. Parallel with extrafusal ms fiber

4-resistance in pnf for


A. Facillitat and stimulate ms to contract
B. To increase motor contraction and motor learning
C. To make patient know direction of mov

pt has cruciate ligament injury a) Strength Quad b) hamstring c) Both

20-after fracture there is decrease rom.to return ms length we do stretch but gradwally and
slowly why?
A. To avoid ms and tendon tear

21- supracondylar ridg of humerus injury


A. Brachial artery
B. Ulnar n
C.median artery

18- groin pain during abd ,external rotation


A. Add sprain
B. Add strain
17- patient with abd scapula,round shoulder ,kyphosis ,ant rotation of pelvic
A. Lordosis
B. Sway back
C. Flat back
D. Kypholordosis

20-after fracture there is decrease rom.to return ms length we do stretch but gradwally and
slowly why?
A. To avoid ms and tendon tear

23-pt with sci there is weekness in his lower limb to test him
A. Give resistance to ms at middle rom
B. Test hip flexors l1-l2
C. Hip ext l3

53_ in poliomyliFs which is incorrect


A. Hyper reflex
B. Reflex absent
C. Ms weakness

50_to test righFng - equilibrium in child


A. Sudden push him from side
B. Turn his head
C. Tilt the child to one side

52_ malfuncFon of calcium lead to...

51_ hypotonia a0 "


‫ ا‬+D ‫ا‬ " ‫ دآ‬A‫ ا‬M&:9‫ا‬
1-paFent with long below knee protheFc his gait 1-vaulFng 2- circumducFon 3-hiking

$ below knee
$‫ ا‬I" ‫ ه‬vaulting

above knee

$‫ا‬ I" ‫ ه‬circumduction

2-Case of above knee amputation with neglectable hip flexor contracture refer to pt the best
way to
-passive stretch ex
-active stretch ex
-strength of agonist

‫ رات‬DA$‫ا‬ ` ‫آن‬2

Prone lying position with pillow below knee k ‫ ر ا‬DA$‫و د آ ن ا‬

‫ > دي‬I" ‫ دي‬passive strech ex

3-pt has osteoprosis with no fracture complain of mid and low backpain and difficult
breathing with any activity your advice is trunk ext.and abdominal stabilization ex/ trunk
rotation and abdominal stabilization ex/ trunk rotation and extention/ trunk flex and
extention

SIT-UPS

The sit-up is the standard abdominal exercise that many people perform on a regular
basis, however osteoporosis sufferers should avoid this movement. This dynamic
abdominal exercise puts extreme stress on the lower back and can result in fractures
of the lumbar spine. Crunches are a safer way to target the abdominal muscles without
causing injury.

TWISTING AT THE WAIST

Exercises that require bending forward at the waist or excessive twisting at the waist
such as golf, tennis or bowling should be avoided. These activities produce high
compressive forces in the spinal area and increase the vulnerability to fracture, ad

‫ آ م‬A‫ا‬ 0 a

48- ulnar nerve late palsy due to fracture of


a- olocranion fossa
b- medial epicondyle
c- lateral epicondyle
Nb---Neuropraxia , transient conduction block of motor and sensory function without nerve
degeneration

Neurotemesis, nerve damage, most serious nerve injury, both nerve and nerve sheath
disruption

Axonotemesis, distribution of axon and myelin sheath with preservation of connective tissue
fragments, results in degeneration of axon distal to the injury site

>&" >& ‫; ت ا‬0 D ‫ا‬


choronaxiae
pnf
cardiacoutput
SA node
Pnf is proprioceptive neuromuscular facilitation it is used to stimulate the
neuromuscular system in an effort to excite proprioceptos in order to produce a desired
movemet
2 years infant with spasFc cp but has developmental age of 6 months the
physiotherapist shoud start with him
1/reaching acFviFes from supine
2/reaching acFviFes from unsupported standing
3/reaching acFviFes from siOng wiyh trunk unsupported
4/acFviFes from pronelying posiFon

Answer 3

burn in body's response to thermal insult from external agent such as a)heat and
cold b)chemical c)electricity and radiation d)all of the above

answer : d

the first response in inflammatory process by : a) monocytes b) lymphocytes c)


leukocytes

answer: c

c correct answer or neutrofils because first 6 to 24 h neutrophils and from 24 to 48 h


by monocytes and lymphocytes

action potential : a)-60 to -90 b)-30 to -90


+30 to – 90

how much heart voltage

‫م‬ ‫ا‬

Lateral condyle fracture of the humerus will lead to valgus deformity

While supracondylar fracture will lead to varus deformity ( cubitus varus)


what is level of base of heart

Second intercostal space

which of the following doesn"t contribute to stabilizers of sternoclavicular J interarticular


disc/ ant. and pos. sc ligaments/ acromiclavicular ligament/ interclavicular ligament

answer : acromioclavicular

one joint hip flexors contructures lead to no effect on lumber spine/ ant.pelvic tilting and
lordosis/ant.pelvic tilting and kyphosis/ pos. pelvic tilting and kyphosis

one joint hip flexor means iliopsoas muscle and tightness of iliopsoas cause lordotic posture
( anterior pelvic tilting and lumbar lordosis ) and bilateral hip flexor contracture cause
lordotic posture and tightness two joint hip flexor rectus femoris and tensor facia latae
cause lordotic posture in kneeling position while one joint hip flexor means iliopsoas muscle
and tightness of iliopsoas cause lordotic posture in standing
correct answer is anterior pelvic tilting and lumbar lordosis

• ‫ت‬ ‫ر‬
ً‫ ء‬١٠:٥٥ & ‫ ا‬، ( ( ٧ · * (+ ‫أ‬


Marwa Mohey - .‫ ا‬planter reflex / dermatome of L4 / stretch of hip external
rotators / spinal cord

conginital hiip dislocation which movements are contraindicated flextion,abduction \


flection,adduction/ extention,abduction/ extention,adduction

answer : extension adduction

position of hip arthroplasty


abd. 30 with flex. maintained at 70_90
abd.20 with flex. maintained at 70_90
abd.10 with flex maintaINED at 70_90
abd.30 with flex. maintained at 20_90

may be d

pt has recent CVA one of these positions is contraindicated to him


1/subine with arm abducted and elbow extended
2/sidelying on sound side and affected side is the uppermost
3/subine with lateral flexFon of trunk toward affected side and turning head to the sound
side
4/siOng with trunk erect and supported

answer 3
‫! & ‪ %‬ح و ا& (‪ ! "#$% & '#‬ذن ا‬

You might also like